Está en la página 1de 162

EVALUACIÓN GENERAL

DE DIAGNÓSTICO 2009

MARCO DE LA EVALUACIÓN
MINISTERIO DE EDUCACIÓN
SECRETARÍA DE ESTADO DE EDUCACIÓN Y FORMACIÓN PROFESIONAL
DIRECCIÓN GENERAL DE EVALUACIÓN Y COOPERACIÓN TERRITORIAL

Instituto de Evaluación
C/ San Fernando del Jarama, 14
28002 Madrid, España
www.institutodeevaluacion.educacion.es

Edita:
© SECRETARÍA GENERAL TÉCNICA
Subdirección General de Documentación y Publicaciones

Catálogo de publicaciones del Ministerio:


educacion.es
Catálago general de publicaciones oficiales:
060.es

Fecha de edición: 2009


NIPO: 820-09-084-5
ISBN:978-84-369-4754-0

Depósito Legal: M-45046-2009


Imprime: Graficas Naciones, S.L.
EVALUACIÓN GENERAL
DE DIAGNÓSTICO 2009

MARCO DE LA EVALUACIÓN

MINISTERIO DE EDUCACIÓN
SECRETARIA DE ESTADO DE EDUCACIÓN Y FORMACIÓN PROFESIONAL
DIRECCIÓN GENERAL DE EVALUACIÓN Y COOPERACIÓN TERRITORIAL

Instituto de Evaluación
Madrid 2009
ÍNDICE

1. ASPECTOS GENERALES .................................................................................... 9

2. POBLACIONES Y MUESTRAS ............................................................................ 17

3. CONTEXTOS .................................................................................................. 21

4. PRUEBAS .......................................................................................................... 27

5. EL OBJETO DE LA EVALUACIÓN: LAS COMPETENCIAS BÁSICAS ...................... 37

6. ANÁLISIS DE RESULTADOS .............................................................................. 47

7. INFORMES Y DIFUSIÓN DE LOS RESULTADOS ................................................ 53

ANEXO I. INDICADORES Y VARIABLES DE CONTEXTO, RECURSOS


Y PROCESOS, SEGÚN FUENTES .......................................................... 59

ANEXO II. DESCRIPCIÓN DE LAS COMPETENCIAS BÁSICAS ................................ 61

ANEXO III. COMPETENCIA EN COMUNICACIÓN LINGÜÍSTICA............................ 71

ANEXO IV. COMPETENCIA MATEMÁTICA ............................................................ 101

ANEXO V. COMPETENCIA EN EL CONOCIMIENTO Y LA INTERACCIÓN


CON EL MUNDO FÍSICO .................................................................... 119

ANEXO VI. COMPETENCIA SOCIAL Y CIUDADANA .............................................. 139

Índice 7
1. ASPECTOS GENERALES
LAS EVALUACIONES GENERALES DE DIAGNÓSTICO

El marco legal

La Ley Orgánica 2/2006, de 3 de mayo, de Educación (LOE) prevé la colaboración entre el


Instituto de Evaluación y los organismos correspondientes de las Administraciones educati-
vas“ en la realización de evaluaciones generales de diagnóstico, que permitan obtener datos
representativos, tanto del alumnado y de los centros de las comunidades autónomas como
del conjunto del Estado. Estas evaluaciones versarán sobre las competencias básicas del
currículo, se realizarán en la Enseñanza Primaria y Secundaria e incluirán, en todo caso, las
previstas en los artículos 21 y 29. La Conferencia Sectorial de Educación velará para que
estas evaluaciones se realicen con criterios de homogeneidad” (artículo 144.1).

En los artículos 21 y 29 de la Ley Orgánica de Educación, se establece la realización de evalua-


ciones de diagnóstico, en la Educación Primaria (al finalizar el segundo ciclo) y en la Educación
Secundaria Obligatoria (al finalizar el segundo curso), respectivamente, indicando que “todos
los centros realizarán una evaluación de diagnóstico de las competencias básicas alcanzadas por
sus alumnos”. Esta evaluación será competencia de las Administraciones educativas y “tendrá
carácter formativo y orientador para los centros e informativo para las familias y para el conjun-
to de la comunidad educativa. Estas evaluaciones tendrán como marco de referencia las eva-
luaciones generales de diagnóstico que se establecen en el Artículo 144.1 de esta Ley”.

La Ley establece, además, que “corresponde a las Administraciones educativas desarrollar y


controlar las evaluaciones de diagnóstico en las que participen los centros de ellas depen-
dientes y proporcionar los modelos y apoyos pertinentes a fin de que todos los centros pue-
dan realizar de modo adecuado estas evaluaciones, que tendrán carácter formativo e inter-
no” (artículo 144.2), y “regular la forma en que los resultados de estas evaluaciones de
diagnóstico que realizan los centros, así como los planes de actuación que se deriven de las
mismas, deban ser puestos en conocimiento de la comunidad educativa” (artículo 144.3).

Aspectos Generales 9
Evaluación General de Diagnóstico 2009
Marco de la evaluación

Finalmente, la Ley prescribe que “En ningún caso, los resultados de estas evaluaciones podrán
ser utilizados para el establecimiento de clasificaciones de los centros” (artículo 144.3).

Evaluaciones de diagnóstico (censales) y evaluaciones generales de diagnóstico


(muestrales)

La LOE prevé dos tipos de procesos para las evaluaciones de diagnóstico.

Un primer proceso se centra en las evaluaciones generales de diagnóstico de carácter mues-


tral que “permitan obtener datos representativos, tanto de los alumnos y centros de las
comunidades autónomas como del conjunto del Estado” mediante la aplicación de prue-
bas externas a los centros seleccionados. “El Instituto de Evaluación y los organismos
correspondientes de las Administraciones educativas” colaborarán en la realización de
estas evaluaciones.

Un segundo proceso radica en las evaluaciones de diagnóstico de carácter censal que reali-
zarán todos los centros y “que tendrán carácter formativo e interno”. El desarrollo y el
control de estas evaluaciones corresponden, en el marco de sus competencias respectivas,
a las Administraciones educativas, que deberán “proporcionar los modelos y apoyos
pertinentes, a fin de que todos los centros puedan realizar de modo adecuado estas
evaluaciones”.

Tanto las evaluaciones generales de diagnóstico del sistema educativo como las evaluacio-
nes de diagnóstico versan sobre las competencias básicas del alumnado y deben dar lugar
a compromisos de revisión y mejora educativa a partir de los resultados. Las evaluaciones
generales de diagnóstico proporcionan datos representativos de las comunidades autóno-
mas y del conjunto del Estado, mientras que las evaluaciones de diagnóstico son evalua-
ciones formativas y orientadoras para los centros e informativas para las familias y para la
comunidad educativa. Las evaluaciones generales de diagnóstico del sistema educativo,
que requieren la coordinación estrecha entre todas las Administraciones educativas, son el
marco de referencia para las evaluaciones de diagnóstico, según establecen los artículos
21 y 29 de la LOE. Corresponde a las Administraciones educativas desarrollar el modo en
que las pruebas generales de diagnóstico puedan ser aprovechadas, en el caso de que lo
consideren oportuno, para las evaluaciones de diagnóstico que realizarán anualmente
todos los centros que impartan Educación Primaria o Educación Secundaria Obligatoria.

Las evaluaciones generales de diagnóstico deben contribuir a la mejora de la calidad de la


educación. Pretenden, por ello, el conocimiento de la situación del sistema educativo, a
través de la valoración de los aprendizajes de los estudiantes, y el impulso de procesos de
innovación y mejora de la educación en todo el sistema. A su vez, los resultados de las

10 Aspectos Generales
Evaluación General de Diagnóstico 2009
Marco de la evaluación

evaluaciones de diagnóstico deben facilitar a las Administraciones educativas la propuesta


de planes de mejora y la adopción de medidas específicas de apoyo educativo en el ámbi-
to de sus competencias; por su parte, los centros podrán tomar decisiones para la mejora
de la educación de sus alumnos a partir del análisis de los resultados por los consejos esco-
lares y por el profesorado.

En este documento, se presenta el marco de las evaluaciones generales de diagnóstico del


sistema educativo, de carácter muestral, que proporcionarán datos representativos del
conjunto del Estado y de las comunidades autónomas y que serán, según lo establecido
en los artículos 21 y 29 de la LOE, el marco de referencia de las evaluaciones de diagnós-
tico que llevarán a cabo anualmente todos los centros. Sin embargo, este documento no
es el marco teórico de estas evaluaciones de diagnóstico, competencia de las diferentes
comunidades autónomas en el ámbito de sus responsabilidades.

La finalidad de las evaluaciones generales de diagnóstico

Acordes con lo establecido en el artículo 140 de la LOE, las evaluaciones generales de diag-
nóstico del sistema educativo deben tener como finalidad contribuir a la mejora de la cali-
dad y la equidad de la educación, orientar las políticas educativas, aumentar la transparen-
cia y eficacia del sistema educativo y ofrecer información sobre el grado de adquisición de
las competencias básicas.

El objetivo inmediato de las evaluaciones generales de diagnóstico es obtener datos repre-


sentativos del grado de adquisición de las competencias básicas del currículo en Enseñanza
Primaria y Secundaria (artículo 144.1 de la LOE).

El objeto de las evaluaciones generales de diagnóstico del sistema educativo es, pues, el
conjunto de las competencias básicas del currículo. Estas competencias básicas se relacio-
nan con contenidos curriculares que suponen conocimientos, habilidades y actitudes
transferibles y útiles para hacer frente a situaciones y problemas que se presentan en la
vida real. En definitiva, se trata de valorar en qué medida la escuela prepara para la vida y
forma a los estudiantes para asumir su papel como ciudadanos en una sociedad moderna.
Las competencias básicas del currículo se refieren a las capacidades de los sujetos para uti-
lizar sus conocimientos, habilidades y actitudes en la comprensión de la realidad y en la
resolución de problemas prácticos planteados en situaciones de la vida cotidiana; en resu-
men, la aplicación de los conocimientos en un contexto determinado para la resolución
de un problema.

Para contribuir a la mejora de la calidad y la equidad de la educación, la evaluación


general de diagnóstico debe estimular en las comunidades educativas la necesidad de

Aspectos Generales 11
Evaluación General de Diagnóstico 2009
Marco de la evaluación

reflexionar sobre la práctica para encontrar evidencias rigurosas que les permitan com-
prender lo que hacen y lo que consiguen, es decir, para promover procesos de autoeva-
luación en las instituciones escolares. En consecuencia, el propósito global de las evalua-
ciones generales de diagnóstico es el conocimiento de la situación del sistema educativo
respecto a los aprendizajes de los estudiantes en las competencias básicas según la infor-
mación recabada y analizada, la comprensión de por qué y cómo se alcanzan los niveles
actuales y, sobre todo, el desencadenamiento de dinámicas sucesivas de transformación y
mejora.

Debe tenerse en cuenta que la mejora de la calidad y la equidad de la educación exige la


selección de variables de contexto que permitan iluminar la interpretación de los resulta-
dos y analizar las comparaciones de los subsistemas de las comunidades autónomas. La
evaluación general de diagnóstico del sistema educativo, además de general y homogé-
nea para permitir visiones de conjunto y comparaciones, debe ser equitativa, atenta a la
diversidad de contextos y sujetos. Por todo ello es necesario considerar los contextos socio-
culturales de alumnos y centros para poder explicar debidamente los resultados de la
evaluación; solo deben realizarse comparaciones en un marco contextual que contribuya
a explicar las diferencias.

La evaluación general de diagnóstico es una evaluación formativa para el conjunto del sis-
tema educativo, pues debe contribuir a su conocimiento, transformación y mejora y faci-
litar orientaciones para las políticas educativas, pero no es una evaluación de centros
docentes. Valora el grado de adquisición de las competencias básicas del currículo por el
alumnado en el sistema educativo en su conjunto. No está previsto que los resultados de
la evaluación general de diagnóstico informen sobre alumnos particulares o sobre centros
concretos, por lo que no tiene previsto entre sus propósitos ser utilizada para dar informa-
ción individualizada ni a estudiantes ni a centros.

La necesidad de coordinación entre las Administraciones educativas

Los artículos de la LOE relativos a la evaluación general del sistema educativo se refieren
reiteradamente a la cooperación entre las diversas administraciones implicadas. El proce-
so de evaluación diagnóstica y general del sistema ha de plantearse desde una perspecti-
va de colaboración, concertación y participación; por ello, es un proceso que exige la coor-
dinación entre las diferentes comunidades autónomas, es decir, no solo una planificación
homogénea sino la previa asunción compartida de los principios, finalidades y significados
de esta evaluación. Todas las administraciones han de tener parte en las decisiones y han
de llegar a unos acuerdos mínimos que faciliten la acción compartida.

12 Aspectos Generales
Evaluación General de Diagnóstico 2009
Marco de la evaluación

Para ello, la LOE prescribe la responsabilidad compartida entre el Instituto de Evaluación y


los organismos correspondientes de las comunidades autónomas. La evaluación general
de diagnóstico debe estar fundamentada en la colaboración y debe ser democrática, basa-
da en el diálogo y en la discusión con todas las partes interesadas.

EL MARCO DE LA EVALUACIÓN GENERAL DE DIAGNÓSTICO DEL SISTEMA EDUCATIVO

Antes de abordar la organización y puesta en marcha de las evaluaciones generales de


diagnóstico es necesario elaborar el correspondiente marco de la evaluación. La elabora-
ción de este marco fue encomendada por el Consejo Rector del Instituto de Evaluación a
un grupo de trabajo integrado por expertos en competencias básicas y en evaluación
educativa.

Contenido del marco de la evaluación

El marco de la evaluación general de diagnóstico del sistema educativo se ha estructurado


en los siguientes bloques:

1. Aspectos generales. Se describen los fundamentos legales de las evaluaciones


generales de diagnóstico, su finalidad, la relación con las evaluaciones de diagnós-
tico y el calendario de aplicación.

2. Poblaciones y muestras. Se detallan las poblaciones implicadas en las pruebas


generales de diagnóstico y los criterios para elaborar las muestras correspondien-
tes. El tamaño de las muestras debe asegurar la representatividad del alumnado
y de los centros de los resultados globales por comunidades autónomas, aunque
no por estratos dentro de cada comunidad.

3. Contextos. Es necesario considerar los contextos socioculturales de alumnos y cen-


tros para poder explicar debidamente los resultados de la evaluación y las diferen-
cias. En este sentido, la definición y la obtención de un índice socioeconómico y cul-
tural del alumnado y de los centros educativos facilitará la interpretación de los
resultados en sus contextos adecuados y las propuestas fundamentadas de mejora
educativa. Asimismo se recogerán datos sobre las variables de recursos y de proce-
sos que contribuyan, junto a las variables de contexto, a explicar los resultados.

4. Pruebas. Se explicitan los criterios técnicos que deben tenerse en cuenta para la
elaboración de las pruebas: tipo de ítems, longitud, tiempos de aplicación,
inclusión de preguntas abiertas, pautas para la elaboración de los ítems, etc.

Aspectos Generales 13
Evaluación General de Diagnóstico 2009
Marco de la evaluación

5. El objeto de la evaluación: las competencias básicas. Se aborda una breve des-


cripción de cada una de las competencias básicas, así como los conocimientos,
destrezas y actitudes relacionados con ellas. Esto supone el desarrollo orientado
hacia la evaluación de lo incluido en los Reales Decretos de enseñanzas mínimas
para la Educación Primaria y para la Educación Secundaria Obligatoria. También
se proponen ejemplos del desglose de cada competencia básica en los elementos
que la componen: dimensiones, subdimensiones u otros componentes.

6. Análisis de los resultados. Incluye los criterios para el análisis de los resultados,
la obtención de puntuaciones, la desagregación de los datos y la determinación
de niveles de rendimiento.

7. Informes y difusión. Se describen los tipos de informes según las audiencias a


las que van destinados y los procedimientos para la difusión de los resultados.

Proceso de elaboración del marco

El Consejo Rector del Instituto de Evaluación estableció, en febrero de 2007, las líneas bási-
cas de la organización y desarrollo de las evaluaciones generales de diagnóstico del siste-
ma educativo y encargó la elaboración del marco de la evaluación a un grupo de exper-
tos nacionales e internacionales integrado por profesores de universidad, directores de ins-
titutos de evaluación y especialistas en la evaluación de las competencias básicas.

Este grupo de expertos trabajó en contacto con los miembros del Grupo Técnico del
Consejo Rector desde marzo hasta julio de 2007. Una vez finalizado el trabajo de los exper-
tos, el Grupo Técnico supervisó el marco de la evaluación elaborado y lo elevó al Consejo
Rector para su aprobación.

Este procedimiento ha permitido, además de la colaboración de cualificados expertos, la


participación de todas las Administraciones educativas en la preparación del marco de la
evaluación. Finalmente, su aprobación por el Consejo Rector ha plasmado la estrecha cola-
boración entre el Instituto de Evaluación y las unidades responsables de las distintas comu-
nidades autónomas en la elaboración del marco de la evaluación general de diagnóstico
del sistema educativo.

Carácter revisable del marco de la evaluación general de diagnóstico

El marco de la evaluación general de diagnóstico no se concibe como un producto acaba-


do, sino como un instrumento en permanente revisión y mejora. La experiencia de los

14 Aspectos Generales
Evaluación General de Diagnóstico 2009
Marco de la evaluación

estudios piloto y de las sucesivas aplicaciones de las pruebas generales de diagnóstico per-
mitirá introducir las modificaciones y las correcciones oportunas en las siguientes versio-
nes del marco.

TEMPORALIZACIÓN Y CALENDARIO

Calendario de aplicación

La aplicación de las evaluaciones generales de diagnóstico se iniciará, una vez implantadas


las enseñanzas objeto de la evaluación, en el curso escolar 2008-2009.

En el curso 2007-2008, se han elaborado las baterías de ítems por los grupos de expertos
en la evaluación de las distintas competencias básicas y se ha llevado a cabo la aplicación
piloto de las pruebas generales de diagnóstico en 106 centros de Educación Primaria y en
84 centros de Educación Secundaria Obligatoria para valorar los requerimientos y los pro-
blemas específicos de aplicación de las pruebas. La experiencia obtenida en esta aplicación
previa servirá de base para llevar a cabo las pruebas en Educación Primaria y en Educación
Secundaria Obligatoria en 2009 y en 2010.

En el curso 2008-2009, se iniciarán las evaluaciones generales de diagnóstico del sistema


educativo en la Educación Primaria y en el curso 2009-2010 en la Educación Secundaria
Obligatoria. En ambos cursos se evaluarán las siguientes competencias: en comunicación
lingüística, matemática, el conocimiento y la interacción con el mundo físico, y social y
ciudadana.

En el segundo semestre de 2009, el Consejo Rector del Instituto de Evaluación establece-


rá el calendario de aplicación de las evaluaciones generales de diagnóstico, a partir
de 2011.

Fechas de la aplicación de las pruebas

Se ha analizado la conveniencia de realizar la prueba general de evaluación de diagnósti-


co antes o una vez finalizado el curso. Analizadas las ventajas e inconvenientes de distin-
tos calendarios de aplicación de las citadas pruebas, se considera necesario evitar la coin-
cidencia de dicha prueba con las evaluaciones del alumnado que los centros realizan a fin
de curso y, en este caso, se ha decidido realizar la aplicación antes de finalizar el curso.

Aspectos Generales 15
Evaluación General de Diagnóstico 2009
Marco de la evaluación

LA METAEVALUACIÓN DE LAS EVALUACIONES GENERALES DE DIAGNÓSTICO


DEL SISTEMA EDUCATIVO

El procedimiento establecido para llevar a cabo la evaluación general de diagnóstico debe-


rá ser debidamente revisado en cada una de sus fases, analizando no solo la validez de los
datos obtenidos y la información que ofrecen sobre el progreso del sistema, sino también
su contribución a la mejora de la educación.

Los resultados de esta metaevaluación serán cruciales para reestructurar, modificar y


mejorar el marco de la evaluación.

16 Aspectos Generales
2. POBLACIONES Y MUESTRAS
POBLACIONES

Las pruebas se aplicarán en todo el territorio español, considerándose las poblaciones cons-
tituidas por el alumnado que se encuentra matriculado en los niveles 4º de Educación
Primaria y 2º de Educación Secundaria Obligatoria respectivamente, en el curso escolar en
que se lleva a cabo la evaluación general de diagnóstico del sistema educativo.

Junto con las pruebas de evaluación se aplicarán los cuestionarios de contexto, mediante
los cuales se recoge información acerca de índices o variables relativos al contexto, los
recursos de los centros educativos y los procesos que se desarrollan en los mismos. La
información sobre estos aspectos se obtendrá a partir de poblaciones de profesorado y
directores/as escolares que se suman a las poblaciones de alumnado ya mencionadas.

En definitiva, para la aplicación de las pruebas de evaluación y de los cuestionarios de


contexto se extraerán muestras a partir de las poblaciones recogidas en la tabla siguiente.

Poblaciones consideradas en la Evaluación General de Diagnóstico

Poblaciones
Información
proporcionada
Evaluación en 4º de EP Evaluación en 2º de ESO

Alumnado de 4º de EP, de ámbito Alumnado de 2º de ESO, de Nivel alcanzado en competencias


estatal. ámbito estatal. básicas.
Variables de contexto y de procesos.
Profesorado del alumnado de Profesorado del alumnado de Variables de recursos y de procesos.
4º de EP. 2º de ESO.
Directores/as de centros escolares Directores/as de centros Variables de contexto y de recursos.
de ámbito estatal que imparten EP. escolares de ámbito estatal
que imparten ESO.
Familias del alumnado de 4º de EP. Variables de contexto y de recursos.

Poblaciones y Muestras 17
Evaluación General de Diagnóstico 2009
Marco de la evaluación

De acuerdo con la práctica habitual en este tipo de evaluaciones, quedan excluidos de la


población los alumnos y alumnas con discapacidad psíquica, minusvalía física o descono-
cimiento de la lengua de las pruebas que les impidan la realización de éstas. En este sen-
tido, no se considerará parte de la población el alumnado en el que concurra alguna de
las siguientes situaciones:

• Cuentan con adaptaciones curriculares significativas, y concurre en ellos alguna


discapacidad psíquica o minusvalía física que representa un obstáculo para la
realización de pruebas escritas u orales, según el caso.

• Proceden de otros países, contando con menos de un año de escolarización en


nuestro sistema educativo, y presentan un conocimiento insuficiente de la
lengua en que se redactan las pruebas.

MUESTRAS

A partir de la población de alumnado de 4º de Educación Primaria y de 2º de Educación


Secundaria Obligatorias, habrán de extraerse las respectivas muestras que completarán las
pruebas y responderán a los cuestionarios de contexto. Para ello, puede adoptarse un pro-
cedimiento de muestreo bietápico estratificado y por conglomerados, tomando como
estratos las diferentes comunidades autónomas y considerando conglomerados a los cen-
tros y, en su caso, dentro de éstos a los grupos de alumnado ubicados en una misma aula.
El muestreo se realizaría de acuerdo con las dos etapas siguientes, que son coincidentes
para las muestras de 4º de Educación Primaria y de 2º de Educación Secundaria
Obligatoria:

Primera etapa: Selección aleatoria de centros educativos en cada una de las comunidades
autónomas. A efectos de la estratificación de la población, se consideran las 17 comunida-
des autónomas junto con el territorio administrado por el Ministerio de Educación, Política
Social y Deporte. El número de centros en cada una de las comunidades puede quedar
fijado de forma que sea proporcional al tamaño de la subpoblación incluida en cada estra-
to. En cualquier caso, el tamaño de las muestras debe garantizar la representatividad nece-
saria al obtener resultados para el Estado en su conjunto y para cada una de las comuni-
dades autónomas.

A fin de garantizar la equiprobabilidad de todos los alumnos y alumnas para formar parte
de la muestra, la elección de los centros ha de tener en cuenta el tamaño de los mismos,
dando a cada uno de ellos una probabilidad de ser elegido proporcional al número de
alumnado escolarizado en los niveles educativos objeto de la evaluación.

18 Poblaciones y Muestras
Evaluación General de Diagnóstico 2009
Marco de la evaluación

Con el fin de no excluir al alumnado de pequeños pueblos o de centros rurales agrupados,


no se establecen límites sobre el tamaño mínimo que deben tener los centros para poder
ser elegidos en la primera etapa de muestreo.

Segunda etapa: En cada uno de los centros seleccionados en la etapa anterior, se elegirá
aleatoriamente, para centros con más de una línea, uno o dos grupos de alumnado de 4º de
Educación Primaria, o bien, en el caso de 2º de Educación Secundaria Obligatoria, un núme-
ro determinado de sujetos elegidos también al azar entre todos los que cursan este nivel en
el centro. Aunque pertenezcan a grupos de clase diferentes, este alumnado será reunido en
un mismo espacio en el momento de la aplicación de las pruebas.

A efectos de la selección de individuos, se considerará todo el alumnado que cursa el nivel


correspondiente (4º de Educación Primaria o 2º de Educación Secundaria Obligatoria) en el
centro. El procedimiento aleatorio que se utilice, eligiendo, según la etapa educativa, un
grupo de clase o un grupo de alumnado, podría determinar la selección de individuos que
se encuentren en alguna de las situaciones que motivan la exclusión de la población obje-
to de estudio. En el caso de 2º de Educación Secundaria Obligatoria, sería necesario la elec-
ción de sujetos adicionales en el centro para sustituir a este alumnado en la muestra final.
Opcionalmente, los centros podrían decidir la aplicación de las pruebas de diagnóstico al
alumnado excluido de la muestra por los motivos mencionados, si bien las respuestas de los
mismos no serán consideradas en el momento del análisis.

En cuanto al tamaño de las muestras, deberá determinarse con el propósito de que resulte sufi-
ciente para proporcionar resultados a nivel del Estado y de las comunidades autónomas. Así
mismo, habrá de tenerse en cuenta el procedimiento de muestreo, de tal manera que se con-
sidere alguna estimación del tamaño medio de los conglomerados. A partir de estos valores,
podrá fijarse el tamaño de las muestras, en número de alumnos y alumnas, y el número de cen-
tros que habrán de ser seleccionados en la primera etapa de muestreo. En cualquier caso, se ha
de tener en cuenta el valor del coeficiente de correlación intraconglomerados, de acuerdo con
estudios anteriores, y habrá de fijarse una confianza no inferior al 95,5% (2 sigmas) para
los resultados del conjunto del Estado, sin superar un error de estimación del 5% referido a
porcentajes.

El tamaño de las muestras estará condicionado, igualmente, por el diseño matricial adop-
tado para la aplicación de diferentes cuadernillos de las pruebas, teniendo en cuenta que
es necesario contar con un número mínimo de respuestas para cada ítem.

Por último, no debe excluirse la posibilidad de confeccionar submuestras en el supuesto de


que algunos de los instrumentos de evaluación presenten dificultades de aplicación objeti-
vas. Suele ser el caso de algunas subdimensiones de competencias que podrían ser evalua-
das, como la expresión oral, y también de otras como el tratamiento de la información y la
competencia digital, en las que puede ocurrir algo similar. Estas submuestras contarían con

Poblaciones y Muestras 19
Evaluación General de Diagnóstico 2009
Marco de la evaluación

menor tamaño y permitirían ofrecer resultados para el conjunto del Estado y no para las
comunidades autónomas.

Seleccionada la muestra de alumnado participante en la evaluación, quedarán automática-


mente determinadas las muestras de directores y directoras y del profesorado. Formarán
parte de la muestra de directoras y directores todos aquellos que ejercen sus funciones
directivas en los centros elegidos. La muestra de profesorado quedará integrada en
Educación Primaria por los tutores y tutoras de los grupos de alumnado seleccionados, y en
Educación Secundaria Obligatoria por el profesorado que imparta docencia a los alumnos
y alumnas de la muestra, incluido el tutor del grupo.

20 Poblaciones y Muestras
3. CONTEXTOS
INTRODUCCIÓN

Las evaluaciones generales de diagnóstico tienen como uno de sus propósitos prioritarios el de
contribuir a la mejora del sistema educativo. Para poder mejorar la calidad y la equidad, y para
poder orientar adecuadamente las políticas educativas, hay que conocer y comprender el fun-
cionamiento general del sistema educativo. El procedimiento que se utiliza a través de evalua-
ciones externas es el del estudio de los resultados obtenidos por los alumnos, los contextos de
todo tipo que condicionan dichos resultados y el funcionamiento y organización de los centros
educativos, así como de los procesos de aula que los hacen posibles.

Consecuentemente, en las evaluaciones generales de diagnóstico del sistema educativo


habrá de recogerse información relativa a los resultados de los alumnos, los contextos y las
características específicas de los centros.

La investigación educativa ha revelado desde hace varias décadas que los resultados en
pruebas de rendimiento están modulados tanto por factores contextuales como por facto-
res relativos a procesos organizativos y/o de aula.

Aunque en algunos marcos conceptuales, por ejemplo, el utilizado en TIMSS (Third


International Mathematics and Science Study desarrollado por la IEA), se habla de factores
contextuales incluyendo a ambos (procesuales y contextuales), parece más oportuno,
desde una perspectiva de mejora y pensando en los destinatarios de los informes, diferen-
ciarlos. Tanto factores de contexto como de procesos (de centro y aula) se incluyen en los
marcos de evaluación de estudios internacionales como el proyecto PISA, el TIMSS y el
PIRLS (Progress in International Reading Literacy Study), y de estudios nacionales como la
evaluación de la Educación Primaria y la evaluación de la Educación Secundaria Obligatoria.

Por otro lado, la LOE también establece que las pruebas generales de diagnóstico deben
permitir obtener datos representativos tanto del alumnado como de los centros educati-
vos de las comunidades autónomas y del conjunto del Estado (artículo 144). Para que el

Contextos 21
Evaluación General de Diagnóstico 2009
Marco de la evaluación

análisis de los resultados pueda contribuir a la mejora del sistema educativo, es imprescin-
dible recoger información del contexto de las diversas comunidades que permita com-
prender las diferencias en los resultados obtenidos por el alumnado de cada una de ellas.

VARIABLES DE CONTEXTO Y DE RECURSOS

El contexto, los procesos y los recursos de la educación suelen evaluarse mediante cuestio-
narios específicos que incluyen múltiples y variadas preguntas concretas a los estudiantes,
los directores, los profesores y, en su caso, las familias. Las respuestas a estas preguntas
constituyen las variables primarias que describen los contextos, los procesos y los recursos
de la educación y el aprendizaje.

Tomando como referencia las experiencias de evaluación, tanto en el ámbito internacional


como local antes señalado, es muy aconsejable utilizar como elemento definitorio del con-
texto de los centros algún tipo de índice socioeconómico y cultural (ISC). Éste será cons-
truido a partir de variables relativas al nivel de estudios y la situación laboral o profesión
del padre y de la madre, y a los recursos culturales en el hogar.

El ISC se puede utilizar a dos niveles: ISC de cada alumno/a considerado individualmente
e ISC medio del alumnado del centro. La experiencia en investigación y evaluación educa-
tiva ha mostrado que lo que incide en los resultados no es tanto el origen social individual
del alumno en sí mismo, sino la composición social del grupo o del centro. Una de las con-
clusiones de PISA 2000 es, precisamente, que la composición social de la población de
estudiantes de una escuela es un mejor predictor del rendimiento de los alumnos que el
entorno social individual. Eso significa que el ISC medio del alumnado del centro combi-
nado con el individual parece explicar más las diferencias entre el alumnado que el ISC
individual por sí solo. Ambos índices -individual y del centro- se pueden utilizar en las com-
paraciones entre comunidades autónomas.

El ISC será tipificado y sería recomendable utilizar la misma escala que PISA por encontrar-
se parte del público ya familiarizado con ella. Las puntuaciones de los resultados de las
pruebas de competencias básicas y las de este tipo de índices tipificados deben expresar-
se en el mismo tipo de escala.

En todo caso se trata de utilizar procedimientos que contribuyan a la mayor equidad posi-
ble en las comparaciones y permitan un mayor poder explicativo de las diferencias entre
comunidades autónomas en los resultados de las pruebas de competencias básicas.

Además de las variables constitutivas del ISC es procedente incluir otras variables e índices
contextuales. Se proponen las siguientes (ver anexo I):

22 Contextos
Evaluación General de Diagnóstico 2009
Marco de la evaluación

• Titularidad.
• Tamaño del centro.
• Alumnado inmigrante (origen, lengua, tiempo en España).
• Alumnado con necesidades educativas especiales.

En el ámbito de los recursos, se proponen las siguientes variables (ver anexo I):
• Plantilla (ratio, experiencia y formación docente).
• Inversión en educación (gasto público educativo en relación al PIB, gasto por
estudiante en relación con la renta PER CÁPITA).

En el ámbito de los procesos, se proponen los siguientes conjuntos de variables (ver anexo I):
• Interés e implicación de las familias en la educación de los hijos.
• Tutoría.
• Tiempo de aprendizaje.
• Clima escolar.
• Actividades didácticas.
• Materiales didácticos.
• Procedimientos de evaluación.
• Liderazgo pedagógico.

En el ámbito de los resultados se proponen, además de las puntuaciones en las pruebas, las
siguientes variables (ver anexo I):
• Desfase.
• Repetición.

Finalmente, los cuestionarios de contexto deben contemplar una parte abierta para ser
completada a la carta por cada comunidad autónoma, para recoger aspectos concretos que
puedan ser de interés en una comunidad y no en otras.

FUENTES DE INFORMACIÓN E INSTRUMENTOS DE CONTEXTO Y DE RECURSOS

Los aspectos de contexto se suelen recoger, por medio de cuestionarios, a equipos directi-
vos y alumnado. En el caso de las evaluaciones generales de diagnóstico, en Educación
Secundaria Obligatoria puede ser suficiente y fiable la información del contexto ofrecida por
el alumnado. Sin embargo, en Educación Primaria se corre el riesgo de que la información
de contexto ofrecida por al alumnado no sea muy fiable o, en todo caso, no sea todo lo fina
que es necesario.

Contextos 23
Evaluación General de Diagnóstico 2009
Marco de la evaluación

Aunque, en general, se considera necesario reducir el número de cuestionarios y limitarse


al alumnado, a la dirección del centro y a los profesores de la muestra, en el caso de 4º de
Educación Primaria, dada la edad del alumnado, es conveniente la incorporación de un
cuestionario a familias para recoger la información relativa al nivel de estudios y situación
laboral de los padres y recursos del hogar.

Recoger la información de contexto por medio de cuestionarios a las familias tiene el ries-
go de que la tasa de respuestas sea baja. Sin embargo, existen procedimientos contrasta-
dos que contribuyen al logro de tasas de respuesta en las familias más que aceptables,
como los aplicados por el Instituto de Evaluación en la evaluación de la Educación Primaria
(tasa de respuestas: 89% en 1999 y 86% en 2003) y en la evaluación de la Educación
Secundaria Obligatoria (tasa de respuestas: 72% en 2000).

En todo caso, el principio que debe aplicarse es el de seleccionar un reducido número de


preguntas que permitan recoger la información de contexto y procesos que se considera
imprescindible para poder controlar el influjo de diversas variables de esa naturaleza (con-
textual y procesual) en los resultados del alumnado en las pruebas de competencias bási-
cas. Tampoco se trata de que todos los agentes contesten las mismas preguntas, aunque
puede haber (y sería conveniente que hubiera) algunas comunes.

Algunos de los datos relativos a las circunstancias del centro y otros aspectos generales se
obtendrán de los indicadores disponibles.

VARIABLES DE PROCESOS

Esta dimensión es la más compleja de medir, pues habitualmente se suele realizar recaban-
do la opinión de los agentes implicados. Se proponen los siguientes conjuntos de variables
de procesos (ver anexo I):

• Interés e implicación de las familias en la educación de los hijos.


• Tutoría.
• Tiempo de aprendizaje.
• Clima escolar.
• Actividades didácticas.
• Materiales didácticos.
• Procedimientos de evaluación.
• Liderazgo pedagógico.

24 Contextos
Evaluación General de Diagnóstico 2009
Marco de la evaluación

En cuanto a las fuentes de información e instrumentos para las variables de proceso, como
no es posible recoger datos consistentes de los procesos mediante observaciones a lo largo
del tiempo, los datos recogidos por medio de cuestionarios tienen que ver con percepcio-
nes de los agentes implicados, y como tales conviene tratarlos.

EN RESUMEN

Para que la evaluación contribuya a mejorar la calidad y la equidad de la educación y per-


mita orientar las políticas educativas para aumentar la eficacia y la transparencia del siste-
ma educativo, se requiere que ésta no se limite a describir los resultados al margen de los
factores que influyen o interactúan con ellos.

Para poder explicar debidamente los resultados es necesario considerar los contextos
socioculturales de alumnado y centros, los recursos educativos y los procesos que contri-
buyen a ellos.

Por ello, se considera necesario acompañar la información sobre los niveles competencia-
les del alumnado de datos de contexto, de procesos y de recursos obtenidos a través de
cuestionarios destinados al alumnado, al profesorado, a la dirección y, en su caso, a las
familias.

En esta primera fase de la evaluación general diagnóstica se ha considerado conveniente


evitar ampliar excesivamente el foco, reduciendo en la medida de lo posible el número de
indicadores, variables y tipos de instrumentos.

Las variables seleccionadas que se proponen son aquellas que en los estudios
internacionales y nacionales han mostrado un mayor poder explicativo.

Contextos 25
4. PRUEBAS
CARACTERÍSTICAS GENERALES

La evaluación de competencias básicas en el alumnado de Educación Primaria y Educación


Secundaria Obligatoria requiere el empleo de instrumentos que incluyan ítems adecuados al
tipo de competencias que han sido consideradas, que tengan en cuenta los contextos o situa-
ciones definidos para que los sujetos demuestren su dominio y aplicación, y cuya administra-
ción resulte viable en el marco de una evaluación general de diagnóstico del sistema educativo.

La evaluación de los aprendizajes del alumnado se ha abordado a través de una variedad


de técnicas aplicables en el aula. Al evaluar competencias, los métodos de evaluación más
adecuados han sido los que basan la valoración en la información obtenida a partir de las
respuestas del alumnado ante situaciones que exigen la aplicación de conocimientos. En el
caso de determinadas competencias, se requeriría una observación directa del desempeño
del alumno, como ocurre en evaluación de ciertas habilidades manipulativas (por ejemplo,
manejo de SOFTWARE para solucionar problemas), actitudes (hacia la lectura, la resolución
de problemas, etc.) o valores (perseverancia, minuciosidad, etc.). Y, en general, el grado en
que un alumno ha desarrollado las competencias básicas podría ser determinado a través
de procedimientos, como la resolución de problemas, la realización de trabajos y activida-
des prácticas, las simulaciones, la elaboración de portafolios, o mediante la elaboración de
balances de competencias a partir de la autoevaluación o de entrevistas con el sujeto.

No obstante, todas estas aplicaciones resultan complejas cuando se trabaja con muestras
amplias de sujetos. Las limitaciones que surgen en este sentido llevan a la utilización de
pruebas escritas administradas colectivamente, que constituyen el procedimiento habitual
de las evaluaciones nacionales e internacionales que vienen realizándose sobre el rendi-
miento del alumnado. Junto a las pruebas de lápiz y papel, la evaluación general de diag-
nóstico del sistema educativo puede recurrir a pruebas orales o pruebas que combinan la
presentación oral con la respuesta escrita, dirigidas a valorar respectivamente la expresión
oral y la comprensión oral de los sujetos.

Pruebas 27
Evaluación General de Diagnóstico 2009
Marco de la evaluación

En el caso de las pruebas que requieren la emisión de respuestas orales por parte de los
alumnos y alumnas evaluados, la administración ha de ser individual, con el consiguiente
incremento de tiempo y esfuerzo. En este sentido, la aplicación de tales pruebas se
realizaría para submuestras de alumnado extraídas de la muestra utilizada en la evaluación,
a fin de reducir el número de sujetos con los que se trabaja.

Las diferentes competencias objeto de evaluación podrían llevar a la necesidad de plantear


diferentes tipos y formatos de ítems. No obstante, todos ellos deberían tener en común su
orientación hacia la evaluación de competencias básicas, sin centrarse exclusivamente en
conocimientos y técnicas propios del currículo aplicado en cada contexto escolar. Las cues-
tiones, por tanto, basándose en los contenidos curriculares, tratarán de evaluar las capaci-
dades de los sujetos para la aplicación de los mismos a situaciones escolares y, especialmen-
te, a situaciones en las que el individuo habrá de desenvolverse en la vida real.

Un formato que han adoptado con frecuencia las pruebas de lápiz y papel empleadas a la
hora de llevar a cabo evaluaciones generales de rendimiento del alumnado es el de prueba
objetiva, a la que se han atribuido ventajas indudables. Una prueba objetiva puede constar
de un número amplio de ítems, por lo que es posible cubrir con mayor exhaustividad el
objeto de evaluación. En la práctica, posiblemente los principales atractivos de una prueba
objetiva son la ausencia de subjetividad, o la rapidez y facilidad para la calificación, que la
hacen especialmente adecuada en situaciones en las que el número de alumnos evaluados
es alto y se pretenden resultados comparables.

Aunque diferentes formatos de preguntas pueden ser utilizados (ítems de verdadero-falso,


de emparejamientos, de completar frases, de respuesta breve), el formato más recomenda-
ble de las preguntas que forman parte de una prueba objetiva es sin duda el ítem de elec-
ción múltiple, que sitúa al alumno ante varias opciones de respuesta para que elija entre
ellas la respuesta correcta.

Si bien el uso de pruebas objetivas conlleva una serie de ventajas entre las que se encuen-
tran las que se han destacado anteriormente, también se han señalado límites a su utiliza-
ción. Así, un problema estaría en el tipo de aprendizaje que permiten medir. Se ha afirma-
do que las pruebas objetivas se prestan a la evaluación de aprendizajes simples (definicio-
nes, conceptos, recuerdo de datos, reconocimiento de hechos, etc.), siendo menos adecua-
das para evaluar aspectos reveladores de un conocimiento profundo, tales como capacidad
de análisis, reflexión y crítica, aplicación de conocimientos, desarrollo de procedimientos,
técnicas y, en definitiva, para la valoración de competencias básicas del tipo de las defini-
das como objeto de la evaluación general de diagnóstico del sistema educativo.

Tratando de salvar estas limitaciones, para llevar a cabo las evaluaciones generales de
diagnóstico se emplearán pruebas en las que se combinen diferentes formatos de ítems,

28 Pruebas
Evaluación General de Diagnóstico 2009
Marco de la evaluación

incluidos los propios de una prueba objetiva, para conformar instrumentos que se pueden
denominar pruebas escritas basadas en situaciones-problema. Se trata de pruebas constitui-
das a partir de casos o situaciones que sirven como base para la interrogación, y que en la
medida de lo posible, remiten a situaciones similares a las que el alumnado puede encon-
trar en su vida escolar o extraescolar.

La redacción de ítems se apoya en la consideración de diferentes tipos de situaciones o


contextos, que permiten abordar la evaluación de la competencia del alumnado para
aplicar con éxito los conocimientos y habilidades adquiridos.

Las preguntas formuladas a partir de cada situación-problema irán dirigidas, en líneas genera-
les, a comprobar el grado de dominio de las competencias objeto de la evaluación general de
diagnóstico del sistema educativo, es decir a la capacidad para transferir conocimientos y habi-
lidades de diferentes ámbitos curriculares aplicándolos a los problemas planteados en una
diversidad de situaciones. Cada situación o caso representa un estímulo a partir del cual se plan-
tea un racimo de cuestiones que podrán encuadrarse en algunos de los siguientes formatos:

• Preguntas de respuesta cerrada, bajo el formato de elección múltiple. Se trata


tanto de preguntas de respuesta dicotómica, en las que sólo una opción es correc-
ta y las restantes se consideran erróneas, como preguntas con escala de respues-
ta graduada, de tal manera que cabe pensar en una respuesta correcta, una o más
respuestas parcialmente incorrectas y una o más respuestas totalmente erróneas.

• Preguntas que exigen el desarrollo de procedimientos y la obtención de


resultados. Este tipo de cuestiones contempla generalmente la necesidad de
alcanzar un resultado único, aunque podrían describirse diferentes caminos para
llegar al mismo. Tanto el procedimiento como el resultado han de ser valorados,
posibilitando el establecimiento de diferentes niveles de corrección en la respues-
ta en función del grado de desarrollo competencial evidenciado.

• Preguntas abiertas que admiten respuestas diversas, las cuales, aun siendo
correctas, pueden diferir de unos sujetos a otros. Para reducir la subjetividad a la
hora de la calificación, las preguntas de este tipo han de ser valoradas por más de
un juez e ir acompañadas de una plantilla de corrección, de tal manera que se
intente cerrar las posibles respuestas a la pregunta estableciendo cuáles se consi-
derarán correctas y cuáles no. La elaboración de criterios de corrección permite
graduar las respuestas estableciendo, también en este caso, niveles de ejecución
intermedios entre las respuestas correcta e incorrecta.

La inclusión de preguntas abiertas en las pruebas habrá de tener en cuenta la complejidad


que entraña su utilización, especialmente por la posibilidad de interpretaciones diversas de
los criterios de corrección y, sobre todo, por el coste que supone la corrección de las mismas.

Pruebas 29
Evaluación General de Diagnóstico 2009
Marco de la evaluación

La división de la prueba en diferentes cuadernillos aplicados a grupos diferentes de alum-


nado permite incluir en la misma un mayor número de ítems. De este modo, se estima el
nivel de desarrollo logrado en las competencias básicas considerando todos los ítems, tanto
a nivel del Estado como de las respectivas comunidades autónomas. En cualquier caso, el
diseño matricial deberá garantizar que los ítems están adecuadamente agrupados, equili-
bradamente distribuidos en los diferentes cuadernillos y ubicados en diferentes lugares de
los mismos para poder evaluar la relación entre su posición en los distintos modelos de cua-
dernillo y su dificultad.

El diseño adoptado precisará el número de reactivos de diversos formatos considerados en cada


prueba, los bloques que se formarán a partir de éstos y los cuadernillos que finalmente se mane-
jarán en la aplicación, con cierta combinación de bloques que permita contar con ítems de
anclaje a partir de los cuales integrar los resultados de los alumnos en una misma escala.

ELABORACIÓN DE LAS PRUEBAS

Para cada una de las competencias evaluadas, se parte de una matriz de especificaciones
que facilite la cobertura de todas las dimensiones o componentes de la competencia con-
siderados, abordándolas desde los grandes bloques de contenido que están en la base de
la estructura adoptada por el currículo escolar. Si bien desde las diferentes áreas del currí-
culo se contribuye al desarrollo de las competencias básicas, al considerar los bloques de
contenido que están presentes en las matrices de especificaciones han de estar especial-
mente representadas las áreas más estrechamente vinculadas a cada una de las competen-
cias objeto de evaluación.

Partiendo de las respectivas matrices de especificaciones, el proceso de elaboración de las


pruebas ha de cubrir los siguientes pasos:

• Redacción de ítems por personal cualificado y debidamente capacitado.


• Revisión de ítems por personal cualificado, diferente de quienes los
redactaron, para validación y verificación a priori de posibles sesgos (cultural-
lingüístico), de género, etc.
• Estudio de las características de los ítems a partir de una aplicación piloto.
• Selección de ítems que serán incluidos en las pruebas, tomando como criterios
la revisión realizada y la aplicación piloto, a las que aluden los dos puntos
anteriores.
• A partir de los ítems seleccionados, configuración de las pruebas y cuadernillos
de aplicación, según el diseño matricial adoptado.

• Diseño gráfico de las pruebas, de acuerdo con el formato que se decida


adoptar.

30 Pruebas
Evaluación General de Diagnóstico 2009
Marco de la evaluación

Redacción de ítems

El proceso de redacción de ítems se apoyará en el análisis de las competencias objeto de


evaluación y en su desglose en dimensiones y componentes, y tendrá en cuenta los crite-
rios de gradación de cada competencia en niveles, así como lo recogido en las correspon-
dientes matrices de especificaciones (ver apartado de este marco de la evaluación relativo
a la descripción de cada una de las competencias básicas).

Para la redacción de ítems se recurrirá a la colaboración de un grupo de expertos en los


ámbitos de competencia y etapas evaluados, incluyendo profesorado, asesores, miembros
de la inspección educativa y participantes en la elaboración de los diseños curriculares.
Conviene que entre el profesorado participante se incluyan también profesores y profeso-
ras de distintas especialidades.

Las pruebas de evaluación someterán al alumnado a situaciones-problema ante las que


deberá demostrar en qué grado es capaz de activar sus conocimientos, habilidades, actitu-
des o valores. Las situaciones elegidas para recoger evidencias sobre el nivel de competen-
cias de los alumnos irán referidas a realidades relativamente cercanas al alumno, similares a
la que podrían plantearse al desenvolverse en la vida real. De ese modo, cuando un alum-
no se enfrenta a una situación-problema está aplicando lo que sabe o sabe hacer a una
realidad que le es familiar o al menos que le resulta verosímil.

Cubierta la tarea de redacción, el resultado es un conjunto de ítems a partir de los cuales


se podrán seleccionar los que finalmente integren las pruebas de evaluación, teniendo en
cuenta para ello los pasos enumerados anteriormente. Tanto los redactores como los revi-
sores recibirán una formación específica para el desarrollo de su trabajo.

Formato de la prueba

Desde el punto de vista formal, las pruebas deben facilitar su uso por los destinatarios de
las mismas y resultar atractivas, motivadoras. Para las pruebas de lápiz y papel, el tamaño
de la hoja, el tipo y tamaño de la letra, la amplitud de los márgenes, los espacios para la
respuesta, los dibujos o gráficos y otros aspectos formales han de contribuir a cubrir los
dos criterios mencionados.

El lenguaje empleado se adaptará a la edad de los alumnos que van a ser evaluados, espe-
cialmente en la evaluación que se realiza en Educación Primaria. Una sintaxis compleja o
un vocabulario inusual vendrían a introducir una dificultad añadida a las pruebas, cuyos
resultados podrían verse en buena medida sesgados por el grado en que los sujetos han
desarrollado su competencia para la comprensión de textos escritos.

Pruebas 31
Evaluación General de Diagnóstico 2009
Marco de la evaluación

Las pruebas se presentan en soporte papel, con las salvedades que el propio contenido de
algunas pruebas impone. De hecho, al evaluar la comprensión oral, el soporte de las prue-
bas incluirá también registros en audio para ser reproducidos a los alumnos en el momen-
to de la aplicación.

Cuando se adopta para una prueba el formato de lápiz y papel, la utilización de hojas de
respuestas separadas del cuadernillo que contiene la prueba suele resultar cómodo a la
hora del vaciado o la corrección de las respuestas, especialmente cuando éstas correspon-
den a una prueba objetiva. Además, supone un ahorro material, al poderse reutilizar los
mismos cuadernillos de preguntas en tantas ocasiones como sea necesario.

En el caso de las pruebas de evaluación que nos ocupan, las respuestas podrán implicar la
redacción de un texto, la elaboración de un dibujo o la ejecución de algoritmos de cálculo,
tareas todas ellas que generan registros escritos de una cierta extensión y que se prestan en
menor medida a ser consignadas directamente sobre una hoja de respuestas. Responder en
el mismo cuadernillo donde aparecen las preguntas resultará más cómodo para el alumno,
que en todo momento tendrá delante la pregunta a la que está respondiendo, y evitará posi-
bles errores derivados de una mala ubicación de las respuestas en la hoja.

La aplicación informatizada de las pruebas es una posibilidad. Una aplicación de estas caracte-
rísticas requeriría, en todo caso, contar con la alternativa en lápiz y papel para cubrir situacio-
nes en las que surjan dificultades técnicas para la aplicación. No obstante, la aplicación infor-
mática se contempla con cautela, dado que la posibilidad de que finalmente los alumnos hayan
respondido a la prueba en formato de lápiz y papel o informático, dependiendo del centro en
que se ubiquen, puede tener alguna repercusión sobre la comparabilidad de las respuestas. El
formato de presentación podría incidir sobre las respuestas, al motivar en mayor o menor medi-
da a los sujetos, al plantear mayores o menores dificultades para acceder a la información, etc.

En definitiva, los materiales necesarios para la aplicación de las pruebas se reducirían a los que
ha de manejar el examinado y los que manejan los administradores /correctores de las pruebas:

• Cuadernillos para el examinado conteniendo las respectivas pruebas, con


espacios claramente indicados donde éste pueda consignar las respuestas.
• Grabaciones en audio correspondientes a textos que formen parte de la prue-
ba, para ser reproducidas mediante los dispositivos adecuados.
• Manual de instrucciones, incluyendo las normas de aplicación, plantillas de
corrección y hojas de anotación de incidencias para los administradores.
• Criterios de corrección y puntuación para todos los ítems, de tal manera que
inequívocamente se indica a los correctores cómo han de calificar las respuestas
del examinado.

32 Pruebas
Evaluación General de Diagnóstico 2009
Marco de la evaluación

LONGITUD Y TIEMPO DE APLICACIÓN DE LAS PRUEBAS

La longitud de las pruebas está limitada por la necesidad de aplicarlas en una o dos sesio-
nes, cuya duración no sobrepase la capacidad de mantener la concentración por parte de
los sujetos, tratando de evitar el cansancio del alumnado que responde. Junto a este crite-
rio, el número de ítems a incluir en cada prueba depende finalmente del tipo de compe-
tencias que se está evaluando y especialmente de la naturaleza de las tareas que se exigen
a los sujetos en cada una de las pruebas. La realización de estudios piloto permitirá valo-
rar el número de ítems que puede ser respondido dentro de los márgenes temporales pre-
vistos para las sesiones de aplicación.

La concentración en la tarea es importante de cara a que las respuestas de los alumnos


correspondan efectivamente a su nivel de desarrollo competencial. Transcurrido un cierto
tiempo, el nivel de atención y concentración desciende considerablemente, por lo que las
respuestas de los sujetos podrían ser irreflexivas y perderían valor de cara al diagnóstico.

Un máximo de 50 minutos o una hora, en Educación Primaria y Educación Secundaria


Obligatoria respectivamente, son las duraciones aconsejables para la aplicación. No obs-
tante, podrían programarse dos sesiones consecutivas con un descanso intermedio de
15 ó 20 minutos.

APLICACIÓN DE LAS PRUEBAS

La aplicación de las pruebas debe ir precedida de una información suficiente que haga
tomar conciencia a alumnado y profesorado sobre el sentido de las mismas, y reduzca
posibles actitudes negativas que obstaculizan su realización.

La aplicación, como la corrección, deberá correr a cargo de agentes externos. No obstan-


te, durante la aplicación es conveniente contar en los grupos de Educación Primaria con
la presencia del profesor o profesora que ejerce la tutoría.

El proceso de aplicación se ajusta a las fases que se describen a continuación.

a) Fase preparatoria

Preparar la aplicación de las pruebas comienza por la sensibilización de la comunidad edu-


cativa sobre el sentido y la naturaleza de la evaluación que se va a llevar a cabo.

A falta de un mayor desarrollo en cuanto a una cultura de la evaluación de nuestro sistema


educativo, la evaluación es inevitablemente asociada a las funciones de control y al rendi-
miento de cuentas. Por ello, una evaluación externa como la que nos ocupa puede suscitar
desconfianza y recelo por parte de los centros, en los que puede surgir el temor de dar una

Pruebas 33
Evaluación General de Diagnóstico 2009
Marco de la evaluación

imagen pobre de los resultados de aprendizaje logrados. De ello se derivaría la interven-


ción del profesorado durante la aplicación, tratando de favorecer un buen resultado por
parte de sus alumnos y alumnas.

Tratándose de una evaluación general que tiene por finalidad recoger datos sobre las
comunidades autónomas y sobre el conjunto del Estado, las repercusiones sobre los cen-
tros y sobre el alumnado pueden verse más lejanas. Otra actitud posible es la de desinte-
rés por las pruebas y la falta de implicación en la realización de las mismas.

Para contrarrestar estas actitudes, se ha considerado necesario adoptar medidas como las
siguientes:

• La aplicación de las pruebas va precedida de una información a la comunidad


educativa, tratando de aclarar el sentido de la evaluación y motivar a su
realización. Al margen de la difusión hecha por diferentes medios sobre los
propósitos y la naturaleza de la evaluación, la Administración educativa se dirige
a los equipos directivos de los centros y a los profesores tutores de los cursos
implicados en la evaluación a fin de proporcionarles la información necesaria.

• Los intentos de sensibilizar a la comunidad educativa, y en particular a los


agentes directamente implicados en la evaluación, van acompañados de
garantías sobre el carácter confidencial de los resultados y sobre su utilización
con fines exclusivamente vinculados a la evaluación general de diagnóstico. Se
aclarará a estas audiencias que no se van a elaborar y publicar datos sobre los
individuos evaluados ni clasificaciones recogiendo el lugar que ocupan los
centros en función de los resultados conseguidos por su alumnado.

b) Fase de ejecución

La aplicación de las pruebas se realizará de acuerdo con el calendario previsto por la


Administración educativa. Parece razonable dedicar por prueba dos sesiones de 50 minu-
tos o una hora que quedan ubicadas en la primera franja horaria de la jornada escolar.

En la medida de lo posible, se establecerá un calendario de aplicación común para todos


los centros. La aplicación se llevará a cabo en los grupos de 4º de Educación Primaria o
2º de Educación Secundaria Obligatoria, reuniendo en una misma aula a todos los alum-
nos y alumnas que responden a la prueba y, en su caso, reservando un espacio para las
actividades de aplicación individual. En la aplicación de las pruebas hay que insistir a los
alumnos en el interés que tiene la realización de esta actividad con la máxima seriedad y
concentración.

34 Pruebas
Evaluación General de Diagnóstico 2009
Marco de la evaluación

CORRECCIÓN DE LAS PRUEBAS

Finalizada la aplicación, los agentes externos que hayan participado en la aplicación u


otros jueces asumirán la tarea de corrección de las pruebas. Para ello, utilizarán los crite-
rios de corrección y puntuación facilitados, así como las pautas y recomendaciones trans-
mitidas por la administración educativa. Tanto para la aplicación como para la corrección,
se podrán organizar sesiones formativas que ayudarán a coordinar la actuación de todos
los agentes.

La puntuación de cada ítem de una prueba supone dar un valor de 0 ó 1, asignando puntua-
ciones intermedias en el caso en que las respuestas hayan sido graduadas desde las realizacio-
nes que denotan un nivel de competencia más bajo hasta las que implican un nivel más alto.

La utilización de plantillas que permiten asignar puntuaciones es fundamental en el caso


de las preguntas de elaboración que requieren una respuesta no cerrada. El manejo de
tales plantillas será uno de los contenidos abordados en las sesiones formativas dirigidas a
los correctores, de tal modo que se viera favorecida la homogeneidad en la corrección.

CUESTIONARIOS DE CONTEXTO

La recogida de información sobre variables relativos al contexto, los recursos de los centros
y los procesos que se desarrollan en los mismos se puede realizar mediante cuestionarios diri-
gidos al alumnado, al profesorado, a la dirección de los centros y, en su caso, a las familias.

Los cuestionarios de contexto incluirán una serie de ítems a través de los cuales se recoja
información suficiente para el estudio de las variables del contexto socioeconómico y cul-
tural de los alumnos y de los centros, de los procesos y los recursos educativos asociados
a la adquisición de las competencias básicas.

El contenido de los cuestionarios dará cobertura al conjunto de variables consideradas en el


apartado referido a los contextos en este marco de la evaluación. De acuerdo con las pobla-
ciones consideradas en la evaluación, los cuestionarios irán dirigidos a los alumnos y alum-
nas que responden a las pruebas de evaluación, los profesores y profesoras de ese alum-
nado (en el caso de Educación Primaria, sólo tutores), los directores o directoras de los cen-
tros y, en su caso, las familias.

En la elaboración de los cuestionarios se observarán los siguientes criterios:

• Se intentará que los cuestionarios no dupliquen la información obtenida desde


diferentes fuentes, salvo que se considere conveniente la triangulación en algún
caso. Para ello, habrá que concretar qué variables deben ser estudiadas a partir

Pruebas 35
Evaluación General de Diagnóstico 2009
Marco de la evaluación

de la información que aporta cada uno de los colectivos de informantes,


planteando las cuestiones correspondientes a quienes poseen mayor capacidad
para informar sobre el aspecto en cuestión.

• Las preguntas serán preferentemente cerradas, con el fin de reducir el esfuerzo


de quienes responden y facilitar también las tareas de corrección y análisis.

La aplicación de los cuestionarios a los diferentes actores implicados se realizará de modo


distinto según cada caso:

• Los cuestionarios dirigidos al alumnado se administrarán a continuación de las


pruebas de evaluación.

• Los cuestionarios dirigidos al profesorado y a la dirección de los centros pueden


ser respondidos utilizando procedimientos telemáticos.

• En el caso de la aplicación de un cuestionario de familias en Educación Primaria,


el propio tutor o tutora, se podrá encargar de hacer llegar el cuestionario a las
familias, fijando un plazo para su devolución.

36 Pruebas
5. EL OBJETIVO DE LA EVALUACIÓN:
LAS COMPETENCIAS BÁSICAS
INTRODUCCIÓN

La Ley 2/2006, de 3 de mayo, Ley Orgánica de Educación, prescribe la evaluación de diag-


nóstico de las competencias básicas alcanzadas por el alumnado al finalizar el segundo ciclo
de la Educación Primaria y al finalizar el segundo curso de la Educación Secundaria
Obligatoria.

Tal precepto conduce a una descripción de dichas competencias que, orientada a la evalua-
ción, refleje el desarrollo posible en esos niveles con arreglo a lo que determinan las ense-
ñanzas mínimas, acordadas para su consideración común a escala de Estado.

Tales enseñanzas mínimas enuncian ocho competencias:

1. Competencia en comunicación lingüística.

2. Competencia matemática.

3. Competencia en el conocimiento y la interacción con el mundo físico.

4. Tratamiento de la información y competencia digital.

5. Competencia social y ciudadana.

6. Competencia cultural y artística.

7. Competencia para aprender a aprender.

8. Autonomía e iniciativa personal.

El Objetivo de la Evaluación: Las Competencias Básicas 37


Evaluación General de Diagnóstico 2009
Marco de la evaluación

Ahora bien, tal descripción demanda a su vez un elemental acotamiento terminológico que
facilite su adecuada comprensión. Para ello, conviene tener presente el marco conceptual
que la Comisión Europea1 ha aportado a los estados integrantes de la Unión como “herra-
mienta de referencia” para la definición y descripción de las competencias. Con ese objeto
proporciona una definición abierta que identifica las competencias como “una combinación
de conocimientos, destrezas y actitudes que incluyen la disposición para aprender y el saber
cómo” y matiza que una competencia clave2 es crucial cuando ésta contribuye a diferentes
aspectos de la vida:

a. La realización y desarrollo personal a lo largo de la vida (capital cultural).

b. La inclusión y la ciudadanía activa (capital social).

c. La aptitud para el empleo (capital humano).

Por último, se subraya que este conjunto de conocimientos, destrezas y actitudes que se
engloba en el término de competencias clave o competencias básicas debería:

a. Ser desarrollado a lo largo de la enseñanza o formación obligatoria,

b. ser transferible, es decir, aplicable en muchas situaciones y contextos, y

c. ser multifuncional, en tanto que pueda ser utilizado para lograr diversos objetivos,
para resolver diferentes tipos de problemas y para llevar a cabo diferentes tipos de
tareas.

1 COMISIÓN EUROPEA, DIRECCIÓN GENERAL DE EDUCACIÓN Y CULTURA, Competencias clave para un aprendizaje a lo largo de
la vida. Un marco de referencia europeo. Noviembre 2004.
2 La normativa española ha optado por el adjetivo básica en su doble acepción de esencial y vinculante, sin que parezca
asociable a la acepción de elemental.

38 El Objetivo de la Evaluación: Las Competencias Básicas


Evaluación General de Diagnóstico 2009
Marco de la evaluación

LA EVALUACIÓN DE LAS COMPETENCIAS BÁSICAS DEL CURRÍCULO

Una evaluación de competencias, entendidas, como se ha señalado, en su dimensión integra-


dora de conjunto de conocimientos, destrezas y actitudes que se requieren para la realización
personal, el ejercicio de la ciudadanía activa y la incorporación satisfactoria en la vida adulta
de nuestras y nuestros jóvenes, que sustenten un aprendizaje a lo largo de toda la vida y que
resultan transferibles y multifuncionales.

Y debe afirmarse para cualquier cohorte de edad que se determine siempre que se pueda
verificar un mínimo grado de maduración tipificable, como se ha hecho al determinar
dicha evaluación en relación con la última parte de los cursos de cuarto de educación pri-
maria y segundo de educación secundaria obligatoria.

Tal evaluación se hace viable en la intersección del enfoque de competencias (más global,
más cercano a la vida real que a la abstracción académica y más asociado a la aplicación del
conocimiento que a la mera verificación de conceptos) con el enfoque de los elementos del
currículo (más ligado a campos de conocimiento, más secuenciado y más ordenado por la
historia de la educación). De este modo, resulta pertinente una evaluación de competen-
cias asociadas a los conocimientos, destrezas y actitudes que se determinan en el currículo.

Y tal evaluación se hace necesaria en cuanto ilustra con sus resultados los diferentes grados
de adquisición de las competencias básicas por el alumnado, en cada curso analizado, pro-
porcionando al profesorado, a las administraciones y, en su caso, a las familias y al propio
alumnado, elementos descriptivos esenciales para reorientar procesos educativos.

En consecuencia, el objeto de la evaluación de la adquisición de las competencias básicas


del currículo se define como la medición del grado de logro de los aspectos del currículo a
que se alude en el artículo 6.2 de la Ley Orgánica de Educación y que se desarrollan en los
Reales Decretos 1513/2006 de 7 de diciembre y 1631/2006 de 29 de diciembre, respecti-
vamente, para la Educación Primaria y la Educación Secundaria Obligatoria, a la luz del
enfoque metodológico y conceptual que se deriva del marco de competencias propuesto
por la Unión Europea.

El enmarque preciso de este tipo de evaluación es así concebido como un espacio de tra-
bajo referido a las competencias básicas desde las enseñanzas mínimas (especialmente
criterios de evaluación y, eventualmente, contenidos).

Esta tarea, en un camino de ida y vuelta permanente de una y otra referencia, exige una
utilización adecuada de ambas fuentes.

El Objetivo de la Evaluación: Las Competencias Básicas 39


Evaluación General de Diagnóstico 2009
Marco de la evaluación

Cuadro de intersecciones entre cada área o materia y cada competencia

No se puede identificar ninguna competencia con una sola materia y viceversa. Una apro-
ximación a una visión integrada conduce al siguiente cuadro de intersecciones, como refe-
rencia para la evaluación de las competencias básicas del currículo de Educación Primaria,
al señalar las contribuciones especialmente aportadas por las diferentes áreas al desarrollo
de las competencias básicas. Se resalta el grado de relación entre las mismas, establecien-
do varios niveles cualitativos de relación3.

Conocimiento Lenguas
Nº E. Artística E. Física Idioma Matemáticas E. Ciudadanía
del medio oficiales

1 •• • •• •• •

2 • • ••
Competencias

3 •• • • •

4 • • • • •

5 •• • • • • ••

6 • •• • • •

7 •• • • •• •• ••

8 • • • • • •

Del mismo modo, las contribuciones más nítidas de las distintas materias de la Educación
Secundaria Obligatoria al desarrollo de las competencias se reflejan en la siguiente matriz
de relaciones.

Ciencias Lenguas
Nº CSGH Idioma Matemáticas EPV Música Tecnol. E. Ciudadanía
de la N. E. Física oficiales
1 • • •• ••

2 • • •• • •
Competencias

3 •• •• • •• •

4 • • •• • • ••

5 • •• • • ••

6 •• • • •• ••

7 •• •• •• •• ••

8 • • • • • • •

3 La intensidad de la relación de cada área o materia con la competencia se gradúa de mayor a menor con ••, • y .

40 El Objetivo de la Evaluación: Las Competencias Básicas


Evaluación General de Diagnóstico 2009
Marco de la evaluación

Esta visualización subraya el carácter transversal de las competencias y proporciona orienta-


ciones para una adecuada aproximación a una evaluación de competencias del currículo,
en la medida en que el mapa de referencias es desigual, refuerza la idea de la competencia
como eje prioritario y resalta firmemente la inconveniencia de asociar de manera limitada
la evaluación de cualquier competencia a una o unas pocas áreas de conocimiento.

Esta primera aportación tiene el carácter de tentativa provisional, abierta a la revisión y


corrección que acometan los grupos de trabajo encargados del desarrollo de la evaluación
y de las pruebas que se propongan para cada competencia.

Por otra parte, esta representación orienta sobre la distribución de las cuestiones que
integrarán cada parte de la prueba de modo que los componentes de cada competencia
tengan el peso relativo adecuado.

FASES DE TRABAJO PARA LA APROXIMACIÓN AL DESGLOSE DE LAS COMPETENCIAS BÁSICAS


Y PARA LA ELABORACIÓN DE LOS ÍTEMS CORRESPONDIENTES Y LA PRUEBA PILOTO

La experiencia de las evaluaciones nacionales e internacionales muestra que los marcos


correspondientes son siempre el fruto de un proceso que comienza con una propuesta ini-
cial y luego se ajusta, de acuerdo con los resultados de las pruebas piloto y de las aplica-
ciones sucesivas. Éste es el proceso que se ha acordado seguir para la elaboración del
Marco de la Evaluación General de Diagnóstico, particularmente en este apartado referido
al desglose de las competencias básicas y a la elaboración de los ítems para las pruebas.

Por ello, se propuso que la aproximación a las dimensiones de las competencias básicas y la
elaboración de los ítems correspondientes y de la prueba piloto se desarrollarán en tres fases.

En una primera fase, hasta finales de julio de 2007, se prepararon las competencias para
la evaluación, es decir, se adoptaron las directrices sobre la definición y precisión de las
dimensiones de cada competencia, basadas en el currículo: se partió de la descripción que
realizan los reales decretos por los que se establecen las enseñanzas mínimas.

Asimismo, se estableció la relación entre cada competencia y las áreas o materias de las
enseñanzas mínimas (intensidad de la relación) y la relación entre las dimensiones y los
otros elementos del currículo de cada una de ellas.

En la segunda fase, desde julio a finales de 2007, los grupos de trabajo constituidos al
efecto detallaron las competencias, establecieron las precisiones que permiten relacionar
las competencias con los elementos de los currículos de las enseñanzas mínimas a los efec-
tos de elaboración de ejercicios para cada competencia.

El Objetivo de la Evaluación: Las Competencias Básicas 41


Evaluación General de Diagnóstico 2009
Marco de la evaluación

En esta fase se propusieron distintos ítems según tipo (cerrados, abiertos,...), según proce-
sos, según elementos de las dimensiones de las competencias, etc. Y se estableció el peso
relativo que deberá tener cada uno de estos elementos en las pruebas. Se elaboraron tam-
bién los ítems para la aplicación piloto.

En la tercera fase, durante 2008, se ha aplicado la prueba piloto. El ajuste y puesta a punto
de las dimensiones de las competencias y sus relaciones con los elementos del currículo se
llevarán a cabo en función de los resultados de esa prueba.

Se realizará la primera elaboración de escalas de rendimiento (aplicación de la teoría de res-


puesta al ítem) para asociar cada ítem a una puntuación de dificultad y asignar a cada partici-
pante en la prueba piloto una puntuación en la misma escala que mida su destreza estimada.

Se establecerá una primera aproximación a la propuesta de niveles de rendimiento (bandas


de puntuaciones) para describir qué caracteriza el dominio de la competencia de los estu-
diantes cuya puntuación se encuentra en cada uno de dichos niveles.

Los ítems para las pruebas se elaboran en 2009. Este año se presenta también el Marco de
la Evaluación General de Diagnóstico 2009.

DIRECTRICES SOBRE LAS DIMENSIONES DE LAS COMPETENCIAS

Para organizar las dimensiones de cada competencia se ha propuesto considerar, en primer


lugar, la descripción de las competencias básicas que realizan los Reales Decretos por los que
se establecen las enseñanzas mínimas, que se ha recogido en el anexo II. En segundo lugar, se
ha propuesto considerar, a modo de ejemplo, los criterios adoptados por la OCDE, la IEA y la
UE en sus estudios internacionales de las competencias básicas. Esto significa renunciar a una
nueva descripción de cada competencia distinta a la ya aportada por el currículo oficial. No
obstante, sí debe considerarse la combinación de elementos del currículo de las áreas o mate-
rias que contribuyan a la competencia, de acuerdo con la propia propuesta del currículo.

Para la evaluación de las competencias básicas del currículo en las evaluaciones diagnósti-
cas, se han tenido en cuenta tres elementos:

a) Las situaciones y contextos en los que se aplica la competencia.

b) Los procesos que debe poner en marcha el alumnado para aplicar el conocimien-
to en contextos y situaciones reales.

c) Los conocimientos, destrezas y actitudes de todo tipo que el alumnado ha apren-


dido y asimilado de forma significativa, tanto en contextos escolares formales
como en situaciones de la vida ordinaria.

42 El Objetivo de la Evaluación: Las Competencias Básicas


Evaluación General de Diagnóstico 2009
Marco de la evaluación

– Las situaciones y contextos

El currículo define las competencias básicas con carácter terminal de las enseñanzas bási-
cas. No obstante, dado que se van a evaluar al término del segundo ciclo de Educación
Primaria y del primer ciclo de Educación Secundaria Obligatoria, deberán analizarse estos
contextos y situaciones para que la evaluación sea realmente competencial.

– Los procesos

Son acciones de índole cognitiva, afectivo social, e incluso física que el alumnado pone en
marcha para solucionar una situación problemática, utilizando el conocimiento pertinen-
te en un contexto determinado.

– Los conocimientos, destrezas y actitudes

Incluyen todos aquellos conceptos, destrezas y actitudes establecidos en el currículo de la


etapa correspondiente debidamente asimilados de forma significativa y que son necesarios
para desempeñarse con pericia en las situaciones y contextos concretos en los que se
desenvuelve el alumnado en su vida ordinaria.

Criterios para determinar estos elementos en cada una de las competencias básicas del
currículo

a) Criterios para concretar el contexto y las situaciones

Los grupos de trabajo han reflexionado y especificado los diferentes contextos y situacio-
nes en los que se desenvuelve la vida de los alumnos y alumnas en el segundo ciclo de
Primaria y en el primer ciclo de Educación Secundaria Obligatoria. Es fundamental concre-
tar este aspecto porque va a aportar significado y sentido a la evaluación diagnóstica de
las competencias básicas del currículo.

Para ello, se han tenido en cuenta los siguientes criterios:

• Los diferentes entornos en los que se desenvuelven los alumnos: familia,


amistades, entorno escolar formal e informal, ocio, etc.

• Las capacidades del alumnado y sus conocimientos previos.

• Los intereses y motivaciones del alumnado.

• Las herramientas de que dispone el alumnado para acometer las tareas


de evaluación (conocimiento de códigos, uso del ordenador, hábitos de
trabajo, etc.).

El Objetivo de la Evaluación: Las Competencias Básicas 43


Evaluación General de Diagnóstico 2009
Marco de la evaluación

b) Criterios para establecer los procesos en cada una de las competencias básicas

En cada una de las competencias básicas se deben establecer unos procesos ajustados a la
entidad de cada una de ellas. Para ello se tendrán en cuenta los procesos incluidos en las
evaluaciones internacionales realizadas con enfoques competenciales, sobre todo PISA y
PIRLS, así como el marco común europeo de referencia para las lenguas y el marco teórico
del CIVIC AND CITIZEN SHIP FRAMEWORK, debidamente adaptados a los currículos de las
etapas correspondientes y a las buenas prácticas del profesorado. Se pretende conjugar la
autonomía del enfoque de cada una de las competencias con una coherencia entre las dife-
rentes competencias.

La descripción de las diferentes competencias básicas del currículo parte de unos supuestos
comunes a todas ellas que deben ser tenidos en cuenta para concretar los procesos en cada
una de las competencias, y son los siguientes:

• Los procesos enfatizarán la aplicación a los contextos definidos de los


conocimientos, destrezas y actitudes asimilados significativamente.

• Los procesos serán pertinentes a la hora de poner el acento en la evaluación en


los aprendizajes imprescindibles.

• Los procesos estarán definidos de tal forma que integrarán significativamente y


en contextos reales los aprendizajes realizados.

• Los procesos aportarán criterios para orientar la enseñanza y la evaluación en


clave de competencias.

• Los procesos tendrán en cuenta el enfoque interdisciplinar de las competencias


básicas.

• Los procesos se aplicarán no sólo a situaciones de las áreas, sino a diferentes


situaciones de la vida escolar, tales como el clima de aula y de centro, las
actividades complementarias y extraescolares, etc.

c) Criterios para seleccionar contenidos y actitudes

No se trata de evaluar todos y cada uno de los contenidos de las áreas del currículo desde
una perspectiva curricular, más bien se trata de seleccionar aquellos conocimientos, destre-
zas y actitudes, desde la perspectiva de su aplicación a la vida y al contexto ordinario del
alumnado. En este sentido, los criterios de evaluación de las diferentes áreas del currículo
son un referente primordial para seleccionar los contenidos más pertinentes. A la hora de
priorizar o seleccionar, en su caso, conocimientos, destrezas y actitudes, se tendrán en
cuenta los siguientes criterios:

44 El Objetivo de la Evaluación: Las Competencias Básicas


Evaluación General de Diagnóstico 2009
Marco de la evaluación

• Su importancia para desenvolverse en las situaciones cotidianas de la vida de los


alumnos (a nivel personal, familiar, comunitaria, social, escolar, etc.).

• La vigencia social de los saberes considerados; no sólo en el momento presente,


sino atendiendo también a un futuro próximo (el desarrollo de las competencias
básicas debe considerarse en el contexto de un proyecto de etapa).

• La implicación en algunos de los procesos científicos y humanísticos más


relevantes para la vida cotidiana de (las ciencias de la naturaleza, ciencias
sociales y humanas, etc.).

El desglose de las competencias básicas

Como ejemplos de referencia para el desglose de las competencias básicas por los grupos
de trabajo, se propusieron los marcos PISA 2006 de ciencias y matemáticas y el proyecto de
marco PISA 2009 de comprensión lectora, tanto sobre textos impresos como sobre textos
electrónicos, el marco PIRLS 2006 y el borrador de marco de la evaluación ICCS (estudio
internacional sobre educación cívica y ciudadana) de la IEA. Estos ejemplos incluyen distin-
tas dimensiones para cada una de las competencias. En resumen, se propusieron estos mar-
cos de evaluación como punto de partida para los trabajos de elaboración, a partir del currí-
culo español, de la propuesta de organización de cada competencia y de los ítems corres-
pondientes para la prueba piloto.

En consecuencia, se propuso a los grupos de trabajo que elaboraran el desglose de las com-
petencias, de acuerdo con el siguiente índice:

1. Presentación de la competencia básica (de acuerdo con los RR. DD. de


Enseñanzas Mínimas).

2. Dimensiones de la competencia, que deben incluir:

a. Contextos y situaciones.

b. Procesos.

c. Bloques de contenido.

d. Actitudes, en su caso.

3. Cuadros de relaciones.

a. Cuadro en el que se relacionan, a través de los criterios de evalua-


ción de las distintas áreas del currículo que contribuyan a la compe-
tencia, los procesos y los bloques de contenidos.

b. Tabla resumen de las actitudes, en su caso.

El Objetivo de la Evaluación: Las Competencias Básicas 45


Evaluación General de Diagnóstico 2009
Marco de la evaluación

4. Matriz de especificaciones de la competencia con una propuesta del peso pon-


deral de los procesos y de los conocimientos y, en su caso, de las actitudes.

5. Matriz de especificaciones con las unidades de evaluación e ítems propuestos.

6. Colección de unidades de evaluación (estímulos e ítems correspondientes).

Los documentos elaborados por los grupos para cuatro de las competencias básicas se reco-
gen en los anexos: comunicación lingüística (anexo III), matemática (anexo IV), conoci-
miento e interacción con el mundo físico (anexo V) y social y ciudadana (anexo VI).

Para elaborar los ítems se formularon a los grupos de trabajo las siguientes precisiones:

• En el conjunto de las pruebas, los porcentajes del tipo de preguntas deben


ajustarse aproximadamente a la siguiente distribución:

– 50% (como mínimo) de preguntas de opción múltiple (4 alternativas).

– 20% (como máximo) de preguntas totalmente abiertas.

– 30% de preguntas de respuesta construida semiabiertas.

• Se elaborarán cinco cuadernillos de cada una de las competencias básicas


evaluadas.

• Cada cuadernillo consta de dos bloques con tres unidades de evaluación cada
uno de ellos.

• Cada unidad de evaluación consta de un estímulo y un promedio de cinco


preguntas basadas en el estímulo propuesto.

46 El Objetivo de la Evaluación: Las Competencias Básicas


6. ANÁLISIS DE RESULTADOS

OBTENCIÓN DE PUNTUACIONES

En un primer momento, las pruebas de evaluación permitirán obtener una puntuación


directa a partir de las puntuaciones asignadas a cada ítem. En el caso de ítems dicotómicos
(acierto-error), la puntuación será 1 ó 0, mientras que en el caso de los ítems politómicos
(respuestas graduadas) la puntuación puede admitir algún valor intermedio.

La puntuación directa puede ser objeto de análisis en sí misma o puede ser considerada
como un simple instrumento para aplicar la teoría de respuesta al ítem (TRI). En el primer
caso, el análisis de la puntuación directa ofrece la posibilidad de una evaluación criterial, de
un diagnóstico acerca de qué porcentaje del alumnado español (o de una comunidad autó-
noma) está adquiriendo cada una de las competencias básicas y en qué medida. La validez
de este diagnóstico quedaría garantizada por dos condiciones previas:

• la fundamentación rigurosa de la matriz de especificaciones, que supondrá adop-


tar decisiones sobre lo que se considera que el alumnado debería saber hacer en
un nivel educativo determinado, y

• la elaboración concienzuda de las pruebas, según un diseño matricial que dé la


posibilidad de elaborar distintas modelos de prueba para una misma evaluación,
con un número suficientemente amplio de ítems que evalúen los distintos aspec-
tos de las competencias básicas de manera completa y equilibrada.

No obstante, esta evaluación criterial, aun siendo deseable, estaría sujeta a críticas como las
que han sido ampliamente desarrolladas en toda la literatura existente en torno a la teoría
clásica de los tests. Al adoptarse este enfoque, la medida de la competencia del alumnado
pasa a depender, en último extremo, de la opinión de unos jueces sobre los niveles de difi-
cultad exigibles en las pruebas, opinión que podría ser contradicha por otros expertos o por
la opinión pública en general.

Análisis de Resultados 47
Evaluación General de Diagnóstico 2009
Marco de la evaluación

Por tanto, parece aconsejable la evaluación normativa basada en una puntuación transforma-
da para la que se construirá una escala común, con una media global de 500 puntos y una
desviación típica de 100 puntos, que es la utilizada en proyectos internacionales como PISA.
Habrá que insistir, de cara al gran público, en que no puede interpretarse como nivel satisfac-
torio el alcanzar el punto medio de la escala, es decir 500, o como insatisfactorio el no alcan-
zarlo. Para valorar los resultados es necesario conocer lo que saben hacer los alumnos y esa
información la ofrecen los distintos niveles fijados por una serie de puntuaciones de corte.

Los puntos de corte de la escala representarán entonces competencias o habilidades alcan-


zadas por los alumnos situados en esa puntuación o por encima de ella. Para determinar la
escala, basta con asignar significado a dos puntos. La evaluación general podría establecer
un número no elevado de niveles de competencia, puesto que su objetivo no es describir
pormenorizadamente el rendimiento individual, sino establecer un diagnóstico general
sobre qué porcentajes de la población evaluada son capaces de poner en práctica una serie
de habilidades y hasta qué punto.

Para garantizar que se miden adecuadamente las competencias básicas del alumnado, con
independencia del nivel de dificultad de las pruebas, la puntuación transformada se obtendrá
a partir de alguno de los modelos de la TRI. Aunque los supuestos y la metodología de estos
modelos pueden ser difíciles de explicar a la opinión pública, hay que insistir además en que
se trata de la solución más rigurosa para medir el resultado de pruebas con diseño matricial
y longitudinal. La propia sociedad demandará la comparación de los resultados de pruebas
distintas aplicadas para observar la evolución del sistema educativo a lo largo del tiempo.

En cuanto al modelo TRI finalmente escogido, el modelo logístico de un parámetro para


ítems dicotómicos, formulado por Rasch (1960), puede ser suficiente para calibrar los ítems
y estimar las puntuaciones, teniendo la ventaja de su mayor simplicidad, al considerar úni-
camente el parámetro de dificultad. No obstante, el modelo de dos parámetros puede ofre-
cer más posibilidades en el pilotaje para seleccionar los ítems que presenten mejor compor-
tamiento psicométrico. En cualquier caso, las decisiones acerca del modelo adoptado
corresponderán a quienes lleven a cabo el análisis, teniendo en cuenta para ello la bondad
de ajuste lograda a cada modelo.

En el caso de los ítems politómicos, la curva característica del ítem habrá de determinarse
a partir de modelos adecuados para respuestas multicategoriales.

INFORMACIÓN PROPORCIONADA

La evaluación deberá ofrecer datos sobre los niveles de adquisición o dominio de cada com-
petencia por parte del alumnado español y los promedios de cada comunidad autónoma y

48 Análisis de Resultados
Evaluación General de Diagnóstico 2009
Marco de la evaluación

del conjunto del Estado. Se concretarán dichos niveles en las diferentes dimensiones o sub-
dimensiones de cada una de las competencias evaluadas. Los resultados básicos se presen-
tarán en dos formas: empleando medidas de tendencia central y dispersión de las puntua-
ciones obtenidas por los alumnos, y presentando los porcentajes de alumnos que se sitúen
en cada nivel de competencia. Siempre se dará información sobre la precisión de las medi-
das, en la forma de errores estándar, intervalos de confianza o similares. La información
fundamental la aportarán los índices de dificultad y discriminación y los datos se manejarán
en puntuación TRI.

Cabe pensar en la posibilidad de descender hasta los descriptores o indicadores que hayan
servido de base para la elaboración de los ítems (o para la fijación de los criterios de correc-
ción en el caso de las actividades y preguntas abiertas). Sin embargo, a medida que des-
cendemos en niveles de concreción, el número de ítems disponible es inferior, con la con-
siguiente pérdida de confianza en las puntuaciones logradas. Por otra parte, los indicado-
res o descriptores vendrían a representar una muestra del universo de descriptores o indi-
cadores que podrían considerarse dentro de una competencia. En definitiva, informar sobre
esos elementos no tiene el mismo interés que informar sobre las competencias y sus prin-
cipales dimensiones, y trae consigo el incremento del volumen de información a presentar
tras el análisis de los resultados.

Para el análisis de los datos de contexto, teniendo en cuenta lo presentado en el apartado


dedicado a este tema en el presente marco de la evaluación, se sugiere el manejo de índi-
ces similares a los empleados en PISA, que ya han sido ampliamente difundidos y resultan
bastante comprensibles para la opinión pública. En el caso de constructos como el índice
socioeconómico, también se puede optar por el método PISA, basado en el análisis de com-
ponentes principales que ofrece las correspondientes puntuaciones factoriales. El análisis de
componentes se realizará en las diferentes comunidades autónomas por separado para ase-
gurar la validez nacional del índice. Por último, se dará un tratamiento diferenciado a los
constructos multifuente, obtenidos mediante ponderación de los resultados de los distintos
cuestionarios (por ejemplo, alumnado, padres, profesorado).

DESAGREGACIÓN DE LOS DATOS

Los resultados se desagregarán en función de los estratos muestrales considerados, que son
los correspondientes a las comunidades autónomas.

Uno de los objetivos de la evaluación general de diagnóstico es permitir una comparación


de los rendimientos obtenidos y ofrecer una referencia para las evaluaciones de las
comunidades autónomas. El análisis de datos debe contribuir a explicar las diferencias.

Análisis de Resultados 49
Evaluación General de Diagnóstico 2009
Marco de la evaluación

Ese fin se conseguiría, al menos, por dos procedimientos. Por un lado, habrá que indicar con
claridad las diferencias estadísticamente significativas, con un intervalo de confianza y un
valor p razonables. Una buena solución será elaborar cuadros con los estadísticos obtenidos
y los correspondientes intervalos de confianza, con y sin ajuste Bonferroni, como hace PISA.
Se tratarían así, al menos, las variables de rendimiento global y por niveles, por un lado, y las
variables de equidad absoluta (dispersión de puntuaciones dentro de cada región), por otro.
De esta manera, se interpretaría adecuadamente qué significa estar cuatro o cinco puestos
por encima o por debajo de la media nacional o de alguna comunidad autónoma de refe-
rencia, que es lo primero que van a ver muchos de los destinatarios de la evaluación.

En segundo lugar, habrá que hacer un análisis lo más completo posible de las variables de
contexto, de entrada y de procesos. Los cuestionarios y la codificación de centros y alum-
nos (por ejemplo, con códigos específicos por tamaño de población o por sexo, respecti-
vamente) permitirán también identificar resultados por alguna característica del alumna-
do o del centro, siempre que el agrupamiento fuera estadísticamente significativo en el
conjunto de la muestra. Sin entrar en los diferentes aspectos que podrían estudiarse, de
los que se trata extensamente en el apartado correspondiente del dictamen, sí habría que
subrayar que la interpretación de las diferencias regionales no debería limitarse a consta-
tar la relación entre el rendimiento y las condiciones socioeconómicas en las distintas
comunidades autónomas. En la medida de lo posible, también habría que identificar dife-
rencias en el sistema educativo o en los procesos de centro y de aula, aunque las decisio-
nes finales al respecto desbordan el ámbito puramente técnico de estas propuestas.

Así que, ciñéndose a los criterios técnicos, la recomendación para el tratamiento de las
diferencias regionales podría apuntar en tres direcciones:
1) Medidas de tendencia central y de dispersión agrupadas en intervalos de rango;
2) regresión simple de puntuaciones de rendimiento sobre una selección de índices
significativos;
3) ANCOVA para determinar diferencias de rendimiento independientes de las
variables de contexto que se consideren.

En cuanto al resto de las desagregaciones referidas a índices no construidos, podrían propo-


nerse las siguientes. En el nivel del alumnado, los resultados se darán al menos por sexo, por
edad (considerando si el alumno está o no en el nivel que le corresponde), por lugar de naci-
miento (teniendo en cuenta la población inmigrante) y por lengua. En estos dos últimos
casos convendría distinguir al alumnado de incorporación reciente a nuestro sistema educa-
tivo. La decisión que se tome sobre evaluación de los alumnos con necesidades educativas
especiales (ACNEE) también condicionará la desagregación de datos en este grupo. En el
caso de la compensación educativa y las adaptaciones curriculares, la diversidad de la nor-
mativa regional desaconseja tratarlos como un mismo colectivo en toda España.

50 Análisis de Resultados
Evaluación General de Diagnóstico 2009
Marco de la evaluación

Hay que insistir en la necesidad de que estos datos individuales sean estadísticamente sig-
nificativos. La experiencia en evaluación dice que, en algunos casos, la fiabilidad se garan-
tiza mejor cuando la variable independiente con la que se quiere relacionar el rendimien-
to se obtiene a partir de la codificación realizada por la propia Administración educativa
en comparación con la información proporcionada por los cuestionarios. La razón es
que la no respuesta tiene sesgo. Por ejemplo, si sólo identificamos a los repetidores o a
los inmigrantes cuyas familias hayan respondido los cuestionarios, nos faltará un grupo
importante, seguramente formado por los alumnos más problemáticos.

En el nivel del centro, puede proponerse la desagregación por titularidad y darse un trata-
miento diferenciado a los centros rurales agrupados o de características similares. El resto
de las características es preferible tratarlas dentro del análisis de contexto e incluso las dos
mencionadas habrá que explicarlas adecuadamente, siguiendo los mismos argumentos
que con las diferencias regionales. También se podrá estudiar la relación del rendimiento
con las características del grupo-aula, sobre todo si se seleccionaran grupos en la segunda
etapa del muestreo. En ese caso, cabría diferenciar distintos tamaños con algún criterio
objetivo (por ejemplo, por cuartiles), aunque sería preferible tratar el tamaño del grupo
como una variable continua y sacar conclusiones a partir de los análisis correspondientes.

La desagregación de datos relativos a índices construidos (nivel sociocultural de centro y


familias, clima del aula, estilos de enseñanza-aprendizaje, etc.) se trata en el apartado refe-
rido a la evaluación del contexto.

ANÁLISIS DE EXCELENCIA Y EQUIDAD

Los datos más llamativos de las evaluaciones de estas características son los relativos a la
calidad o excelencia en el rendimiento del alumnado. El indicador de referencia será la
media global de cada grupo estudiado, pero también atraerá un gran interés el porcenta-
je de alumnos situados en los distintos niveles de rendimiento. Un sistema educativo de
calidad debe mostrar una puntuación media elevada con pocos alumnos en los niveles de
rendimiento inferiores. En el caso de un gran número de alumnos situados en los niveles
superiores, se suele preferir la expresión “excelencia”.

De todas formas, una educación de calidad supone también un alto grado de equidad en
el sistema. El problema está en definir qué entendemos por equidad. En muchos de los
informes manejados en España, se tiende a identificar equidad con desviación típica.
Incluso en el caso de efectuar el cálculo prescindiendo de los percentiles extremos, la ima-
gen que se ofrece es bastante discutible. No está nada claro que, en el caso de medidas
de tendencia central bajas, sean preferibles unos resultados homogéneos, como es el caso

Análisis de Resultados 51
Evaluación General de Diagnóstico 2009
Marco de la evaluación

de España en PISA. Sí es evidente, en cambio, que una media global alta tiene un plus de
calidad si a ella contribuye la gran mayoría del alumnado sin grandes diferencias (como el
modelo finlandés). La comparación de la varianza intercentros con la varianza intracentros
tampoco aporta mucha más información sobre la equidad del sistema; como demuestran
casi todos los estudios, es un indicador, bien de segregación geográfica, bien de la norma-
tiva sobre elección de centro, bien de la organización del currículo.

Lo que resulta relevante es averiguar qué efecto compensador positivo tiene el sistema edu-
cativo sobre las condiciones de entrada del alumnado. Uno de los análisis clave es la regre-
sión múltiple con los indicadores de contexto individuales y de centro como variables
independientes. El estudio de la pendiente y el intercepto de la regresión aporta, por sí
solo, una información significativa acerca de la eficacia escolar entendida como efecto
compensador. La comparación entre puntuación esperada y puntuación obtenida es un
dato de fácil comprensión para el público y de gran impacto. Para corregir el efecto de
una baja dispersión en bajas puntuaciones de rendimiento se podría recurrir al coeficiente
de variación. El inconveniente de este tipo de análisis es, una vez más, que eluden cual-
quier tipo de enfoque criterial. Sirven para establecer comparaciones -la más llamativa
sería entre regiones-, pero no permiten hacer un diagnóstico global sobre la equidad del
sistema educativo en el conjunto de España, aunque sí detectarían posibles diferencias
entre las diferentes competencias y niveles evaluados.

En un segundo paso, el análisis debería ser capaz de apuntar qué tipo de decisiones contri-
buyen a dicho efecto compensador. En el nivel de la organización educativa general, sería
interesante indagar sobre las consecuencias de determinadas decisiones curriculares de las
comunidades autónomas, aunque el asunto es técnicamente complejo y entra de lleno en el
nivel de decisión político. En la práctica de los centros, habría que intentar identificar buenas
prácticas en procesos de gestión y de enseñanza-aprendizaje. Para ello sería necesario contar
con indicadores bien construidos, a partir de análisis factoriales de los cuestionarios de direc-
tores, tutores y alumnos. Los modelos jerárquicos lineales permitirían localizar en qué nivel
actúan las buenas prácticas y medir el margen de error por nivel, entre otras informaciones.

Un aspecto sobre el que convendrá pronunciarse es la evaluación del valor añadido de los
centros. Atendiendo a las conclusiones de la reunión recientemente celebrada en Oslo
sobre el tema, el enfoque que habría que adoptar en este apartado es el estudio longitu-
dinal. Habría que intentar comparar los resultados de centros con alumnado situado en
el mismo nivel de rendimiento en el momento de su incorporación, para observar cómo
han evolucionado a lo largo del tiempo. En la práctica, eso exigirá aplicar dos ciclos de
evaluación a una misma muestra de alumnado.

52 Análisis de Resultados
7. INFORMES Y DIFUSIÓN DE LOS RESULTADOS
Si entre las finalidades de las evaluaciones generales de diagnóstico, como parte de la
evaluación del sistema educativo, se encuentran Contribuir a mejorar la calidad y la equidad
de la educación y Orientar las políticas educativas (Art. 140), está claro que la difusión que
se haga de los resultados de evaluación y el tipo de informes que se elaboren, como
resultado de la misma, facilitarán o dificultarán el logro de ambas finalidades.

AUDIENCIAS

Para que pueda mejorar la calidad y equidad de la educación que ofrecen los centros edu-
cativos es necesario asegurarse de que los resultados de la evaluación se difundan adecua-
damente entre el profesorado. Por otra parte, si se quiere promover la participación de las
familias y con ello contribuir a la mejora de la educación, también habrá que considerar la
necesidad de difundir los resultados de la evaluación entre las familias.

Desde una perspectiva de servicio público, deben considerarse también los ciudadanos
y ciudadanas como destinatarios potenciales de información acerca de los resultados
evaluación.

Dado que uno de los objetivos es orientar las políticas educativas, entre los agentes desti-
natarios potenciales de información acerca de los resultados de la evaluación están los
administradores del Ministerio de Educación y los de las consejerías de educación de las
comunidades autónomas.

Por último, siguiendo el ejemplo del proyecto PISA, deben elaborarse informes pensando
en audiencias especializadas (investigadoras e investigadores, expertos en evaluación edu-
cativa,…), y establecer procedimientos para la difusión de las bases de datos y sus corres-
pondientes manuales, con el fin de facilitar explotaciones secundarias que contribuyan a
desarrollar el conocimiento sobre la educación con las aportaciones de investigadores e
investigadoras.

Informes y Difusión de los Resultados 53


Evaluación General de Diagnóstico 2009
Marco de la evaluación

¿Qué tipos de informe se elaborarán?

Se elaborarán los tipos de informe que aparecen en la tabla adjunta. Al decidir la estructu-
ra del informe y a la hora de redactar cada uno, habrá de tenerse muy presente la audien-
cia correspondiente.

Los informes para los ciudadanos y ciudadanas en general deben procurar ofrecer infor-
mación básica, relevante y expresada en un lenguaje comprensible. Deben incluir un glo-
sario de términos técnicos definidos para audiencias no expertas.

Informe / Cobertura Audiencia Formato / Contenido

Marco teórico de las EGD e Profesionales de la educación, Marco teórico y descripción


informe técnico de las EGD investigación y evaluación detallada de los aspectos
educativa. metodológicos de las EGD.

Los administradores de la Resumen de resultados principales


Resumen ejecutivo educación del Estado y de las con énfasis en aquellos que pueden
comunidades autónomas. ser útiles en la toma de decisiones
de política educativa.

Informe de España. Resultados Los ciudadanos y ciudadanas en Datos globales del Estado y de las
de 4º EP y 2º ESO general. comunidades autónomas.

Tipos de informes

Conviene evitar informes tipo avance de resultados que reportan pocas ventajas y pueden
contribuir a generar más confusión que información en la opinión pública y en el resto de
audiencias. Los informes deben presentarse cuanto antes pero asegurando siempre el
tiempo necesario para realizar los análisis que procedan y la calidad de la información que
contienen.

Los informes para administradores de la educación deben ser breves, pero la brevedad no
debe conseguirse a costa de la simplificación. Se trata de informes dirigidos a personas que
pueden y deben tomar decisiones de política educativa.

Los informes para la comunidad educativa deben ofrecer información útil para la reflexión
y la toma de decisiones orientadas a la mejora de la educación.

Los informes de resultados de 4º curso de Educación Primaria y 2º curso de Educación


Secundaria Obligatoria podrán publicarse conjunta o separadamente.

54 Informes y Difusión de los Resultados


Evaluación General de Diagnóstico 2009
Marco de la evaluación

En definitiva, los informes de nivel estatal se pueden concretar en dos:

a. Un Informe completo sobre los resultados de las evaluaciones generales de


diagnóstico, bajo un nombre a la vez adecuado y sugestivo (por ejemplo:
“Competencias alcanzadas por el alumnado español”, o bien “En qué es compe-
tente nuestro alumnado de Educación Primaria y Secundaria”). Este informe
deberá proporcionar una información más detallada sobre los resultados, agru-
pados en las diferentes categorías y explicados de manera distinta, y muy espe-
cialmente proporcionar un estudio suficiente de las variables que de hecho influ-
yen o pudieran influir en ellos. Podrá darse también una sucinta noticia acerca
de los procesos mismos de evaluación, pero de modo asequible al común de los
ciudadanos. No importaría demorar la aparición de este Informe completo hasta
que haya podido estudiarse ampliamente, tanto por los responsables del
Instituto de Evaluación como por los responsables de los organismos autonómi-
cos correspondientes, siempre que, en cualquier caso, no se demore en exceso
su publicación.

A partir de éste se elaborará un “resumen ejecutivo”, dirigido a los administra-


dores de la educación del Estado y de las comunidades autónomas, contenien-
do un resumen de resultados principales, con énfasis en aquellos que pueden ser
útiles en la toma de decisiones de política educativa.

b. Marco teórico de las EGD e Informe Técnico sobre las evaluaciones generales
de diagnóstico, consistente en una descripción del marco teórico acompañada
de descripción detallada de los aspectos metodológicos de las EGD. Este infor-
me, en cualquier caso, deberá incluir ejemplos concretos de ítems utilizados.

EN RESUMEN

Habrá dos tipos de informes: uno orientado a expertos, que contendrá toda la información
técnico-científica pertinente y otro de difusión, dirigido al público en general.

Las comunidades autónomas podrán elaborar, con sus datos, en el ámbito de sus compe-
tencias, los tipos de informes que consideren oportunos.

Informes y Difusión de los Resultados 55


ANEXOS
Evaluación General de Diagnóstico 2009
Marco de la evaluación

ANEXO I
INDICADORES Y VARIABLES DE CONTEXTO, RECURSOS Y PROCESOS, SEGÚN FUENTES
Fuentes
Sub- Sistema de
Ámbito Indicador Variables Alumnado Tutores Director
Indicador Indicadores
Género. Género. •
Titularidad. Titular. •
Tamaño del

centro. Número de alumnos.
Número de unidades. •
Número de profesores. •
Estatus socio- Nivel de
económico y estudios •
cultural. de los padres. Nivel de estudios del padre.
Nivel de estudios de la madre. •
Situación
laboral de los •
padres. Situación laboral del padre.
Situación laboral de la madre. •
Contexto Recursos impresos Ordenador.
y audiovisuales •
en el hogar.
Internet. •
Material de consulta. •
Literatura. •
Revistas científicas o de
divulgación. •

Inmigración. Origen. País de origen. •


Lengua. Lengua materna. •
Tiempo en el: sistema Tiempo transcurrido
educativo español desde la llegada. •

Necesidades
educativas •
especiales. Nº de alumnos diagnosticados.
Plantillas. Alumnos por
grupo •
Experiencia
docente. Años en la docencia. •

Formación del Valoración del grado de


profesorado. suficiencia de la formación •
general recibida.
Valoración del grado de
Recursos
suficiencia de la formación •
en TIC recibida.
Inversión Inversión en
pública en relación con •
educación. el PIB.
Gasto por
estudiante en
relación con la •
renta per cápita.
Interés e Los padres le facilitan los
implicación de recursos necesarios para
las familias en sus estudios.
la educación
de los hijos.
Los padres dejan que organice
sus estudios y tareas.
Tutoría. Nº de reuniones colectivas
con las familias. •
Procesos
Clima escolar. Satisfacción del Satisfacción con sus
alumnado. compañeros de clase. •

Satisfacción con la relación con


sus profesores. •

Satisfacción con la educación


que reciben en su centro. •

Anexo I: Indicadores y Variables de Contexto, Recursos y Procesos, según fuentes 59


Evaluación General de Diagnóstico 2009
Marco de la evaluación

Fuentes
Sub- Sistema de
Ámbito Indicador Variables Alumnado Tutores Director
Indicador Indicadores
Clima de aula. Moral y compromiso del
alumnado •

Ambiente ordenado en el aula. •


Actividades
didácticas.
Los alumnos exponemos

temas o trabajos.
Mientras explica nos pregunta. •
Hacemos debates a clase •
Trabajamos en pequeños grupos
clases expositivas y trabajo •
individual.
El profesor explica la mayor

parte de la clase.
Mientras el profesor explica

nosotros preguntamos
Hacemos los ejercicios propuestos •
Trabajamos individualmente. •
Materiales
didácticos.
Uso de libros de aula y biblioteca •
Uso de ordenadores. •
Uso de vídeos, diapositivas

y casetes.
Uso de diarios y revistas. •
Uso del libro de texto. •
Procedimientos
de evaluación.
Procesos
Revisa actividades que hacemos

a clase.
Pregunta la lección. •
Hace exámenes orales •
Tiene en cuenta la autoevaluación. •
Pone controles escritos. •
Corrige deberes y cuadernos. •
Controles tras cada tema. •
Liderazgo Valoración Promover el máximo
pedagógico. de las funciones aprovechamiento de las
directivas capacidades, cualidades, • •
relacionadas con experiencia y conocimientos
el profesorado. del profesorado.
Informarse respecto a la eficacia
docente y orientadora de los
profesores y profesoras • •
directamente o por vía de los
departamentos.
Promover trabajos en equipo
• •
con el profesorado.
Generar procedimientos para la
formación de los profesores y • •
profesoras en el centro educativo.
Tiempo de Número de horas de clase

aprendizaje. a la semana.
Tiempo dedicado a hacer
• •
los deberes.
Desfase. Año de nacimiento. •
Resultados
Repetición. •

60 Anexo I: Indicadores y Variables de Contexto, Recursos y Procesos, según fuentes


ANEXO II

DESCRIPCIÓN DE LAS COMPETENCIAS BÁSICAS

Los Reales Decretos sobre enseñanzas mínimas señalan, en su Anexo I, el sentido de la


incorporación de las competencias básicas al currículo, en cuanto permite poner el acento
en aquellos aprendizajes que se consideran imprescindibles, desde un planteamiento integra-
dor y orientado a los saberes adquiridos, e indica las finalidades de su inclusión: En primer lugar
integrar los diferentes aprendizajes, tanto los formales, incorporados a las diferentes áreas o
materias, como los informales y no formales. En segundo lugar, permitir a todos los estudian-
tes integrar sus aprendizajes, ponerlos en relación con distintos tipos de contenidos y utilizar-
los de manera efectiva cuando les resulten necesarios en diferentes situaciones y contextos. Y,
por último, orientar la enseñanza, al permitir identificar los contenidos y los criterios de evalua-
ción que tienen carácter imprescindible y, en general, inspirar las distintas decisiones relativas
al proceso de enseñanza y de aprendizaje.

Paralelamente, se advierte que el currículo se estructura en torno a áreas de conocimiento, es


en ellas en las que han de buscarse los referentes que permitirán el desarrollo de las competen-
cias en esta etapa. Así pues, en cada área se incluyen referencias explícitas acerca de su contri-
bución a aquellas competencias básicas a las se orienta en mayor medida. Por otro lado, tanto
los objetivos como la propia selección de los contenidos buscan asegurar el desarrollo de todas
ellas. Los criterios de evaluación sirven de referencia para valorar el progreso en su adquisición.

Para subrayar claramente el perfil de competencias de cara a su evaluación se presenta a


continuación la versión desarrollada del Anexo I de los Reales Decretos de Enseñanzas
Mínimas a los que se ha aludido, en un formato en que se recogen de manera analítica sus
cinco ingredientes fundamentales: descripción de cada competencia y finalidad de la
misma, conocimientos, destrezas y actitudes que la integran.

Anexo II: Descripción de las Competencias Básicas 61


Evaluación General de Diagnóstico 2009
Marco de la evaluación

Definición y finalidad
Conocimientos Destrezas Actitudes
de la competencia
• Esta competencia se refiere • Escuchar, exponer y • Leer y escribir son • Implica la capacidad
a la utilización del lenguaje dialogar implica ser acciones que suponen y empática de ponerse en el
como instrumento de consciente de los principales refuerzan las habilidades lugar de otras personas; de
comunicación oral y escrita, tipos de interacción verbal, que permiten buscar, leer, escuchar, analizar y
de representación, ser progresivamente recopilar y procesar tener en cuenta opiniones
interpretación y comprensión competente en la expresión información, y ser distintas a la propia con
de la realidad, de y comprensión de los competente a la hora de sensibilidad y espíritu crítico;
mensajes orales que se
construcción y comunicación comprender, componer y de expresar adecuadamente
intercambian en situaciones
del conocimiento y utilizar distintos tipos de –en fondo y forma– las
comunicativas diversas y
de organización y adaptar la comunicación al textos con intenciones propias ideas y emociones, y
autorregulación del contexto. Supone también la comunicativas o creativas de aceptar y realizar críticas
pensamiento, las emociones utilización activa y efectiva diversas. La lectura facilita la con espíritu constructivo.
y la conducta. de códigos y habilidades interpretación y
lingüísticas y no lingüísticas comprensión del código • Saberes actitudinales y
Finalidad y de las reglas propias del que permite hacer uso de la axiológicos: por una parte,
intercambio comunicativo lengua escrita y es, además, y en relación con los otros,
• Los conocimientos,
en diferentes situaciones, fuente de placer, de saber escuchar, saber
destrezas y actitudes propios
para producir textos orales descubrimiento de otros contrastar opiniones, tener
de esta competencia
adecuados a cada situación entornos, idiomas y en cuenta las ideas y
permiten expresar de comunicación. culturas, de fantasía y de opiniones de los demás,
pensamientos, emociones,
saber, todo lo cual rechazar estereotipos o
vivencias y opiniones, así • Comprender y saber
contribuye a su vez a expresiones sexistas,
como dialogar, formarse un comunicar son saberes prác-
ticos que han de apoyarse conservar y mejorar la discriminatorias, etc. Por
juicio crítico y ético, generar
en el conocimiento reflexivo competencia comunicativa. otra, en relación con uno
ideas, estructurar el
sobre el funcionamiento del mismo, actitudes que
conocimiento, dar
lenguaje y sus normas de • La habilidad para favorecen la mejora en la
coherencia y cohesión al
uso, e implican la capacidad seleccionar y aplicar expresión oral o escrita, la
discurso y a las propias
de tomar el lenguaje como determinados propósitos u precisión en el lenguaje, y la
acciones y tareas, adoptar
Comunicación lingüistica

objeto de observación y objetivos a las acciones confianza para expresarse


decisiones, y disfrutar
análisis. Expresar e interpretar propias de la comunicación en público y por escrito.
escuchando, leyendo o
diferentes tipos de discurso lingüística (el diálogo, la
expresándose de forma oral acordes a la situación lectura, la escritura, etc.) está • Una actitud favorable a
y escrita, todo lo cual comunicativa en diferentes vinculada a algunos rasgos la lectura facilita
contribuye además al contextos sociales y fundamentales de esta enormemente la adquisición
desarrollo de la autoestima y culturales, implica el competencia como las de las destrezas necesarias
de la confianza en sí mismo. conocimiento y aplicación
habilidades para representar- para la interpretación y
efectiva de las reglas de comprensión del código
• Comunicarse y conversar se mentalmente, interpretar
funcionamiento del sistema
y comprender la realidad, y que permite hacer uso de la
son acciones que suponen de la lengua y de las
organizar y autorregular el lengua escrita y, además, es
habilidades para establecer estrategias necesarias para
interactuar lingüísticamente conocimiento y la acción fuente de placer, de
vínculos y relaciones
de una manera adecuada. dotándolos de coherencia. descubrimiento de otros
constructivas con los demás
mundos no cotidianos, de
y con el entorno, y
• Disponer de esta • Con distinto nivel de fantasía, de información, de
acercarse a nuevas culturas,
competencia conlleva tener dominio y formalización aprendizaje lingüístico y de
que adquieren consideración conciencia de las –especialmente en lengua saber.
y respeto en la medida en convenciones sociales, de escrita – esta competencia
que se conocen. Por ello, la los valores y aspectos significa, en el caso de las • Del mismo modo, es
competencia de culturales y de la lenguas extranjeras, poder precisa la valoración de las
comunicación lingüística está versatilidad del lenguaje en
comunicarse en algunas de lenguas extranjeras como
presente en la capacidad función del contexto y la
ellas y, con ello, enriquecer medio para comunicarse y
efectiva de convivir y de intención comunicativa.
las relaciones sociales y relacionarse con
resolver conflictos.
• Estos conocimientos desenvolverse en contextos compañeros y compañeras
suponen tomar conciencia distintos al propio. de otros países, como
• El lenguaje, como
de la diversidad de textos y Asimismo, se favorece el posibilidad de acceso a
herramienta de comprensión
de la estructura de cada acceso a más y diversas informaciones nuevas y
y representación de la
uno de ellos, de estrategias fuentes de información, como instrumento para
realidad, debe ser
de coherencia y cohesión comunicación y aprendizaje. conocer otras culturas.
instrumento para la en textos orales y escritos,
igualdad, la construcción de así como de comprensión y • En síntesis, el desarrollo de
relaciones iguales entre producción de textos a la competencia lingüística al
hombres y mujeres, la partir de modelos. final de la educación
eliminación de estereotipos y
• En síntesis, el desarrollo de obligatoria comporta el
expresiones sexistas. La
la competencia lingüística al dominio de la lengua oral y
comunicación lingüística
final de la educación escrita en múltiples
debe ser motor de la
obligatoria comporta el contextos, y el uso funcional
resolución pacífica de
dominio de la lengua oral y de, al menos, una lengua
conflictos en la comunidad
escrita en múltiples extranjera.
escolar.
contextos, y el uso funcional
de, al menos, una lengua
extranjera.

62 Anexo II: Descripción de las Competencias Básicas


Evaluación General de Diagnóstico 2009
Marco de la evaluación

Definición y finalidad
Conocimientos Destrezas Actitudes
de la competencia
• Habilidad para utilizar y • Implica el conocimiento • Destrezas necesarias para • Disposición favorable y de
relacionar los números, sus y manejo de los elementos aplicar los principios y los progresiva seguridad y
operaciones básicas, los matemáticos básicos procesos matemáticos confianza hacia la
símbolos y las formas de (distintos tipos de números, básicos en situaciones información y las situaciones
expresión y razonamiento medidas, símbolos, cotidianas del ámbito (problemas, incógnitas, etc.)
matemático tanto para elementos geométricos, personal, social y laboral, que contienen elementos o
producir e interpretar etc.) en situaciones reales o para seguir y valorar soportes matemáticos, así
distintos tipos de simuladas de la vida cadenas argumentales, como hacia su utilización
información, como para cotidiana, y la puesta en identificando las ideas cuando la situación lo
ampliar el conocimiento práctica de procesos fundamentales, y estimar y aconseja, basadas en el
sobre aspectos cuantitativos de razonamiento que llevan enjuiciar la lógica y validez respeto y el gusto por la
y espaciales de la realidad y a la solución de los de argumentaciones e certeza y en su búsqueda a
para resolver problemas problemas o a la obtención informaciones. través del razonamiento.
relacionados con la vida de información.
cotidiana y el mundo • Habilidad para seguir
laboral. determinados procesos de
pensamiento (como la
Finalidad inducción y la deducción,
entre otros) y para aplicar
• Forma parte de la
algunos algoritmos de
competencia matemática, la
cálculo o elementos de la
habilidad para interpretar y
lógica, lo que conduce a
expresar con claridad y
identificar la validez de los
precisión informaciones,
razonamientos y a valorar el
datos y argumentaciones, lo
grado de certeza asociado a
que aumenta la posibilidad
los resultados derivados de
real de seguir aprendiendo
los razonamientos válidos.
a lo largo de la vida, tanto
Matemática

en el ámbito escolar o • El desarrollo de la


académico como fuera competencia matemática al
de él, y favorece la final de la educación
participación efectiva en la obligatoria, conlleva utilizar
vida social. espontáneamente, en los
ámbitos personal y
• Esta competencia cobra
social, los elementos y
realidad y sentido en la
razonamientos matemáticos
medida que los elementos y
para interpretar y producir
razonamientos matemáticos
información, para resolver
son utilizados para
problemas provenientes de
enfrentarse a aquellas
situaciones cotidianas y para
situaciones cotidianas que los
tomar decisiones. En
precisan. Por tanto, la
definitiva, supone aplicar
identificación de tales
aquellas destrezas y
situaciones, la aplicación de
actitudes que permiten
estrategias de resolución de
razonar matemáticamente,
problemas, y la selección de
comprender una argumen-
las técnicas adecuadas para
tación matemática y
calcular, representar e
expresarse y comunicarse en
interpretar la realidad a partir
el lenguaje matemático,
de la información disponible
utilizando las herramientas
están incluidas en ella. En
de apoyo adecuadas, e
definitiva, la posibilidad real
integrando el conocimiento
de utilizar la actividad
matemático con otros tipos
matemática en contextos tan
de conocimiento para dar
variados como sea posible.
una mejor respuesta a las
Por ello, su desarrollo en la
situaciones de la vida de
educación obligatoria se
distinto nivel de
alcanzará en la medida en
complejidad.
que los conocimientos
matemáticos se apliquen de
manera espontánea a una
amplia variedad de
situaciones, provenientes de
otros campos de
conocimiento y de la vida
cotidiana.

Anexo II: Descripción de las Competencias Básicas 63


Evaluación General de Diagnóstico 2009
Marco de la evaluación

Definición y finalidad
Conocimientos Destrezas Actitudes
de la competencia
• Habilidad para interactuar • El desarrollo de esta • Entre las destrezas • Las actitudes necesarias para
con el mundo físico, tanto en competencia requiere el necesarias para alcanzar esta alcanzar esta competencia
sus aspectos naturales como aprendizaje de los conceptos competencia destacan, por una están relacionadas con la
en los generados por la acción básicos que permiten el parte las asociadas a la valoración del conocimiento
humana, de modo que facilite análisis de los fenómenos aplicación del pensamiento científico, de las formas de
la comprensión de sucesos, la desde los diferentes campos científico (plantear preguntas, obtenerlo y de la información
predicción de consecuencias y de conocimiento científico formas de proceder, asociada a él, frente a las
la actividad dirigida a la involucrados en ella. estrategias de evaluación, formas no científicas de
mejora y preservación de las inferencias, etc.) y que acercamiento a la realidad.
• En el ámbito de esta
condiciones de vida propia, de permiten la comprensión y
competencia se encuentran • Asimismo es necesaria la
los demás hombres y mujeres los conceptos, de las resolución de problemas
y del resto de los seres vivos. sobre el mundo natural y lo valoración de las cuestiones
diferentes dimensiones del éticas asociadas al
ámbito científico-técnico, tecnológico. Esto supone
Finalidad poner en juego procesos conocimiento científico, al
necesarios para la realización
cognitivos más o menos desarrollo tecnológico, a la
• Esta competencia se refiere a de los procesos propios del
complejos, tales como relación de las personas con la
ámbitos del conocimiento muy pensamiento científico y de las
relaciones elementales entre observar indirectamente, naturaleza y al uso del
diversos (ciencia, salud, comparar en función de propio cuerpo.
actividad física, consumo, ellos que permiten asociar las
causas con los efectos y, en su criterios no directamente
procesos tecnológicos, etc.) observables o plantear • Colaboran asimismo a esta
caso, la cuantificación de unos
hipótesis, con el objetivo de competencia determinadas
• Los avances que se van y otros. Entre estos conceptos
conocer mejor los fenómenos actitudes en relación con el
produciendo en los ámbitos están las magnitudes físicas
elementales, los asociados a naturales y las máquinas y, en entorno natural, como el uso
científico y tecnológico, y la
las sustancias, sus la medida de lo posible, responsable de los recursos
influencia decisiva que tienen
características, los cambios en predecir el comportamiento de naturales, la conservación del
sobre el medio ambiente, la
ellas y los efectos que pueden unos y otras. Todo ello debe medio ambiente y de la
salud y la calidad de vida de
Conocimiento e interacción con el mundo físico

producir sobre las personas y llevar a la explicación, diversidad de la Tierra y la


los ciudadanos, hacen que
sobre el medio ambiente. utilizando criterios científicos, valoración de la incidencia de
cada vez sea más importante
de algunos de los cambios más la acción humana en la
el desarrollo del pensamiento • Relacionado con lo anterior destacados que tienen lugar en biosfera. Además, en el
científico-técnico para está también la comprensión la naturaleza, espontáneos o ámbito de la salud son
interpretar la información que de los procesos básicos provocados. esenciales también las
se recibe y para predecir y mediante los cuales se actitudes asociadas al
tomar decisiones sobre ellos producen las transformaciones • Colaboran al desarrollo de mantenimiento de un
de manera autónoma y, en en la naturaleza, espontáneas o esta competencia las régimen de vida saludable, a
general, para comprender y inducidas (de sustancias, de destrezas asociadas a la una adecuada alimentación y
resolver problemas en el formas de energía...) y el papel localización y orientación, a al rechazo al consumo de
mundo actual. de los objetos técnicos y, en la representación y a la sustancias nocivas.
general, de las acciones
• El adecuado desarrollo de obtención de información,
humanas en esas
esta competencia requiere cualitativa y cuantitativa, y a la
transformaciones.
tener en cuenta las diferentes resolución de problemas sobre
dimensiones presentes en el • En particular, debe el espacio físico y sobre los
ámbito científico y adquirirse un conocimiento fenómenos que se producen
tecnológico. Por ello, se debe suficiente de los diferentes en él. La habilidad con lo
trabajar, por una parte, sobre componentes de un tecnológico requiere, además,
el conocimiento de los ecosistema y de las relaciones destrezas asociadas a la plani-
objetos y los espacios que se establecen y ficación y manejo, siguiendo
cotidianos, los procesos comprensión de los cambios criterios de economía y
tecnológicos, la distribución que se producen en la eficacia, de soluciones
de los fenómenos en el naturaleza y en la dinámica de técnicas para cubrir las
espacio geográfico y el medio la tierra por la forma en la que necesidades habituales de la
ambiente, aplicando criterios interactúan los seres vivos vida cotidiana y el mundo
asociados al pensamiento entre sí y con el medio. laboral.
científico: planteándose
• En relación con el propio • Por último, se debe señalar
preguntas, razonando sobre la
cuerpo, es necesario el todo el conjunto de destrezas
experiencia, realizando
conocimiento de los asociadas al mantenimiento y
inferencias, etc.
beneficios para la salud de mejora de la salud y la
• Pero, por otra parte, se trata determinadas prácticas condición física propia, así
de que al tomar decisiones (ejercicio físico, seguridad, como de la biodiversidad y las
se tenga en cuenta la higiene) y de los riesgos de
condiciones saludables del
importancia del uso determinados hábitos
entorno.
responsable de los recursos dependientes de actividades
naturales, la preservación del profesionales o personales
medio ambiente, el consumo (formas de alimentación,
racional y responsable de los prácticas antihigiénicas,
productos y el fomento de la consumo de sustancias
tóxicas, etc.) y de los riesgos
cultura de protección de la
para el medio ambiente de
salud como elemento clave de
determinadas actividades
la calidad de vida de las
humanas.
personas.
• En relación con el espacio,
• Las habilidades asociadas al
está asociado a esta
movimiento en el espacio
competencia el conocimiento
físico y a la salud se conjugan de los rasgos esenciales del
en otro de los aspectos de espacio natural en el que se
esta competencia, el referido desarrolla la vida y la
a la actividad física y al actividad humana, la
control del propio cuerpo. interacción de las personas con
ese espacio, la ocupación,
utilización y ordenación, así
como los paisajes geográficos
resultantes.

64 Anexo II: Descripción de las Competencias Básicas


Evaluación General de Diagnóstico 2009
Marco de la evaluación

Definición y finalidad
Conocimientos Destrezas Actitudes
de la competencia

• Consiste en disponer de • Técnicas y estrategias • Destrezas relacionadas con • Actitud positiva ante las
habilidades para buscar, diversas para acceder a la la búsqueda, selección, nuevas tecnologías de la
obtener, procesar y comunicar información según la fuente a registro y tratamiento o información y la
la información y transformarla la que se acuda y el soporte análisis de la información, comunicación, como una
en conocimiento. Incorpora que se utilice (oral, impreso, procedente de fuentes fuente potencial de
diferentes habilidades, que audiovisual, digital o tradicionales (libros, enriquecimiento personal y
van desde el acceso y multimedia). Requiere el diccionarios, atlas, medios de social. Interés por utilizar las
selección de la información dominio de lenguajes comunicación social, etc.), de TIC como herramienta de
hasta su transmisión de ésta específicos básicos (textual, aplicaciones multimedia y de aprendizaje y de
en distintos soportes una vez numérico, icónico, visual, las TIC (buscadores, itinerarios comunicación y por seguir
tratada, incluyendo la utiliza- gráfico y sonoro) y de sus de búsqueda relevantes, etc.). aprendiendo nuevos usos y
ción de las tecnologías de la pautas de decodificación y posibilidades.
información y la comunicación transferencia, así como saber • Disponer de información no
como un elemento esencial aplicar en distintas situaciones produce de forma automática • Valoración positiva del uso
para informarse y y contextos el conocimiento conocimiento. de las tecnologías para trabajar
comunicarse. Transformar la información en de forma autónoma y en
de los diferentes tipos de
información, sus fuentes, sus conocimiento exige de grupos, como instrumento de
Finalidad
posibilidades y su localización, destrezas de razonamiento colaboración y de desarrollo
• Ser competente en la así como los lenguajes y para organizarla, relacionarla, de proyectos de trabajo
utilización de las tecnologías soportes más frecuentes en los analizarla, sintetizarla y hacer cooperativos.
de la información y la que ésta suele expresarse. inferencias y deducciones de En síntesis, el tratamiento
comunicación como distinto nivel de complejidad; de la información y la
instrumento de trabajo en definitiva, comprenderla e competencia digital implican
intelectual incluye utilizarlas
Tratamiento de la información y competencia digital

integrarla en los esquemas ser una persona autónoma,


en su doble función de previos de conocimiento. eficaz, responsable, crítica y
transmisoras y generadoras de Significa, asimismo, reflexiva al seleccionar, tratar y
información y conocimiento. comunicar la información y los utilizar la información y sus
Se utilizarán en su función conocimientos adquiridos fuentes, así como las distintas
generadora al emplearlas, por
empleando recursos herramientas tecnológicas;
ejemplo, como herramienta
expresivos que incorporen, no también tener una actitud
en el uso de modelos de
sólo diferentes lenguajes y critica y reflexiva en la
procesos matemáticos, físicos,
técnicas específicas, sino valoración de la información
sociales, económicos o
también las posibilidades que disponible, contrastándola
artísticos. Asimismo, esta
competencia permite procesar ofrecen las tecnologías de la cuando es necesario, y
y gestionar adecuadamente información y la respetar las normas de
información abundante y comunicación. conducta acordadas
compleja, resolver problemas socialmente para regular el
• En definitiva, la uso de la información y sus
reales, tomar decisiones,
competencia digital comporta fuentes en los distintos
trabajar en entornos
hacer uso habitual de los soportes.
colaborativos ampliando los
recursos tecnológicos
entornos de comunicación
para participar en disponibles para resolver
comunidades de aprendizaje problemas reales de modo
formales e informales, y eficiente. Al mismo tiempo,
generar producciones posibilita evaluar y seleccionar
responsables y creativas. nuevas fuentes de información
e innovaciones tecnológicas a
• La competencia digital medida que van apareciendo,
incluye utilizar las tecnologías en función de su utilidad para
de la información y la acometer tareas u objetivos
comunicación extrayendo su específicos.
máximo rendimiento a partir
de la comprensión de la
naturaleza y modo de operar
de los sistemas tecnológicos, y
del efecto que esos cambios
tienen en el mundo personal y
sociolaboral. Asimismo supone
manejar estrategias para
identificar y resolver los
problemas habituales de
software y hardware que
vayan surgiendo. Igualmente
permite aprovechar la
información que proporcionan
y analizarla de forma crítica
mediante el trabajo personal
autónomo y el trabajo
colaborativo, tanto en su
vertiente sincrónica como
diacrónica, conociendo y
relacionándose con entornos
físicos y sociales cada vez más
amplios. Además de utilizarlas
como herramienta para
organizar la información,
procesarla y orientarla para
conseguir objetivos y fines de
aprendizaje, trabajo y ocio
previamente establecidos.

Anexo II: Descripción de las Competencias Básicas 65


Evaluación General de Diagnóstico 2009
Marco de la evaluación

Definición y finalidad
Conocimientos Destrezas Actitudes
de la competencia
• Esta competencia hace • Requiere entender los • Forman parte • Demostrar comprensión
posible comprender la rasgos de las sociedades fundamental de esta de la aportación que las
realidad social en que se actuales, su creciente competencia aquellas diferentes culturas han
vive, cooperar, convivir y pluralidad y su carácter habilidades sociales que hecho a la evolución y
ejercer la ciudadanía evolutivo. permiten saber que los progreso de la humanidad,
democrática en una conflictos de valores e y disponer de un
sociedad plural, así como • La dimensión ética de la intereses forman parte de la sentimiento común de
comprometerse a contribuir competencia social y convivencia, resolverlos con pertenencia a la sociedad
a su mejora. En ella están ciudadana entraña ser actitud constructiva y tomar en que se vive. En
integrados conocimientos consciente de los valores del decisiones con autonomía definitiva, mostrar un
diversos y habilidades entorno, evaluarlos y empleando, tanto los sentimiento de ciudadanía
complejas que permiten reconstruirlos afectiva y conocimientos sobre la global compatible con la
participar, tomar decisiones, racionalmente para crear sociedad como una escala identidad local.
elegir cómo comportarse en progresivamente un sistema de valores construida
determinadas situaciones y de valores propio y mediante la reflexión crítica • Ejercer una ciudadanía
responsabilizarse de las comportarse en coherencia y el diálogo en el marco de activa e integradora y
elecciones y decisiones con ellos al afrontar una los patrones culturales mostrar un comportamiento
adoptadas. decisión o un conflicto. Ello básicos de cada región, país coherente con los valores
supone entender que no o comunidad. democráticos, que a su vez
Finalidad toda posición personal es conlleva disponer de
ética si no está basada en el • Entre las habilidades de habilidades como la toma
• Globalmente supone
respeto a principios o esta competencia destacan de conciencia de los
utilizar, para desenvolverse
valores universales como los conocerse y valorarse, saber propios pensamientos,
socialmente, el
que encierra la Declaración comunicarse en distintos valores, sentimientos y
conocimiento sobre la
de los Derechos Humanos. contextos, expresar las acciones, y el control y
evolución y organización de
propias ideas y escuchar las autorregulación de los
las sociedades y sobre los
• Conocimiento y ajenas, ser capaz de ponerse mismos.
rasgos y valores del sistema
comprensión de los valores en el lugar del otro y
democrático, así como
en que se asientan los comprender su punto de • En síntesis, esta
utilizar el juicio moral para
Social y ciudadana

estados y sociedades vista aunque sea diferente competencia supone


elegir y tomar decisiones,
democráticas, de sus del propio, y tomar comprender la realidad
y ejercer activa y
fundamentos, modos de decisiones en los distintos social en que se vive,
responsablemente los
organización y niveles de la vida afrontar la convivencia y los
derechos y deberes de la
funcionamiento. Esta comunitaria, valorando conflictos empleando el
ciudadanía.
competencia permite conjuntamente los intereses juicio ético basado en los
reflexionar críticamente individuales y los del grupo. valores y prácticas
• Esta competencia favorece
sobre los conceptos de Además implica, la democráticas, y ejercer la
la comprensión de la
democracia, libertad, valoración de las diferencias ciudadanía, actuando con
realidad histórica y social
igualdad, solidaridad, a la vez que el criterio propio,
del mundo, su evolución,
corresponsabilidad, reconocimiento de la contribuyendo a la
sus logros y sus problemas.
participación y ciudadanía, igualdad de derechos entre construcción de la paz y la
La comprensión crítica de la
con particular atención a los los diferentes colectivos, en democracia, y manteniendo
realidad exige experiencia,
derechos y deberes particular, entre hombres y una actitud constructiva,
conocimientos y conciencia
reconocidos en las mujeres. Igualmente la solidaria y responsable ante
de la existencia de distintas
declaraciones práctica del diálogo y de la el cumplimiento de los
perspectivas al analizar esa
internacionales, en la negociación para llegar a derechos y obligaciones
realidad. Conlleva recurrir al
Constitución española y en acuerdos como forma de cívicas.
análisis multicausal y
la legislación autonómica, resolver los conflictos, tanto
sistémico para enjuiciar los
así como a su aplicación por en el ámbito personal como
hechos y problemas sociales
parte de diversas en el social.
e históricos y para
instituciones.
reflexionar sobre ellos de
• En definitiva, el ejercicio
forma global y crítica,
de la ciudadanía implica
así como realizar
disponer de habilidades
razonamientos críticos y
para participar activa y
lógicamente válidos sobre
plenamente en la vida
situaciones reales, y
cívica. Significa construir,
dialogar para mejorar
aceptar y practicar normas
colectivamente la
de convivencia acordes con
comprensión de la realidad.
los valores democráticos,
ejercitar los derechos,
libertades, responsabilidades
y deberes cívicos, y
defender los derechos de
los demás.

66 Anexo II: Descripción de las Competencias Básicas


Evaluación General de Diagnóstico 2009
Marco de la evaluación

Definición y finalidad
Conocimientos Destrezas Actitudes
de la competencia
• Esta competencia supone • Conocimiento básico de • Apreciar el hecho cultural • Supone una actitud de
conocer, comprender las principales técnicas, en general, y el hecho aprecio de la creatividad
apreciar, y valorar recursos y convenciones de artístico en particular, lleva implícita en la expresión de
críticamente diferentes los diferentes lenguajes implícito disponer de ideas, experiencias o
manifestaciones culturales y artísticos, así como de las aquellas habilidades y sentimientos a través de
artísticas, utilizarlas como obras y manifestaciones actitudes que permiten diferentes medios artísticos,
fuente de enriquecimiento y más destacadas del acceder a sus distintas como la música, la
disfrute y considerarlas patrimonio cultural. Además manifestaciones, así literatura, las artes visuales y
como parte del patrimonio supone identificar las como habilidades de escénicas, o de las
cultural de los pueblos. relaciones existentes entre pensamiento, perceptivas y diferentes formas que
esas manifestaciones y la comunicativas, sensibilidad adquieren las llamadas artes
Finalidad sociedad -la mentalidad y y sentido estético para populares. Exige asimismo
• Esta competencia implica las posibilidades técnicas de poder comprenderlas, valorar la libertad de
poner en juego habilidades la época en que se crean-, valorarlas, emocionarse y expresión, el derecho a la
de pensamiento divergente o con la persona o disfrutarlas. diversidad cultural, la
y convergente, puesto que colectividad que las crea. importancia del diálogo
Esto significa también tener • En síntesis, el conjunto de intercultural y la realización
comporta reelaborar ideas y
conciencia de la evolución destrezas que configuran de experiencias artísticas
sentimientos propios y
del pensamiento, de las esta competencia se refiere compartidas.
ajenos; encontrar fuentes,
corrientes estéticas, las tanto a la habilidad para
formas y cauces de
modas y los gustos, así apreciar y disfrutar con el
comprensión y expresión;
como de la importancia arte y otras manifestaciones
planificar, evaluar y ajustar
representativa, expresiva y culturales, como a aquellas
los procesos necesarios para
comunicativa que los relacionadas con el empleo
alcanzar unos resultados, ya
factores estéticos han de algunos recursos de la
sea en el ámbito personal o
desempeñado y expresión artística para
académico. Se trata, por
desempeñan en la vida realizar creaciones propias;
tanto, de una competencia
cotidiana de la persona y implica un conocimiento
que facilita tanto expresarse
de las sociedades. básico de las distintas
y comunicarse como
Cultural y artística

manifestaciones culturales y
percibir, comprender y
artísticas, la aplicación de
enriquecerse con diferentes
habilidades de pensamiento
realidades y producciones
divergente y de trabajo
del mundo del arte y de la
colaborativo, una actitud
cultura.
abierta, respetuosa y crítica
• Requiere poner en hacia la diversidad de
funcionamiento la iniciativa, expresiones artísticas y
la imaginación y la culturales, el deseo y
creatividad para expresarse voluntad de cultivar la
mediante códigos artísticos propia capacidad estética y
y, en la medida en que las creadora, y un interés por
actividades culturales y participar en la vida cultural
artísticas suponen en y por contribuir a la
muchas ocasiones un conservación del patrimonio
trabajo colectivo, es preciso cultural y artístico, tanto de
disponer de habilidades de la propia comunidad, como
cooperación para contribuir de otras comunidades.
a la consecución de un
resultado final, y tener
conciencia de la
importancia de apoyar y
apreciar las iniciativas y
contribuciones ajenas.

Anexo II: Descripción de las Competencias Básicas 67


Evaluación General de Diagnóstico 2009
Marco de la evaluación

Definición y finalidad
Conocimientos Destrezas Actitudes
de la competencia

• Aprender a aprender • Esta competencia requiere • Manejo de distintas • Disponer de un sentimien-


supone disponer de el conocimiento de las pro- estrategias y técnicas: de to de competencia personal,
habilidades para iniciarse en pias capacidades (intelectua- estudio, de observación y que redunda en la
el aprendizaje y ser capaz de les, emocionales, físicas), del registro sistemático de motivación, la confianza en
continuar aprendiendo de proceso y las estrategias hechos y relaciones, de uno mismo y el gusto por
manera cada vez más eficaz necesarias para desarrollarlas, trabajo cooperativo y por aprender.
y autónoma de acuerdo con así como de lo que se puede proyectos, de resolución de
los propios objetivos y hacer por uno mismo y de lo problemas, de planificación y • Curiosidad de plantearse
necesidades. que se puede hacer con organización de actividades y preguntas, identificar y
ayuda de otras personas o tiempos de forma efectiva, o manejar la diversidad de
recursos. del conocimiento sobre los respuestas posibles ante una
Finalidad misma situación o problema
diferentes recursos y fuentes
• La adquisición de esta • Consciencia de lo que se para la recogida, selección utilizando diversas estrategias
competencia supone una sabe y de lo que es necesario y tratamiento de la y metodologías que permitan
mejora en la capacidad de aprender, de cómo se información, incluidos los afrontar la toma de
enfrentarse con éxito al aprende, y de cómo se recursos tecnológicos. decisiones, racional y
aprendizaje autónomo ya gestionan y controlan de críticamente, con la
que hace permite apoyarse forma eficaz los procesos de • Habilidades para obtener información disponible.
en experiencias vitales y de aprendizaje, optimizándolos información -ya sea
aprendizajes anteriores con el y orientándolos a satisfacer individualmente o en • Por otra parte, esta
fin de utilizar y aplicar los objetivos personales. colaboración- y, muy competencia requiere
nuevos conocimientos y Requiere conocer las propias especialmente, para plantearse metas alcanzables
capacidades en diversos potencialidades y carencias, transformarla en a corto, medio y largo plazo
contextos. Saber aprender es sacando provecho de las conocimiento propio, y cumplirlas, elevando los
indispensable para conseguir primeras y teniendo relacionando e integrando la objetivos de aprendizaje de
los mejores logros de las motivación y voluntad para nueva información con los forma progresiva y realista.
capacidades individuales. superar las segundas desde conocimientos previos y • Perseverancia en el
Implica tomar conciencia de una expectativa de con la propia experiencia aprendizaje, desde su
que el aprendizaje tiene un éxito, aumentando personal y sabiendo aplicar valoración como un
coste, pero que éste es progresivamente la seguridad los nuevos conocimientos y elemento que enriquece la
asumible y puede ser una para afrontar nuevos retos capacidades en situaciones vida personal y social y que
fuente de competencia de aprendizaje. parecidas y contextos es, por tanto, merecedor del
personal. diversos. esfuerzo que requiere.
• Por ello, comporta tener
• Esta competencia se conciencia de aquellas • En síntesis, aprender a Conlleva ser capaz de
autoevaluarse y
Aprender a aprender

fortalece cuando se capacidades que entran en aprender implica la


comprueba que el resultado juego en el aprendizaje, conciencia, gestión y control autorregularse,
del aprendizaje es realmente como la atención, la de las propias capacidades y responsabilidad y
eficaz o, si no lo es, se concentración, la memoria, conocimientos desde un compromiso personal, saber
identifica el error y uno se la comprensión y la sentimiento de competencia administrar el esfuerzo,
siente capaz de subsanarlo. expresión lingüística o la o eficacia personal, e incluye aceptar los errores y
De este modo, si se es capaz motivación de logro, tanto el pensamiento aprender de y con los
de plantear metas entre otras, y obtener un estratégico, como la demás.
alcanzables a corto plazo y rendimiento máximo y capacidad de cooperar, de
cumplirlas, se podrán ir personalizado de las autoevaluarse, y el manejo
elevando los objetivos de mismas. eficiente de un conjunto de
aprendizaje de forma recursos y técnicas de trabajo
progresiva y realista. intelectual, todo lo cual se
desarrolla a través de
• El proceso de aprender a experiencias de aprendizaje
aprender no se circunscribe a conscientes y gratificantes,
una disciplina sino que tanto individuales como
afecta al desarrollo del colectivas.
pensamiento y al propio
proceso del aprendizaje,
repercutiendo en aspectos
personales y de relación social,
en la medida en que supone
ser capaz de compensar
carencias y sacar el mejor
partido del conocimiento de
uno mismo, lo que conlleva
una progresiva madurez
personal. Su desarrollo afecta
directamente a otras
competencias relacionadas
con la obtención y
procesamiento de la
información y el desarrollo de
la iniciativa personal.
• La práctica de esta
competencia debe iniciarse
con la enseñanza obligatoria,
por ser el período más
adecuado para la adquisición
de hábitos. Las limitaciones en
su adquisición de esta
competencia durante ese
periodo condicionan y
pueden mermar las
posibilidades de aprendizajes
posteriores y con ello la
adaptación a los cambios en
los ámbitos personal, social y
laboral.

68 Anexo II: Descripción de las Competencias Básicas


Evaluación General de Diagnóstico 2009
Marco de la evaluación

Definición y finalidad Conocimientos Destrezas Actitudes


de la competencia
• Esta competencia se • Exige tener una visión • Supone poder transformar • Requiere una actitud
refiere, por una parte, a la estratégica de los retos y las ideas en acciones; es positiva hacia el cambio y
adquisición de la conciencia oportunidades que ayude decir, proponerse objetivos la innovación que
y aplicación de un conjunto a identificar y cumplir y planificar y llevar a cabo presupone flexibilidad de
de valores y actitudes objetivos y a mantener la proyectos. planteamientos, pudiendo
personales interrelacionadas, motivación para lograr Requiere, por tanto, comprender dichos cambios
como la responsabilidad, el éxito en las tareas poder reelaborar los como oportunidades,
la perseverancia, el emprendidas, con una planteamientos previos o adaptarse crítica y
conocimiento de sí mismo sana ambición personal, elaborar nuevas ideas, constructivamente a ellos,
y la autoestima, la académica y profesional. buscar soluciones y llevarlas afrontar los problemas y
creatividad, la autocrítica, el Igualmente ser capaz de a la práctica. Además, encontrar soluciones en
control emocional, la poner en relación la oferta analizar posibilidades y cada uno de los proyectos
capacidad de elegir, de académica, laboral o de limitaciones, conocer las vitales que se emprenden.
calcular riesgos y de ocio. fases de desarrollo de un
afrontar los problemas, así proyecto, planificar, tomar
como la capacidad de decisiones, actuar, evaluar
demorar la necesidad de lo hecho y autoevaluarse,
satisfacción inmediata, de extraer conclusiones y
aprender de los errores y de valorar las posibilidades de
asumir riesgos. mejora.

• Por otra parte, remite a la • En la medida en que la


capacidad de elegir con autonomía e iniciativa
criterio propio, de imaginar personal involucran a
proyectos, y de llevar menudo a otras personas,
adelante las acciones esta competencia obliga a
necesarias para desarrollar
Autonomía e iniciativa personal

disponer de habilidades
las opciones y planes sociales para relacionarse,
personales -en el marco cooperar y trabajar en
de proyectos individuales equipo ponerse en el lugar
o colectivos- del otro, valorar las ideas de
responsabilizándose de los demás, dialogar y
ellos, tanto en el ámbito negociar, la asertividad para
personal, como social y hacer saber adecuadamente
laboral. a los demás las propias
decisiones, y trabajar de
forma cooperativa y flexible.

• Otra dimensión importan-


te de esta competencia,
muy relacionada con esta
vertiente más social, está
constituida por aquellas
habilidades y actitudes
relacionadas con el
liderazgo de proyectos, que
incluyen la confianza en
uno mismo, la empatía, el
espíritu de superación, las
habilidades para el diálogo
y la cooperación, la
organización de tiempos y
tareas, la capacidad de
afirmar y defender derechos
o la asunción de riesgos.

• En síntesis, la autonomía y
la iniciativa personal
suponen ser capaz de
imaginar, emprender,
desarrollar y evaluar
acciones o proyectos
individuales o colectivos
con creatividad, confianza,
responsabilidad y sentido
crítico.

Anexo II: Descripción de las Competencias Básicas 69


ANEXO III

COMPETENCIA EN COMUNICACIÓN LINGÜÍSTICA

1. Presentación de la competencia en comunicación lingüística


Esta competencia se refiere a la utilización del lenguaje como instrumento de comunica-
ción oral y escrita, de representación, interpretación y comprensión de la realidad, de
construcción y comunicación del conocimiento y de organización y autorregulación del
pensamiento, las emociones y la conducta.

Los conocimientos, destrezas y actitudes propios de esta competencia permiten expresar


pensamientos, emociones, vivencias y opiniones, así como dialogar, formarse un juicio crí-
tico y ético, generar ideas, estructurar el conocimiento, dar coherencia y cohesión al dis-
curso y a las propias acciones y tareas, adoptar decisiones, y disfrutar escuchando, leyen-
do o expresándose de forma oral y escrita, todo lo cual contribuye además al desarrollo de
la autoestima y de la confianza en sí mismo.

Comunicarse y conversar son acciones que suponen habilidades para establecer vínculos
y relaciones constructivas con los demás y con el entorno, y acercarse a nuevas culturas,
que adquieren consideración y respeto en la medida en que se conocen. Por ello, la com-
petencia de comunicación lingüística está presente en la capacidad efectiva de convivir y
de resolver conflictos.

Escuchar, exponer y dialogar implica ser consciente de los principales tipos de interacción
verbal, ser progresivamente competente en la expresión y comprensión de los mensajes
orales que se intercambian en situaciones comunicativas diversas y adaptar la comunicación
al contexto. Supone también la utilización activa y efectiva de códigos y habilidades lingüís-
ticas y no lingüísticas y de las reglas propias del intercambio comunicativo en diferentes
situaciones, para producir textos orales adecuados a cada situación de comunicación.

Leer y escribir son acciones que suponen y refuerzan las habilidades que permiten buscar,
recopilar y procesar información, y ser competente a la hora de comprender, componer y

Anexo III: Competencia en Comunicación Lingüística 71


Evaluación General de Diagnóstico 2009
Marco de la evaluación

utilizar distintos tipos de textos con intenciones comunicativas o creativas diversas. La lec-
tura facilita la interpretación y comprensión del código que permite hacer uso de la len-
gua escrita y es, además, fuente de placer, de descubrimiento de otros entornos, idiomas
y culturas, de fantasía y de saber, todo lo cual contribuye a su vez a conservar y mejorar
la competencia comunicativa.

La habilidad para seleccionar y aplicar determinados propósitos u objetivos a las acciones


propias de la comunicación lingüística (el diálogo, la lectura, la escritura, etc.) está vincula-
da a algunos rasgos fundamentales de esta competencia como las habilidades para repre-
sentarse mentalmente, interpretar y comprender la realidad, y organizar y autorregular el
conocimiento y la acción dotándolos de coherencia.

Comprender y saber comunicar son saberes prácticos que han de apoyarse en el conoci-
miento reflexivo sobre el funcionamiento del lenguaje y sus normas de uso, e implican la
capacidad de tomar el lenguaje como objeto de observación y análisis. Expresar e inter-
pretar diferentes tipos de discurso acordes a la situación comunicativa en diferentes con-
textos sociales y culturales, implica el conocimiento y aplicación efectiva de las reglas de
funcionamiento del sistema de la lengua y de las estrategias necesarias para interactuar lin-
güísticamente de una manera adecuada.

Disponer de esta competencia conlleva tener conciencia de las convenciones sociales, de


los valores y aspectos culturales y de la versatilidad del lenguaje en función del contexto y
la intención comunicativa. Implica la capacidad empática de ponerse en el lugar de otras
personas; de leer, escuchar, analizar y tener en cuenta opiniones distintas a la propia con
sensibilidad y espíritu crítico; de expresar adecuadamente -en fondo y forma- las propias
ideas y emociones, y de aceptar y realizar críticas con espíritu constructivo.

Con distinto nivel de dominio y formalización -especialmente en lengua escrita- esta com-
petencia significa, en el caso de las lenguas extranjeras, poder comunicarse en algunas de
ellas y, con ello, enriquecer las relaciones sociales y desenvolverse en contextos distintos al
propio. Asimismo, se favorece el acceso a más y diversas fuentes de información, comuni-
cación y aprendizaje.

En síntesis, el desarrollo de la competencia lingüística al final de la educación obligatoria


comporta el dominio de la lengua oral y escrita en múltiples contextos, y el uso funcional
de, al menos, una lengua extranjera.4

Las referencias a la lengua castellana en este documento tendrán su equivalente a las otras
lenguas oficiales del Estado que se desarrollará por parte de las CC.AA correspondientes.

4 Se reproduce, como descripción básica de la competencia, la que aparece en los Reales Decretos de Mínimos para Educación
Primaria y para Educación Secundaria Obligatoria. Es de esta descripción de la que parten los distintos estudios para elaborar
las tablas que especifican el proceso de evaluación.

72 Anexo III: Competencia en Comunicación Lingüística


Evaluación General de Diagnóstico 2009
Marco de la evaluación

2. Dimensiones de la competencia

Tal y como se puede percibir a partir de la descripción en el apartado anterior, la comple-


jidad de la competencia en comunicación lingüística tiene como consecuencia la necesi-
dad de elaborar un marco de evaluación muy preciso. Ha de tener en cuenta, por un lado,
la competencia comunicativa en la propia lengua y en lengua extranjera y, en ambas, se
deben contemplar tanto las habilidades orales como las escritas y, en todos los casos, tanto
referidas a las capacidades de comprensión como a las de expresión.

Por otro lado, es preciso tener en cuenta que el proceso de evaluación de diagnóstico, al
ser general, tiene que concretarse en aquellos contenidos que todo estudiante español ha
de conocer. No son otros esos contenidos que los que figuran en los distintos Reales
Decretos de Mínimos5 elaborados tanto para la Educación Primaria como para la Educación
Secundaria Obligatoria.

Por último, como el objeto de evaluación no son en sí los contenidos sino la competencia
en su manejo, es preciso, primero definir en qué contextos o situaciones se van a situar las
distintas pruebas o ejercicios, en segundo lugar, describir el conjunto de procesos inscritos
en las distintas dimensiones de la competencia y, por último, identificar el conjunto de acti-
tudes asociadas a la evaluación de la competencia.

2.a. Contextos y situaciones

Tal y como ocurre con los apartados anteriores, los Reales Decretos de Mínimos propor-
cionan también información suficiente para establecer los contextos o situaciones en que
se han de plantear las unidades de evaluación para cada una de las destrezas. En las
páginas que siguen se describen esos contextos, asociados a las destrezas de comprensión
y expresión tanto oral como escrita. La descripción de cada contexto consta de dos
partes: una para describir la situación y temática tipo y otra para describir la modalidad
discursiva o textual susceptible de ser empleada en cada caso:

5 En todas las ocasiones en las que, a lo largo de este documento, se hace referencia a los Reales Decretos de Mínimos, se está
aludiendo a alguno de los siguientes marcos legales:
• Anexo I del RD 1513/2006, de 7 de diciembre, por el que se establecen las enseñanzas mínimas de la Educación
Primaria y del RD 1631/2006, de 29 de diciembre, por el que se establecen las enseñanzas mínimas correspondientes a
la Educación Secundaria Obligatoria.
• Anexo II del RD 1513/2006, de 7 de diciembre, por el que se establecen las enseñanzas mínimas de la Educación
Primaria y del RD 1631/2006, de 29 de diciembre, por el que se establecen las enseñanzas mínimas correspondientes a
la Educación Secundaria Obligatoria.

Anexo III: Competencia en Comunicación Lingüística 73


Evaluación General de Diagnóstico 2009
Marco de la evaluación

Competencia en comunicación lingüística / Contextos

Situación y temática Modalidades discursivas y textuales

Temática: históricos, de actualidad, Tipo de texto: continuo y discontinuo.


Comprensión lectora

sobre la naturaleza, temas escolares, Formas, tipologías y géneros: informativos, explicativos, argumentativos,
L. castellana /

temas familiares. descriptivos, narrativos, publicitarios, literarios (poemas y cuentos).


Situaciones: sociales, escolares,
4º EP

Vocabulario: común, específico de materias como las ciencias sociales, las


familiares y conocidas. naturales, el arte, etc.
Modelos de texto: cartas, normas de clase, reglas de juego, noticias, textos
escolares, tablas de doble entrada de uso habitual en la vida cotidiana, textos
dialogados.

Temática: históricos, sociales, de Tipo de texto: audiovisuales.


Comprensión oral

actualidad, sobre la naturaleza, temas


L. castellana /

Formas, tipologías y géneros: informativos, explicativos, argumentativos,


escolares, temas familiares. descriptivos, narrativos, publicitarios, literarios (cuentos).
4º EP

Situaciones: habituales en el Vocabulario: común, específico de materias como las ciencias sociales, las
entorno social y escolar, familiares y naturales, el arte, etc.
conocidas.
Modelos de texto: noticias orales, breves reportajes, exposiciones cortas,
instrucciones para realizar tareas, diálogos.

Temática: históricos, sociales, de Tipo de texto: continuos, discontinuos, visuales.


Expresión escrita /

actualidad, sobre la naturaleza, Formas, tipologías y géneros: informativos, normativos, instructivos,


L. castellana /

temas escolares, temas familiares, explicativos, argumentativos, descriptivos, narrativos, publicitarios, literarios
oficios.
4º EP

(cuentos).
Situaciones: cotidianas y escolares, Vocabulario: común, específico de materias como las ciencias sociales, las
hechos próximos a la experiencia de naturales, matemáticas, arte, etc.
los estudiantes.
Modelos de texto: cartas, normas de convivencia, avisos, solicitudes (pueden
incluir tablas, gráficos, imágenes, esquemas…), diálogos.

Temática: históricos, sociales, de Tipo de texto: continuos, discontinuos, visuales, gráficos.


Comprensión lectora /

actualidad, sobre la naturaleza, temas Formas, tipologías y géneros: descripción, narración, exposición, argumenta-
escolares, temas familiares, oficios, ción, textos publicitarios y periodísticos (crónicas, noticias), instructivos, artículos
L. castellana /

información científica y técnica, de diccionario, enciclopedia, glosario o web, textos literarios (cuento, poema,
2º ESO

información sobre hechos o texto teatral).


acontecimientos destacables.
Vocabulario: terminología básica específica de las distintas áreas, usos coloquiales
Situaciones: vida cotidiana, relacio- y formales, registro divulgativo.
nes sociales, situaciones próximas a la
experiencia del alumnado. Modelos de texto: planteamientos de problemas, noticias, crónicas, normas, avi-
sos, comunicaciones, artículos de diccionario, glosarios, enciclopedias, webs,
declaraciones públicas, instrucciones sencillas, textos dialogados.

Temática: información divulgativa de Tipo de texto: audiovisual, oral.


carácter científico y técnico, temas
Comprensión oral /

Formas, tipologías y géneros: descripción, narración, disertación,


sociales, informaciones de actualidad,
L. castellana /

argumentación, géneros periodísticos (noticia, crónica), documental, publicidad,


presentaciones de tareas para su exposiciones, instrucciones.
2º ESO

realización, temas escolares.


Vocabulario: terminología específica básica de distintas áreas del conocimiento,
Situaciones: presentación de un usos coloquiales y formales, registros divulgativos.
proyecto, presentación de un plan de
trabajo, temas próximos a la Modelos de texto: presentación de una tarea, instrucciones, declaraciones
experiencia del alumnado y al ámbito públicas, avisos, normas, noticias, diálogos.
académico.

Situaciones: Vida cotidiana, relaciones Tipo de texto: continuo, discontinuo, visual y gráfico.
sociales, ámbitos próximos a la Formas, tipologías y géneros: argumentaciones, descripción, narración,
experiencia del alumnado, situaciones exposición, publicitarios, periodísticos (crónicas, noticias), literarios (relato breve,
personales, sociales y culturales poema).
Expresión escrita /

destacables, participación en el ámbito


L. castellana /

público, desarrollo de proyectos, Vocabulario: vocabulario específico básico de las distintas materias, registro
planteamiento y resolución de coloquial, registro formal, registro técnico y académico.
2º ESO

problemas, situaciones de intercambio Modelos de texto: memorias, informes sobre tareas realizadas, cartas de
comercial, participación en foros. solicitud, resúmenes, breves exposiciones, glosarios, documentos técnicos,
Temática: fenómenos naturales, documentos que integren información textual, imágenes y gráficos, textos
demografía, rasgos de las sociedades, propios del ámbito personal, textos dialogados.
mundo de la subjetividad (ideas,
sentimientos, emociones), hechos de
actualidad, informes sobre tareas,
Ámbito académico y escolar.

74 Anexo III: Competencia en Comunicación Lingüística


Evaluación General de Diagnóstico 2009
Marco de la evaluación

Situación y temática Modalidades discursivas y textuales

Temática: temas conocidos. Tipo de texto: continuo, discontinuo.


Comprensión lectora /
Lengua extranjera /

Situaciones: situaciones en las que se Formas, tipologías y géneros: narraciones, descripciones, instrucciones.
pretenda una finalidad concreta, Vocabulario: palabras, términos y expresiones conocidos.
aspectos de la vida cotidiana referida
4º EP

a horarios, comidas, tradiciones, Modelos de texto: notas, normas de aula, cartas, carteles, cuentos, diálogos.
festividades y formas de relacionarse
de las personas.

Temática: temas familiares y de Tipo de texto: orales.


Lengua extranjera /
Comprensión oral /

interés. Vocabulario: conocido.


Modelos de texto: diálogos básicos, pequeñas narraciones.
4º EP

Temática y situaciones: necesidades Modelos de texto: partir de modelos con una finalidad determinada y con un
Lengua extranjera /

inmediatas como pedir permiso, formato establecido, notas, instrucciones o normas, cartas, carteles, folletos,
Expresión escrita /

pedir en préstamo, localizar objetos o cómics o descripciones sencillas, diálogos.


personas, el tiempo atmosféricos,
4º EP

gustos, habilidades, rutinas, hábitos,


lenguaje de aula.

Temática: asuntos familiares, Tipo de texto: continuos, discontinuos.


Comprensión lectora /
Lengua extranjera /

contenidos de otras materias. Vocabulario: expresiones comunes, frases hechas, léxico apropiado a contextos
Situaciones: pedir información y concretos cotidianos.
aclaración, pedir permiso, trabajar en
2º ESO

Modelos de texto: correspondencia, anuncios, folletos diversos, narraciones,


grupo, juegos, simulaciones, juegos artículos de revistas juveniles, páginas web, letras de canciones, diálogos.
de rol.

Temática: actividades habituales, Tipo de texto: orales.


Lengua extranjera /
Comprensión oral /

asuntos cotidianos y predecibles Modelos de texto: narraciones sencillas, diálogos.


procedentes de los medios de
comunicación.
2º ESO

Situaciones: pedir información y


aclaración, pedir permiso, trabajar
en grupo, juegos, simulaciones,
juegos de rol.

Temas: familiares, otras materias del Tipo de texto: continuos y discontinuos.


Lengua extranjera /

currículo, experiencias personales.


Expresión escrita /

Vocabulario: lenguaje formal y no formal, expresiones comunes, frases hechas,


Situaciones: situaciones habituales, léxico apropiado a situaciones cotidianas.
familiares, pedir información y
2º ESO

Modelos de texto: correspondencia postal o informática, textos diversos con


aclaración, pedir permiso, juegos, estructura y conectores sencillos, descripciones, narraciones, planes, proyectos,
simulaciones, juegos de rol. cartas, postales, formularios, diálogos.

Anexo III: Competencia en Comunicación Lingüística 75


Evaluación General de Diagnóstico 2009
Marco de la evaluación

2.b. Procesos

El planteamiento inicial para la confección del mapa de procesos toma como principio de
estructuración el hecho de que la evaluación ha de centrarse en las dos grandes áreas que
delimitan el campo de la competencia comunicativa: la de la comprensión y la de la expre-
sión. Como en la sociedad actual las destrezas comunicativas de cualquier hablante se des-
arrollan tanto, a través de procedimientos orales como escritos, estas dos grandes áreas se
subdividen, en otro nivel, en dos apartados cada una, lo que origina un mapa que se
puede representar del siguiente modo:

Área de la comprensión Área de la expresión

Oral Lectora Oral Escrita

Este esquema, que afecta a toda la competencia en comunicación lingüística, permite ya


identificar las cuatro destrezas básicas que van a ser objeto de evaluación:

Comprensión oral Comprensión lectora Expresión oral Expresión escrita

Para cada par de destrezas, el de comprensión y el de expresión, se han descrito, además,


los procesos básicos en los que se va a centrar la evaluación concebidos como conjuntos
de acciones encaminadas a un fin común. Tales procesos son los que se identifican en la
siguiente tabla:

Competencia comunicativa

Expresión

Comprensión

Textualización
Revisión y presentación
Transferencia y aplicación

Planificación
Reflexión y valoración
Integración y sintesis
e identificación
Aproximación

Organización

Adecuación
Coherencia

Cohesión

76 Anexo III: Competencia en Comunicación Lingüística


Evaluación General de Diagnóstico 2009
Marco de la evaluación

Lo que se obtiene, finalmente, son dos mapas de procesos: uno aplicable a las destrezas
relacionadas con la comprensión y otro a las de expresión:

Comprensión oral y lectora

Aproximación e Integración y Reflexión y Transferencia


identificación Organización síntesis valoración y aplicación

Expresión oral y escrita

Textualización
Revisión y
Planificación presentación
Coherencia Cohesión Adecuación

En el mapa de procesos, elaborado para la expresión oral y escrita, se contemplan tres


grandes procesos: la planificación textual, la textualización o elaboración del texto y la revi-
sión y presentación. El segundo de ellos ha sido matizado en tres aspectos diferentes cuya
denominación coincide con las características básicas de cualquier texto. Esta matización
responde al hecho de que permite diferenciar tareas o actividades que, en la práctica, se
realizan en bloque y sin discriminación consciente. Estos tres matices se refieren a la com-
posición de un texto coherente, bien cohesionado y adecuado. Se ha considerado adecua-
do explicitar estos matices por la especial incidencia de cada uno en aspectos distintos de
la elaboración de textos. Así, la coherencia está relacionada con la progresión temática del
texto (introducción de información nueva apoyándose en la dada), la cohesión con el uso
de los elementos de conexión en un texto y los signos de puntuación, y la adecuación con
la elección del vocabulario y registro adecuado. La misma razón ha hecho preferible man-
tener separados los procesos de coherencia y cohesión que, en realidad, son dos caras de
una misma cosa que es la unidad del texto. De modo que el proceso que se evalúa es
uno pero incluye contenidos referidos a esos tres aspectos de la textualidad y permite
discriminarlos.

Lo que permite identificar con claridad qué es lo que se va a evaluar, que no es la simple
distinción de los procesos que hasta ahora se han mencionado, sino su descripción. Es pre-
ciso, por tanto, explicar en qué consiste cada uno de ellos. Se han elaborado, por tanto,
definiciones tanto para los procesos de comprensión como para los de expresión.

Anexo III: Competencia en Comunicación Lingüística 77


Evaluación General de Diagnóstico 2009
Marco de la evaluación

Los procesos vinculados con las destrezas de comprensión se definen del siguiente modo:

Denominación Descripción

Extraer datos, hechos o informaciones básicas de un texto y relacionarlos con términos y conceptos
propios de un determinado campo del conocimiento o la experiencia que pueda identificar o recordar.
Es un proceso que requiere capacidad para percibir, descodificar, realizar operaciones básicas de
Aproximación e inferencia sobre datos explícitos y llevar a cabo una exploración rápida e inicial. De este modo el
identificación alumnado podrá acceder a una primera comprensión semántica y cognitiva del texto y, activar, en
consecuencia, el marco general de referencia, la situación o el contexto en que debe interpretarlo y
entenderlo.

Organizar la información obtenida reconociendo e identificando partes y relaciones entre ellas que les
Organización permitan elaborar esquemas, agrupar la información en bloques funcional o semánticamente afines,
reconocer un orden y vincular la información con campos bien acotados y definidos de conocimiento.

Sintetizar la información extraída mediante la comparación o el contraste de la información obtenida


del texto y de las operaciones anteriores, el examen de las relaciones identificadas y la eliminación de
Integración y la información no relevante. Esta labor de integración permitirá ordenar información dispersa en el
síntesis texto en secuencias bien conectadas para captar la intención y el sentido global del texto y, en fin,
reelaborar la información mediante formas como el resumen, mapas conceptuales o esquemas.

Se trata de un proceso de reflexión crítica para hacer valoraciones sobre las cualidades del texto en lo
referido a la calidad, relevancia, utilidad, eficacia y eficiencia de la información que permite obtener.
En el transcurso del proceso es preciso aportar nuevos datos y realizar inferencias complejas en las
Reflexión y
que entran en juego tanto las informaciones aportadas por el texto como los conocimientos y las
valoración experiencias previos del estudiante. Tratándose también de un proceso de valoración aportará datos o
ideas que tiendan bien a la convergencia o bien a la divergencia, tanto parciales como totales, con
respecto a la información que aporta al texto original y el modo en que lo hace.

Si la definición del proceso anterior tiene que ver con la capacidad de manejar, reordenar y valorar la
información que proporciona un texto vinculando los aspectos formales y de contenido, el proceso
de transferencia y aplicación consiste en adaptar, aplicar, diseñar, inventar, recrear o relacionar la
información de modo diferente para generar nuevos patrones, proponer soluciones alternativas o
avanzar incrementando cualitativamente la información que proporciona el texto. El proceso supone,
Transferencia y por un lado, que el alumnado ha asimilado la información que le proporciona el texto, por otro, que
aplicación es capaz de manejarla de modo autónomo y, por último, que puede transformarla y aplicarla a
situaciones nuevas o diversas. No se trata tan solo de manejar la información que proporciona el texto
sino también de utilizarla para aplicarla a situaciones distintas, es decir, de aprehender el sentido del
texto y extenderlo a realidades distintas de aquellas que lo motivan. En definitiva, se trata de un
proceso de adquisición y aplicación de conocimiento a partir de una fuente textual concreta y, en ese
sentido, la información que proporciona el texto de partida ha de verse globalmente incrementada.

78 Anexo III: Competencia en Comunicación Lingüística


Evaluación General de Diagnóstico 2009
Marco de la evaluación

La definición de los procesos relacionados con las destrezas de expresión es la que sigue:

Denominación Descripción

Consiste básicamente en las operaciones que se realizan para, a partir de las indicaciones recibidas,
centrar el tema del texto que se va a elaborar y seleccionar las ideas que se van a tratar en relación
Planificación con él dependiendo de la finalidad y la situación en que se emita el mensaje. Las tareas de
planificación son las que, inicialmente, se pueden relacionar en un guión.
Coherencia

Se trata de los procesos implicados en la confección de un texto unitario y, a su vez, organizado


en partes, además de la inclusión de toda la información necesaria para que resulte inteligible. En
definitiva, alude la progresión temática de las ideas que aparecen en el texto.
Cohesión

En este caso el proceso tiene que ver con el uso de los mecanismos de cohesión léxica y gramatical
Textualización

adecuados en cada parte del texto (concordancia y corrección verbal, concordancia de género, uso
de nexos adecuados), así como a la correcta puntuación del texto.
Adecuación

Tiene que ver con el uso de un vocabulario adecuado a la situación a la que se destina el texto o la
utilización de los recursos expresivos y el registro comunicativo que sean necesarios.

Este proceso se refiere, por una parte, a todas las habilidades relativas a los aspectos que se pueden
considerar como más básicos en la confección de textos como son la legibilidad de la caligrafía, la
Revisión limpieza del texto en cuanto a la presentación de márgenes o la existencia de tachones y la corrección
y presentación ortográfica. Por otro lado, tiene que ver también con destrezas relativas a la capacidad para ajustar el
contenido del texto elaborado a un contexto preciso en cuanto a la precisión léxica, la combinatoria
semántica, la corrección gramatical y el mantenimiento constante del registro adecuado.

2.c. Bloques de contenidos

Si bien los procesos descritos anteriormente remiten a un conjunto de habilidades, destre-


zas y conocimientos que integran la competencia en comunicación lingüística, tales pro-
cesos han de versar sobre un determinado conjunto de contenidos de referencia básica
para todos los evaluados. Esa referencia es la que proporcionan los Reales Decretos de
Mínimos para la Educación Primaria y la Educación Secundaria Obligatoria.

Tomándolos como punto de partida, es posible identificar varios bloques de contenidos en


torno a los cuales se han de centrar las unidades de evaluación. Para la evaluación de los
procesos de comprensión, esos bloques de contenidos son los siguientes:

Bloques de contenidos

Relaciones semánticas

Conocimientos de gramática

Enunciados y textos

Elementos contextuales

Léxico o vocabulario

Anexo III: Competencia en Comunicación Lingüística 79


Evaluación General de Diagnóstico 2009
Marco de la evaluación

En la expresión escrita el cuadro de contenidos abarcaría lo siguiente:

Bloques de contenidos

Léxico o vocabulario

Conocimientos de gamática

Enunciados y textos

Caligrafía y ortografía

2.d. Actitudes

En la descripción que el RD de Mínimos establece de la competencia en comunicación lin-


güística, aparecen varias indicaciones explícitas acerca de las actitudes que se han de des-
arrollar a la par que las habilidades y la adquisición de nuevos conocimientos. Estas indi-
caciones son las siguientes:

• El lenguaje es, entre otras cosas, instrumento para la organización y autorre-


gulación del pensamiento, las emociones y la conducta.
• Permite formarse un juicio crítico y ético.
• Permite el desarrollo de la autoestima y la confianza en uno mismo.
• Permite acercarse a nuevas culturas, tenerlas en consideración y respetarlas.
• Es instrumento para la igualdad efectiva mediante la eliminación de estereoti-
pos y expresiones sexistas.
• Permite la resolución pacífica de conflictos en la comunidad.
• Habilidades para representarse mentalmente.
• Habilidades para comprender e interpretar la realidad.
• Habilidades para organizar y autorregular el conocimiento y la acción dotándo-
los de coherencia.
• Capacidad empática de ponerse en lugar de otras personas.
• Tener en cuenta opiniones distintas a la propia con sensibilidad y espíritu crítico.
• Aceptar y realizar críticas con espíritu constructivo.

Enumeradas de este modo, no son más que simples indicaciones acerca del tipo de acti-
tudes que los estudiantes han de llegar a mantener ante distintas situaciones. Sin embar-
go, este conjunto de indicaciones es susceptible de un mayor grado de organización de
tal modo que pueden clasificarse todas ellas atendiendo a un conjunto más limitado de

80 Anexo III: Competencia en Comunicación Lingüística


Evaluación General de Diagnóstico 2009
Marco de la evaluación

categorías básicas. De entre todas esas indicaciones puede identificarse un grupo que tiene
que ver con el desarrollo del conocimiento de uno mismo y la autonomía personal. Otro
de los grupos de indicaciones se relaciona con el desarrollo de capacidades para la convi-
vencia en sociedad. Y un tercer grupo está vinculado con la capacidad de comprender la
realidad con autonomía y sentido crítico. Es evidente que ninguno de estos tres grupos es
independiente de los otros dos y que existen entre ellos claras relaciones de interdepen-
dencia. Sin embargo, se puede percibir también que cada uno de ellos apunta en una
dirección distinta partiendo de un mismo núcleo o componente afectivo o emocional del
proceso de aprendizaje. Estas tres direcciones pueden esquematizarse y describirse, utili-
zando las anteriores indicaciones, del siguiente modo.

Autonomía Socialización Ética

Actitudes que tienen que ver con el Son actitudes vinculadas a la capacidad Se trata de actitudes vinculadas con el
uso del lenguaje para representarse empática de situarse en lugar de otras desarrollo de la capacidad de aceptar
mentalmente con el fin de organizar y personas, lo que permite, por un lado, opiniones o ideas distintas a las
regular tanto el propio pensamiento conocer y respetar otras culturas, por propias y adquirir un sentido crítico y
como las acciones y las propias otro, eliminar tópicos o estereotipos y, constructivo que permita comprender,
emociones. Se trata de actitudes por último, resolver pacíficamente los interpretar y valorar adecuadamente la
situadas en el ámbito de desarrollo de conflictos mediante el uso del diálogo. realidad.
la autoestima y la confianza en uno
mismo.

3. Cuadros de relaciones

Como ya se ha indicado, en conjunto de contenidos que dan cuerpo al proceso de evalua-


ción proceden de los Reales Decretos de Mínimos. En concreto se han extraído básicamen-
te de los criterios de evaluación para 4º de Educación Primaria y de 2º de Educación
Secundaria Obligatoria y se han matizado tomando como apoyo los bloques de conteni-
dos mínimos de los Reales Decretos. El conjunto de contenidos obtenidos procede de todas
las materias que contribuyen al desarrollo de la competencia en comunicación lingüística.
Son, pues, diversos y también de diversa procedencia. Sin embargo, es posible realizar con
ellos una agrupación temática que facilite el trazado de un marco de evaluación compren-
sible. Los cuadros de relaciones que se presentan a continuación suponen un cruce de esos
bloques de contenidos con los procesos identificados y definidos en el apartado 2.c. Las
tablas en que se representan estas relaciones se organizan del siguiente modo:

• Cada columna se refiere a uno de los procesos que se han definido con
anterioridad.
• Cada fila se refiere a uno de los bloques de contenido identificados.
• En las celdas se han ido colocando los contenidos procedentes de
los criterios de evaluación de los Reales Decretos de Mínimos.

Anexo III: Competencia en Comunicación Lingüística 81


Evaluación General de Diagnóstico 2009
Marco de la evaluación

Una vez sintetizados y ordenados los contenidos, se han distribuido en las siguientes
tablas6:

• Comprensión lectora en castellano 4º EP.


• Comprensión lectora en castellano 2º de ESO.
• Comprensión lectora en lengua extranjera 4º EP.
• Comprensión lectora en lengua extranjera 2º ESO.
• Comprensión oral en castellano 4º EP.
• Comprensión oral en castellano 2º ESO.
• Comprensión oral en lengua extranjera 4º EP.
• Comprensión oral en lengua extranjera 2º ESO.
• Expresión escrita en castellano 4º EP.
• Expresión escrita en castellano 2º ESO.
• Expresión escrita en lengua extranjera 4º EP.
• Expresión escrita en lengua extranjera 2º ESO.

6 En las tablas que se presentan en el texto se cruzan, como se explica, los bloques de contenidos con los procesos
identificados. En el anexo I se presentan tablas similares que, en lugar de los bloques de contenidos, indican la procedencia
de los distintos contenidos.

82 Anexo III: Competencia en Comunicación Lingüística


Evaluación General de Diagnóstico 2009
Marco de la evaluación

3.a. Cuadros en los que se relacionan, a través de los criterios de evaluación, los procesos
y los bloques de contenidos

Cuadro de relaciones de Educación Primaria

Competencia en comunicación lingüística / Comprensión lectora / Lengua castellana / 4º EP

Aproximación Integración y Reflexión y Transferencia y


Procesos e identificación
Organización
síntesis valoración aplicación

• Localizar, detectar • Relacionar efectos • Sintetizar los • Reflexionar sobre


semánticas
Relaciones

y recuperar con causas. Manejo resultados de la el contenido y


información de lasnociones de obtención de evaluarlo.
explícita. duración, sucesión información.
y simultaneidad:
presente-pasado-
Conocimientos

futuro, anterior-
de gramática

posterior, duración y • Considerar y


simultaneidad (antes evaluar la
de, después de, al estructura y el uso
mismo tiempo que, del lenguaje.
mientras...).

• Entender • Comprender • En textos • Integrar los


información o ideas argumentaciones. literarios: reconocer resultados de la
relevantes explícitas Realizar inferencias el conflicto en un obtención de
en los textos. directas en la cuento; comprender información en
• En textos lectura de textos las relaciones entre textos de uso
literarios: ideas (acontecimientos personajes cuando escolar y social
principales de predecibles, deducir no aparecen con las propias
poemas cuando el propósito de los explícitas; ideas.
textos o identificar anticipación de
Enunciado y texto

están indicadas • Síntesis de ideas


expresamente, generalizaciones). acontecimientos, propias y ajenas.
reconocer el • En textos comprender el
conflicto en un literarios: interpretar sentido global.
cuento. convenciones • Observar y
específicas como reconocer los
Contenidos

• Interpretar los
temas recurrentes, efectos que la indicadores
elementos del inserción, la textuales y
relato, rima. supresión, el contextuales
cambio de orden, la para formular y
segmentación y la probar conjeturas.
recomposición
producen en los • Considerar el
enunciados y en texto de manera
los textos. crítica.

• Obtener • Interpretar • Interpretación y


información de determinados valoración de la
fuentes (textuales y aspectos no información que
visuales) y de la estrictamente proporcionan las
Elementos contextuales

observación de la textuales que imágenes en el


realidad. ayudan a la contexto social.
identificación de
ideas principales:
tipografía en
titulares o
entradillas,
en portadas;
subrayados, negritas
en epígrafes y otros
lugares destacados
de los textos.

• Entender el
vocabulario
Léxico o

vocabulario
apropiado y la
terminología propia
de las distintas
materias.

Anexo III: Competencia en Comunicación Lingüística 83


Evaluación General de Diagnóstico 2009
Marco de la evaluación

Cuadro de relaciones de Educación Primaria (continuación)

Competencia en comunicación lingüística / Comprensión oral / Lengua castellana / 4º EP


Aproximación Integración y Reflexión y Transferencia y
Procesos e identificación
Organización
síntesis valoración aplicación
de gramática semánticas
Conocimientos Relaciones

• Identificar •Manejo de las


información nociones de
específica. duración, sucesión y
simultaneidad:
presente-pasado-
futuro,anterior-
posterior, duración y
simultaneidad (antes
de, después de, al
mismo tiempo que,
mientras...).
Enunciado y texto

• Obtener, • Reconocer las ideas • Captar el sentido


seleccionar y principales y de textos orales de
Contenidos

relacionar secundarias. uso habitual.


información • Comprender • Captar el sentido
relevante. argumentaciones. global.
Relacionar efectos
con causas.

• Obtener • Integrar o • Interpretación y


información sintetizar los valoración de la
contextuales
Elementos

histórica y social de resultados de la información que pro-


fuentes visuales, de la información porcionan las
observación de la obtenida a partir de imágenes en el con-
realidad. fuentes visuales, texto social.
observación de
realidades.
vocabulario

• Comprender el
Léxico o

vocabulario
apropiado.

Competencia en comunicación lingüística / Expresión escrita / Lengua castellana / 4º EP


Textualización Revisión y
Procesos Planificación presentación
Coherencia Cohesión Adecuación
• Manejo de las • Explorar recursos
Léxico o vocabulario

nociones de expresivos y
duración, sucesión y creativos simples.
simultaneidad:
• Utilizar la
presente-pasado-
terminología
futuro, anterior-
lingüística y
posterior, duración y
gramatical propia de
simultaneidad (antes
la composición de
de, después de, al
textos.
mismo tiempo que,
Conocimientos

mientras...).
de gramática

• Uso correcto de la • Uso adecuado de


concordancia. las normas
Uso correcto de gramaticales.
Contenidos

conectores básicos.

• Planificar procesos • Elaborar • Claridad en la


Enunciado y texto

sencillos de argumentaciones. exposición.


construcción de Elaborar textos
objetos. Planificación estructurados.
de textos.
• Coherencia de los
• Elaborar guiones razonamientos.
para ordenar Secuenciación
información. ordenada de las ideas.

• Uso correcto de la • Cuidado de los


puntuación (punto, aspectos formales de
Caligrafía y
ortografía

coma, signos de los textos:


interrogación y presentación clara,
exclamación...). limpia, ordenada.
• Uso adecuado de las
normas ortográficas.

84 Anexo III: Competencia en Comunicación Lingüística


Evaluación General de Diagnóstico 2009
Marco de la evaluación

Cuadro de relaciones de Educación Primaria (continuación)

Competencia en comunicación lingüística / Comprensión lectora / Lengua extranjera / 4º EP

Aproximación Integración y Reflexión y Transferencia y


Procesos e identificación
Organización
síntesis valoración aplicación
• Leer, captar y
semánticas
Relaciones

extraer algunas
informaciones
específicas de textos
sencillos.
Conocimientos
de gramática
Enunciado
Contenidos

• Leer, captar y
y texto

extraer el sentido
global de textos
sencillos.
contextuales

• Identificar
Elementos

diferencias y
similitudes sobre
aspectos de la vida
cotidiana.

• Reconocer y
extraer palabras y
expresiones conocidas
vocabulario
Léxico o

que aparecen en
expresiones más
extensas aunque el
texto no se
comprenda en su
totalidad.

Competencia en comunicación lingüística / Comprensión oral / Lengua extranjera / 4º EP

Aproximación Integración y Reflexión y Transferencia y


Procesos e identificación
Organización
síntesis valoración aplicación
• Identificar
semánticas
Relaciones

información
específica.
Conocimientos
de gramática
Contenidos

Enunciado

• Captar el sentido
y texto

global en textos
sencillos.
contextuales
Elementos

• Reconocer y
extraer palabras y
vocabulario
Léxico o

expresiones
conocidas.

Anexo III: Competencia en Comunicación Lingüística 85


Evaluación General de Diagnóstico 2009
Marco de la evaluación

Cuadro de relaciones de Educación Primaria (continuación)

Competencia en comunicación lingüística / Expresión escrita / Lengua extranjera / 4º EP


Textualización Revisión y
Procesos Planificación presentación
Coherencia Cohesión Adecuación
• Uso del
Léxico o vocabulario

vocabulario
adecuado.
Conocimientos
de gramática

• Uso adecuado de
formas y estructuras
básicas.
Contenidos

• Composición de • Redacción de • Uso de estrategias


Enunciado y texto

textos a partir de textos cortos a partir básicas (destinatario,


modelos. de modelos con propósito, borrador,
formato establecido. revisión del texto,
etc.).
• Interés por el
cuidado y la
presentación.

• Corrección
Caligrafía y

ortográfica.
ortografía

• Conocimiento de
las relaciones entre
sonido y grafía.

86 Anexo III: Competencia en Comunicación Lingüística


Evaluación General de Diagnóstico 2009
Marco de la evaluación

Cuadro de relaciones de Educación Secundaria Obligatoria

Competencia en comunicación lingüística / Comprensión lectora / Lengua castellana / 2º ESO

Aproximación Integración y Reflexión y Transferencia y


Procesos e identificación
Organización
síntesis valoración aplicación

• Obtener y • Organizar la • Elaborar inferencias


seleccionar ideas, información sobre datos
Relaciones semánticas

hechos o datos obtenida en un explícitos.


relevantes y no esquema o guión.
• Reconocer e
relevantes. • Mecanismos de interpretar el sentido
• Extraer referencia interna de recursos retóricos
informaciones (pronombres, elipsis, y expresivos.
concretas hiperónimos de
localizadas en varios significado concreto).
párrafos del texto. • Identificar secuencias
de cualidades
(descripciones).

• Formas de deixis • Interpretar las


espacial, temporal y formas verbales y
personal. algunos papeles
Conocimientos
de gramática

• Conectores temáticos de sus


textuales de orden, complementos
explicativos y de (paciente, agente,
contraste. causa...).

• Formas de inicio,
desarrollo cronológico,
desenlaces.

• Reconocer los • Distinguir cómo • Identificar • Extraer • Relacionar el


enunciados en los está organizada la significados conclusiones. contenido con la
que aparece el tema información. contextuales de • Valoración crítica propia experiencia.
general y los • Secuencias enunciados. de mensajes • Formarse una
secundarios. temporales de • Comprender y explícitos e opinión propia a
Contenidos

• Identificar los simultaneidad y resumir implícitos. partir de la


elementos de una evolución. argumentaciones, • Diferenciar el información
historia (personajes, • Secuencias de causa descripciones, contenido literal del obtenida.
tiempo, lugares...), y consecuencia. narraciones, sentido.
un texto teatral o un exposiciones.
poema. • Identifican • Valorar el contenido,
• Seguir instrucciones.
Enunciado y texto

secuencias de hechos la estructura, el uso


en narraciones sean • Realizar síntesis de del lenguaje, los
de desarrollo lineal informaciones. elementos del texto.
o no. • Comprensión • Valoración crítica
global de textos. de secuencias
Identificar el tema discriminatorias,
general y temas tópicos, temas o
secundarios. motivos
• Reconocer el característicos
propósito o la de los géneros
intención de un texto literarios y
aunque no sea divulgativos.
explícito.
• Realizar un resumen.
• Comparar,
contrastar, ampliar y
reducir informaciones
y enunciados.
vocabulario contextuales
Elementos

• Usos coloquiales y • Lectura e


formales. interpretación de
información en tablas
o gráficos.
Léxico o

• Comprender
vocabulario y
terminología
específica.

Anexo III: Competencia en Comunicación Lingüística 87


Evaluación General de Diagnóstico 2009
Marco de la evaluación

Cuadro de relaciones de Educación Secundaria Obligatoria (continuación)

Competencia en comunicación lingüística / Comprensión oral / Lengua castellana / 2º ESO

Aproximación Integración y Reflexión y Transferencia y


Procesos e identificación
Organización
síntesis valoración aplicación

• Seleccionar • Integrar información • Elaborar inferencias


información pertinente en un a partir de los datos
pertinente. esquema, guión, obtenidos.
semánticas
Relaciones

gráfico, tabla.
• Reconocer ideas,
hechos o datos sean • Identificación y
o no relevantes en comprensión de
el texto. mecanismos de
referencia interna en
el texto.

• Identificación de
conectores textuales
de orden, explicativos
y de contraste.
• Identificación y
comprensión de
formas de deixis
Conocimientos
de gramática

personal, espacial y
temporal.
• Identificación y
comprensión de
recursos que expresan
simultaneidad y
evolución.
• Identificación y
comprensión de
recursos que expresan
Contenidos

causa y consecuencia.

• Comprender • Identificar • Valoración crítica • Formarse una


argumentaciones, significados de los mensajes opinión propia.
narraciones, contextuales de los recibidos.
descripciones, enunciados. • Análisis
explicaciones. • Sintetizar comparativo y
• Seguir instrucciones información. valoración crítica de
Enunciado y texto

para secuencias de Comprender informaciones.


más de tres mensajes de distintos • Valorar la
operaciones. ámbitos sociales y del relevancia de la
conocimiento. información
recibida.
• Captar la idea
global del mensaje.
• Reconocer el
propósito del texto.
• Ser capaz de retener
las ideas esenciales y
resumir el texto.

• Apreciar las
contextuales
Elementos

diferencias
contextuales y
formales entre lo oral
y lo escrito, lo
coloquial y lo formal.

• Identificar y
vocabulario

comprender
Léxico o

terminología
específica básica de
las distintas áreas de
conocimiento.

88 Anexo III: Competencia en Comunicación Lingüística


Evaluación General de Diagnóstico 2009
Marco de la evaluación

Cuadro de relaciones de Educación Secundaria Obligatoria (continuación)

Competencia en comunicación lingüística / Expresión escrita / Lengua castellana / 2º ESO


Textualización Revisión y
Procesos Planificación presentación
Coherencia Cohesión Adecuación
vocabulario

• Expresión verbal de • Precisión en el uso


Léxico o

las relaciones. de términos.

• Encadenamiento • Formas de deixis • Uso de las normas


adecuado de las personal, temporal y gramaticales.
ideas. espacial.
• Uso de conectores
de orden, explicativos
y de contraste.
Conocimientos de gramática

• Uso de mecanismos
de referencia interna
(pronominalización,
elipsis,
hiperónimos ...).
• Recursos para la
argumentación.
• Recursos para
expresar
simultaneidad y
evolución.
• Recursos para
expresar causa y
consecuencia.

• Valorar la • Elaborar informes • Elaborar trabajos


importancia de y documentos que a partir de
planificar el texto. integren información informaciones
Contenidos

textual, visual y procedentes de otras


• Realizar versiones gráfica.
hasta alcanzar una fuentes.
• Explicar una opinión
definitiva adecuada a • Elaborar
basada en datos.
los propósitos conclusiones a partir
comunicativos. • Explicar fenómenos de datos obtenidos.
naturales.
• Presentar • Comunicar los
informaciones • Utilizar diferentes resultados de un
variantes del discurso:
previamente problema o un
descripción,
preparadas. narración, estudio.
Enunciado y texto

• Formular las argumentación, • Uso del registro


preguntas necesarias textos explicativos. adecuado.
para elaborar un • Exposición de • Composición de
estudio. opiniones y juicios textos con un
• Planificar propios con propósito.
pequeños argumentos
razonados.
trabajos.
• Seleccionar
información,
organizarla y
presentar
conclusiones.
• Elaboración de
esquemas, síntesis y
resúmenes.
• Elaborar textos
propios de ámbitos
personales y
escolares.

• Uso de las normas


Caligrafía y

ortográficas.
ortografía

• Uso de las normas


tipográficas.
• Uso de los signos de
puntuación.

Anexo III: Competencia en Comunicación Lingüística 89


Evaluación General de Diagnóstico 2009
Marco de la evaluación

Cuadro de relaciones de Educación Secundaria Obligatoria (continuación)

Competencia en comunicación lingüística / Comprensión lectora / Lengua extranjera / 2º ESO

Aproximación Integración y Reflexión y Transferencia y


Procesos e identificación
Organización
síntesis valoración aplicación
• Identificación de • Elaborar inferencias
semánticas
Relaciones

palabras clave. a partir de los datos


• Obtienen informa- obtenidos.
ción específica. • Contrastar
• Identificación de información.
detalles específicos.
Conocimientos

• Identificación de
de gramática

los principales ele-


mentos morfológicos.
• Conectores
textuales de orden.
Contenidos

• Identificar • Identifican el • Valorar el


Enunciado

secuencias de tema de un texto. contenido, la


y texto

hechos. • Comprensión de la estructura, el uso


idea general del del lenguaje y los
texto. elementos del texto.
contextuales

• Inferencia de
Elementos

significados por el
contexto.

• Reconocen
vocabulario
Léxico o

vocabulario y de
expresiones
habituales.

Competencia en comunicación lingüística / Comprensión oral / Lengua extranjera / 2º ESO

Aproximación Integración y Reflexión y Transferencia y


Procesos e identificación
Organización
síntesis valoración aplicación
• Obtención de • Inferir significados
semánticas
Relaciones

información general y por comparación de


específica. palabras o frases
similares en lenguas
• Identificar palabras
conocidas.
clave.
Enunciado Conocimientos
de gramática

• Comprender la
Contenidos

y texto

idea general.

• Uso del contexto


verbal y no verbal.
• Uso de los
contextuales

conocimientos
Elementos

previos.
• Inferir significados
por el contexto.
• Inferir significados
por elementos
visuales.
vocabulario
Léxico o

90 Anexo III: Competencia en Comunicación Lingüística


Evaluación General de Diagnóstico 2009
Marco de la evaluación

Cuadro de relaciones de Educación Secundaria Obligatoria (continuación)

Competencia en comunicación lingüística / Expresión escrita / Lengua extranjera / 2º ESO


Textualización Revisión y
Procesos Planificación presentación
Coherencia Cohesión Adecuación
• Ampliación de
vocabulario

expresiones
Léxico o

comunes, frases
hechas, léxico
apropiado al
contexto.
Conocimientos
de gramática

• Elementos básicos
de cohesión.
Contenidos

• Uso de conectores
sencillos.

• Componer textos a • Composición de • Fórmulas que


Enunciado
y texto

partir de modelos. textos con diferencian el


estructuras lenguaje formal del
sencillas. no formal.

• Reglas básicas de
Caligrafía y
ortografía

ortografía y
puntuación.
• Interés por la
presentación cuidada.

3.b. Tabla resumen de las actitudes y su relación con los criterios de evaluación

Tomando en consideración los tres ejes actitudinales descritos en el apartado 1.3 es posi-
ble identificar en los RR.DD de Mínimos, conjuntos de indicaciones relativas a las actitudes
vinculadas a la competencia en comunicación lingüística. Esas indicaciones se encuentran,
por un lado, en el desarrollo de los contenidos y, por otro, en la expresión de los criterios
de evaluación de las distintas materias. Tanto en un caso como en otro la referencia a las
actitudes responde a dos situaciones básicas:

a) En ocasiones se trata de referencias aisladas en las que se enuncian contenidos


actitudinales no vinculados en concreto a ningún otro tipo de contenido. Este
tipo de indicaciones, que proceden de todas las materias en cuanto que contri-
buyen al desarrollo de la competencia en comunicación lingüística, se recogen,
tanto para Educación Primaria como para Educación Secundaria Obligatoria, en
las tablas que aparecen en el anexo II de este documento.

b) En otros casos se trata de referencias a actitudes integradas en la descripción de


otros contenidos o asociadas a ellos de distintas formas. Estos contenidos están
recogidos en las tablas que presentan seleccionados los contenidos actitudinales
vinculados con los procesos de comprensión y de expresión.

Anexo III: Competencia en Comunicación Lingüística 91


Evaluación General de Diagnóstico 2009
Marco de la evaluación

Las referencias a actitudes del tipo a) permiten elaborar una relación de todos los conteni-
dos actitudinales recogidos en los RR.DD de Mínimos. Las del tipo b), al estar asociadas o
integradas con otros contenidos, permiten establecer una pauta a partir de la cual vincu-
lar todos los contenidos actitudinales de tipo a) con las destrezas de comprensión y expre-
sión. Además, como la descripción de la competencia en comunicación lingüística señala
tres tipos o direcciones en el desarrollo de las actitudes, es posible elaborar tablas que
sinteticen y ordenen el conjunto de actitudes que han de formar parte del proceso de
evaluación.

Debe señalarse, con respecto a esta organización de los contenidos actitudinales, lo


siguiente:

• No es posible asignar referencias a actitudes para cada uno de los procesos que
se han descrito en las distintas destrezas. Muchas de las actitudes recorren varios
procesos y, en la mayoría de los casos, se trata de actitudes asociadas al desarro-
llo de comportamientos comunicativos.

• No parece posible tampoco asignar de modo específico los distintos contenidos


actitudinales a las destrezas de comprensión y expresión. Algunas de las actitu-
des relacionadas con las destrezas de comprensión podrían ser también detecta-
bles a través de conductas expresivas y viceversa.

• Los contenidos actitudinales que se han relacionado en las tablas anteriores no


se refieren a actitudes hacia la comunicación sino a actitudes que se manifiestan
en las conductas comunicativas.

92 Anexo III: Competencia en Comunicación Lingüística


Evaluación General de Diagnóstico 2009
Marco de la evaluación

Las tablas a las que nos hemos referido son las siguientes:

Tabla de actitudes en Comprensión

Competencia en comunicación lingüistica / Comprensión

Aceptación de los errores como instrumento de mejora.


Autonomía

Perseverancia en la búsqueda de soluciones.


Autonomía en la realización de tareas de aprendizaje.
Confianza en la propia capacidad para enfrentarse con éxito a situaciones inciertas.
Conocimiento de uno mismo, actitud analítica con respecto a las propias acciones y emociones.

Actuar de forma coordinada y cooperativa.


Socialización

Actitud de aceptación hacia las demás personas.


Interés por la intervención de las demás personas.
Ejes actitudinales

Asunción responsable de las tareas.


Actitudes de respeto, tolerancia e interés por los demás.
Favorecer la convivencia en el aula y la participación en las actividades

Asunción de las propias responsabilidades.


Respeto a las normas de interacción.
Evitar discriminaciones y actitudes de rivalidad fundadas en estereotipos y prejuicios.
Actitud de rechazo hacia las discriminaciones.
Apreciar las particularidades socioculturales de hablantes de otras lenguas.
Ética

Respeto hacia los valores y comportamientos de otros pueblos superando estereotipos.


Capacidad para ponerse en lugar de otro.
Valoración crítica de mensajes explícitos e implícitos.
Actitud positiva para contrastar puntos de vista.
Capacidad para distanciarse de los mensajes, analizarlos y valorarlos críticamente.
Síntesis de ideas propias y ajenas.
Formarse una opinión propia.

Tabla de actitudes en Expresión

Competencia en comunicación lingüística / Expresión

Regulación de las propias emociones.


Autonomía

Regulación de la propia actividad con progresiva autonomía.


Trazar planes y emprender procesos de decisión.
Valoración crítica de la propia producción.
Comunicación de sentimientos, ideas y opiniones.
Ejes actitudinales

Comunicar a los demás juicios personales u opiniones con argumentos razonados.

Predisposición al diálogo.
Socialización

Uso social y colaborativo de la escritura.


Uso social y colaborativo de las distintas formas de comunicación.
Interés por manifestarse de modo claro cuidando la presentación de la información.
Actitud de participación en las distintas formas de interacción comunicativa.
Capacidad para resolver conflictos.

Valoración de la diversidad lingüística y cultural.


Ética

Crear sentimientos comunes que favorezcan la convivencia.


Respeto hacia los valores y comportamientos de otros pueblos superando estereotipos y prejuicios.
Actitud crítica y constructiva.

Anexo III: Competencia en Comunicación Lingüística 93


Evaluación General de Diagnóstico 2009
Marco de la evaluación

4. Matriz de especificaciones de la competencia

Matriz de especificaciones de procesos y conocimientos. Comprensión oral y lectora

Procesos
Bloques de Primaria Secundaria
contenidos Aproximación Integración y Reflexión y Transferencia y (%) (%)
Organización
e identificación síntesis valoración aplicación

Relaciones
semánticas
25+- 5 15+- 5

Conocimientos
25+- 5 25+- 5
de gramática

Enunciados y
15+- 5 25+- 5
textos

Elementos
contextuales 10+- 5 15+- 5

Léxico o
vocabulario
25+- 5 20+- 5

Primaria (%) 30 +-5 30 +-5 30 +-5 5+- 2 5+- 2 100

Secundaria (%) 15 +-5 20 +-5 30 +-5 30 +-5 5 +-2 100

Matriz de especificaciones de procesos y conocimientos. Expresión escrita

Procesos
Bloques de Primaria Secundaria
Textualización Revisión y
contenidos Planificación (%) (%)
Coherencia Cohesión Adecuación Presentación

Léxico o
30+- 5 25+- 5
vocabulario

Conocimientos
de gramática
20+- 5 30+- 5

Enunciado y
20+- 5 30+- 5
texto

Caligrafía y
ortografía
30+- 5 15+- 5

Primaria (%) 10 +-5 25 +-5 25 +-5 10 +-5 30+- 5 100

Secundaria (%) 15 +-5 25 +-5 25 +-5 15 +-5 20 +-5 100

94 Anexo III: Competencia en Comunicación Lingüística


Evaluación General de Diagnóstico 2009
Marco de la evaluación

En cuanto al peso de las actitudes, tomando en cuenta los tres ejes que se han descrito,
los porcentajes son los siguientes:

En Educación Primaria

Autonomía 30%

Socialización 50%

Ética 20%

En Educación Secundaria Obligatoria

Autonomía 40%

Socialización 30%

Ética 30%

Anexo III: Competencia en Comunicación Lingüística 95


Evaluación General de Diagnóstico 2009
Marco de la evaluación

5. Ejemplo de unidad de evaluación para Educación Primaria


(final del segundo ciclo)

MANZANAS HECHICERAS
Ingredientes para cuatro personas:
• 4 manzanas ácidas
• Leche condensada al gusto
• Mantequilla
• Azúcar
• Sirope de naranja
• Canela en polvo
• Raspadura de limón
• 1 limón
• Hojas de menta
• Guindas y uvas en almíbar

Preparación:

Pelar las manzanas y descorazonarlas, metiéndolas en agua con el jugo del limón
durante una hora.
Embadurnar un cuenco con mantequilla y espolvorear azúcar por encima.
Colocar las manzanas enteras en el cuenco y en el hueco de las mismas echar
leche condensada.
Regar las manzanas con un chorrito de sirope de naranja, y espolvorear por enci-
ma con la raspadura del limón y la canela en polvo.
Introducir el cuenco con las manzanas en el microondas a máxima potencia
durante diez minutos.
Servir las manzanas tibias, adornándolas con hojitas de menta y guindas y uvas
en almíbar.

96 Anexo III: Competencia en Comunicación Lingüística


Evaluación General de Diagnóstico 2009
Marco de la evaluación

1. ¿Cuál es el ingrediente principal? ¿Por qué?

2. ¿Qué utensilio de los siguientes NO se necesita para preparar la receta?

A Sartén.
B Cuenco.
C Exprimidor.
D Microondas.

3. ¿Cuál es el orden adecuado para preparar la receta?

a Servir las manzanas tibias.


b Pelar las manzanas y descorazonarlas.
c Colocar las manzanas enteras en el cuenco.
d Introducir el cuenco con las manzanas en el microondas.

A a-b-c-d
B b-c-d-a
C b-d-c-a
D a-d-c-b

4. ¿Qué significa descorazonar la manzana?

A Quitarle la piel.
B Lavar las manzanas.
C Entristecer a la manzana.
D Quitarle las semillas del centro.

Anexo III: Competencia en Comunicación Lingüística 97


Evaluación General de Diagnóstico 2009
Marco de la evaluación

5. De todos los ingredientes, ¿cuáles sirven solo para adornar?

A Hojas de menta y guindas.


B Leche condensada y azúcar.
C Mantequilla y sirope de naranja.
D Canela en polvo y raspadura de limón.

6. ¿Con qué adjetivo calificarías este plato?

A Ácido.
B Dulce.
C Salado.
D Amargo.

7. Considerando los tiempos que se indican en la preparación, ¿cuánto tardarás


aproximadamente en preparar esta receta?

A Una hora.
B Media hora.
C Una hora y media.
D Tres cuartos de hora.

8. El texto tiene dos partes, ¿qué nos dice en cada una de ellas?

1ª parte:

2ª parte:

98 Anexo III: Competencia en Comunicación Lingüística


Evaluación General de Diagnóstico 2009
Marco de la evaluación

9. ¿Qué quiere decir que se utiliza leche condensada al gusto?

A El que cocina decide según su criterio.


B Depende del resto de los ingredientes.
C La cantidad viene indicada en la elaboración.
D Hay que darse gusto con la leche condensada.

10. ¿Qué significa espolvorear en el texto?

A Hacer desaparecer alguna cosa.


B Esparcir por encima algo en polvo.
C Quitar las partículas que se pegan.
D Extender la masa en algún recipiente.

11. ¿Por qué crees que han puesto el título Manzanas hechiceras?

A Las manzanas así preparadas encantan, cautivan a todos.


B Es una receta que usaban los hechiceros en algunas tribus.
C Se pueden usar las manzanas cocinadas para hacer hechizos.
D La receta propuesta pertenece a un libro de recetas de medicamentos.

12. ¿Qué ingredientes aportarán sabor dulce al plato?

A Leche condesada, azúcar y limón.


B Leche condesada, menta y azúcar.
C Manzanas verdes ácidas, limón y mantequilla.
D Guindas y uvas en almíbar, leche condesada y azúcar.

Anexo III: Competencia en Comunicación Lingüística 99


Evaluación General de Diagnóstico 2009
Marco de la evaluación

13. Escribe la preparación de esta receta.

Ingredientes del batido de chocolate:

• Un litro de leche

• Un vaso de nata

• 4 cucharadas de cacao molido

• 300 gramos de helado de chocolate

• Una hoja de menta

• Canela

100 Anexo III: Competencia en Comunicación Lingüística


ANEXO IV

COMPETENCIA MATEMÁTICA

1. Presentación de la competencia matemática

La competencia matemática consiste en la habilidad para utilizar y relacionar los números,


sus operaciones básicas, los símbolos y las formas de expresión y razonamiento matemá-
tico, tanto para producir e interpretar distintos tipos de información, como para ampliar
el conocimiento sobre aspectos cuantitativos y espaciales de la realidad, y para resolver
problemas relacionados con la vida cotidiana y con el mundo laboral.

Esta competencia implica:

• El conocimiento y manejo de los elementos matemáticos básicos en situaciones


reales o simuladas de la vida cotidiana, y la puesta en práctica de procesos de
razonamiento que llevan a la solución de los problemas o a la obtención de infor-
mación.

• Una disposición favorable y de progresiva seguridad y confianza hacia la infor-


mación y las situaciones que contiene elementos o soportes matemáticos, así
como hacia su utilización cuando la situación lo aconseja, basadas en el respeto
y el gusto por la certeza y en su búsqueda a través del razonamiento.

La competencia contribuye a la posibilidad real de seguir aprendiendo a lo largo de la vida


pues forma parte de ella la habilidad para interpretar y expresar con claridad y precisión
informaciones, datos y argumentaciones. Cobra realidad y sentido en la medida en que los
elementos y razonamientos matemáticos son utilizados para enfrentarse a aquellas situa-
ciones cotidianas que los precisan; por tanto, la identificación de tales situaciones, la apli-
cación de estrategias de resolución de problemas, y la selección de las técnicas adecuadas
para calcular, representar e interpretar la realidad a partir de la información están incluidas
en ella.

Anexo IV: Competencia Matemática 101


Evaluación General de Diagnóstico 2009
Marco de la evaluación

2. Dimensiones de la competencia

2.a. Situaciones o contextos

Las situaciones o contextos se refieren a las situaciones problemáticas (cualquier escenario


cotidiano que exige dar una respuesta) de la vida diaria en las que el alumnado tiene que
hacer uso de su competencia matemática para afrontarla con éxito y aportar una respues-
ta consecuente.

Tal y como se recoge en el marco teórico de PISA 2003, un aspecto importante de la com-
petencia matemática lo constituye el involucrarse en las Matemáticas, es decir, ejercitarlas y
utilizarlas en una amplia variedad de situaciones.

Los ejercicios que se planteen en una evaluación de la competencia matemática han de loca-
lizarse en una parte del mundo de los estudiantes. Las situaciones más cercanas serán las rela-
tivas a su vida personal y las de su vida escolar para continuar con las relativas a la vida labo-
ral y el ocio y la vida en la comunidad local y la sociedad, tal y como se presentan en la vida
diaria. A mucha distancia de todas ellas están las situaciones de tipo científico.

Queda patente que la referencia a “la vida diaria del estudiante” incluye su vida privada,
familiar-laboral y social con sus compañeros y familiares, así como su vida como ciudadano
dentro de una comunidad.

Para las unidades de evaluación que se presenten para evaluar la competencia matemáti-
ca se definen y utilizan cuatro tipos de situaciones:

• Personal: situaciones relacionadas con el yo, la familia y los grupos de compañeros.

• Educacional/profesional: situaciones relacionadas con la vida escolar y laboral.

• Pública: situaciones de la comunidad local y la sociedad.

• Científica: situaciones que se refieren a estructuras, símbolos y objetos matemá-


ticos… es decir, relativas al universo matemático. Se trata de explicar escenarios
hipotéticos y explorar sistemas o situaciones potenciales apoyándose en esta
competencia.

El contexto de un “ítem enmarcado en un estímulo” lo constituye el modo concreto en


que ése se presente dentro de una situación. Engloba elementos específicos utilizados en
el enunciado del estímulo.

2.b. Procesos

En la evaluación de la competencia matemática se tienen en cuenta seis procesos de índo-


le cognitiva, que se enmarcan dentro de tres agrupamientos de destrezas, y éstas son:

102 Anexo IV: Competencia Matemática


Evaluación General de Diagnóstico 2009
Marco de la evaluación

• Destrezas de reproducción: Hacen referencia a la reproducción de los conoci-


mientos practicados, tales como el reconocimiento de tipos de procesos y pro-
blemas matemáticos familiares y la realización de operaciones habituales. Estas
destrezas son necesarias para la realización de los ejercicios más sencillos.

• Destrezas de conexión: Exigen que los alumnos/as vayan más allá de los pro-
blemas habituales, realicen interpretaciones y establezcan interrelaciones en
diversas situaciones, pero todavía en contextos relativamente conocidos. Estas
destrezas acostumbran a estar presentes en los problemas de dificultad.

• Destrezas de reflexión: Implican perspicacia y reflexión por parte del alumno/a, así
como creatividad a la hora de identificar los elementos matemáticos de un problema
y establecer interrelaciones.

Los seis niveles o gradaciones de procesos cognitivos que permiten el dominio de las des-
trezas mencionadas se presentan en la tabla siguiente y encajan, con su correspondiente
definición, en los tres grupos anteriores.

Destrezas Procesos Descripción Acciones


Comprensión identificación

Representa las acciones de recordar y reconocer los Nombrar, definir, encontrar, mostrar, imitar,
Acceso e

términos, los hechos, los conceptos elementales de deletrear, listar, contar, recordar, reconocer,
un ámbito de conocimiento y de reproducir localizar, reproducir, relatar.
Reproducción

fórmulas establecidas.

Supone acciones como captar el sentido y la Explicar, ilustrar, extractar, resumir, completar,
intencionalidad de textos, de lenguajes específicos traducir a otros términos, aplicar rutinas,
y códigos relacionales e interpretarlos para resolver seleccionar, escoger.
problemas.
Aplicación

Comporta aptitud para seleccionar, transferir y Clasificar, resolver problemas sencillos,


aplicar información para resolver problemas con construir, aplicar, escoger, realizar, resolver,
cierto grado de abstracción y la de intervenir con desarrollar, entrevistar, organizar, enlazar.
Conexión

acierto en situaciones nuevas.

Significa la posibilidad de examinar y fragmentar la Comparar, contrastar, demostrar,


valoración
Análisis y

información en partes, encontrar causas y motivos, experimentar, planear, resolver problemas


realizar inferencias y encontrar evidencias que complejos, Analizar, simplificar, relacionar,
apoyen generalizaciones. Se empareja con inferir, concluir.
compromiso.

Se corresponde con las acciones de compilar Combinar, diseñar, imaginar, inventar,


Síntesis y
creación

información y relacionarla de manera diferente, planificar, predecir, proponer, adaptar,


establecer nuevos patrones, descubrir soluciones estimar.
alternativas. Puede asociarse a la resolución de
Reflexión

conflictos.

Representa capacidades para formular juicios con Criticar, concluir, determinar, juzgar,
valoración
Juicio y

criterio propio, cuestionar tópicos y exponer y recomendar, establecer criterios y/o límites.
sustentar opiniones fundamentándolas. En otro
orden se asociaría a acciones de planificación
compleja, de reglamentación y de negociación.

Anexo IV: Competencia Matemática 103


Evaluación General de Diagnóstico 2009
Marco de la evaluación

Además, se señalan una serie de verbos que indican acciones asociadas a cada uno de los
procesos y enriquecen la descripción de los mismos. Tal asociación no es de carácter lineal
y determinados verbos o acciones se asociarán a procesos diferentes dependiendo de la
explicitación concreta que se haga a través de los criterios de evaluación.

2.c. Bloques de contenidos

Para organizar los fenómenos del mundo natural, social y mental han de utilizarse herramien-
tas tales como los conceptos, estructuras e ideas. El currículo de Matemáticas se ha organi-
zado de una manera lógica alrededor de diferentes líneas de contenido, que reflejan ramas
de conocimiento históricamente establecidas, si bien en el mundo real, los fenómenos sus-
ceptibles de un tratamiento matemático no aparecen organizados de un modo tan lógico,
dado que la competencia matemática abarca un campo mucho más amplio de conocimien-
tos, destrezas y actitudes que las incluidas históricamente en la disciplina.

No ha de perderse de vista que todas las áreas y materias del currículo contribuyen a la
adquisición de la competencia matemática, si bien no todas ellas en igual medida.

En cualquier caso, los contenidos han de presentarse de forma que describan su relación
con los fenómenos y los tipos de situaciones problemáticas. Son muchas las posibles orga-
nizaciones de carácter fenomenológico que se les pueden dar a los contenidos.

Puesto que el área de Matemáticas es la que contribuye en mayor medida a la adquisición


de la competencia, es importante seleccionar un conjunto de problemáticas surgidas de la
evolución histórica de las Matemáticas que englobe una variedad y profundidad suficien-
te de ellas que dejen ver los elementos esenciales de las mismas y a la vez que permitan la
contribución del resto de las áreas del currículo a la adquisición de esta competencia.

Se observa que los bloques de contenido establecidos en el currículo de enseñanzas míni-


mas del área de Matemáticas permiten conjugar los aspectos señalados anteriormente, es
decir, incluye la contribución de las restantes áreas curriculares, de ahí que se opte por su
consideración como bloques de contenidos para el marco de la evaluación general de
diagnóstico, quedando en este caso como siguen:

4º Educación Primaria 2º Educación Secundaria Obligatoria

Números y operaciones Contenidos comunes


La medida: estimación y cálculo de magnitudes Números
Geometría Álgebra
Tratamiento de la información, azar y probabilidad Geometría
Funciones y gráficas
Estadística y probabilidad

104 Anexo IV: Competencia Matemática


Evaluación General de Diagnóstico 2009
Marco de la evaluación

2.d. Actitudes

Desempeña un papel importante a la hora de determinar su interés, su atención y reaccio-


nes hacia los aspectos relacionados con la competencia matemática.

En la evaluación de esta competencia se incluirán algunas preguntas, especialmente liga-


das a procesos de juicio y regulación, que permitan la evaluación de las actitudes de los
estudiantes pero siendo éste un aspecto más de la evaluación y no se tratará de forma
específica.

Se ha de señalar que en el currículo de Educación Primaria, por ejemplo, el área de


Matemáticas es probablemente una de las que más contenidos actitudinales incluye, ya
que se presentan en contextos de resolución de problemas y se orientan en el sentido de:
interés por situaciones que requieran utilizar las Matemáticas, por la búsqueda de estrate-
gias, por la exploración de soluciones alternativas…

3. Cuadros de relaciones

Se presentan las tablas, tanto para 4º de Educación Primaria como para 2º de Educación
Secundaria Obligatoria (ESO), en que se presentan los bloques de contenidos, los
procesos y la relación con los criterios de evaluación que permiten la adquisición de la
competencia.

Con el término de competencia matemática se enfatiza el uso funcional del conocimiento


matemático en numerosas y diversas situaciones y, para hacer posible y viable el uso de la
competencia matemática, se requieren unos conocimientos y destrezas matemáticas básicas.

En el caso de la evaluación de la competencia matemática en el nivel de 2º ESO, y con la pre-


misa del énfasis en una evaluación que incida en el uso funcional de las Matemáticas, se hace
necesario tener en cuenta los criterios de evaluación del primer ciclo: es en un análisis de
carácter global dónde se pueden poner de manifiesto las destrezas que se hacen necesarias
para el desarrollo de la competencia matemática.

Anexo IV: Competencia Matemática 105


Evaluación General de Diagnóstico 2009
Marco de la evaluación

3.a. Cuadros en los que se relacionan, a través de los criterios de evaluación, los procesos
y los bloques de contenidos

Cuadro de relaciones de Educación Primaria

Reproducción Conexión Reflexión


Bloques
contenidos Acceso e Análisis y Síntesis y Juicio y
Compresión Aplicación
identificación valoración creación regulación
• Lng.: • Interpretar el • Utilizar en • Comparar y • Utilizar • Apreciar si
Localizar y valor posicional contextos ordenar estrategias llegan a
recuperar de cada una de cotidianos, la números por personales para resultados
información las cifras de un lectura y el valor la resolución de válidos,
explícita. número. escritura de posicional y problemas. exactos o
números en la recta estimados, en
• Utilizar el naturales hasta numérica. • Explicar las función de los
conocimiento 6 cifras. estrategias números que
del sistema de • En situaciones utilizadas en el intervienen y
numeración • Manejar, en de vida cálculo mental. de la situación
decimal (la situaciones cotidiana, de cálculo
estructura del reales, la interpretar y • Expresar de
en que se
sistema representación expresar forma escrita, y
produce.
decimal) y las de cantidades situaciones con ordenada el
propiedades de hasta 6 cantidades. proceso.
de las cifras, partiendo
• Mostrar • C. Medio:
operaciones. del concepto de
flexibilidad a la Comunicar de
valor de
• C. Medio: hora de forma escrita
posición.
Obtener elegir el los resultados
información • Dominar la procedimiento acompañados
relevante sobre organización de más adecuado. de tablas,
hechos o la serie escrita gráficos, etc.
Criterios de evaluación: 1, 2, 3 y 8

fenómenos de las cifras de • Resolver


• Lng.:
Números y operaciones

delimitados. un número y problemas


relacionados Interpretar e
situarlo en la
• Lng.: con el entorno integrar las
recta.
Captar el que exijan cierta ideas propias
sentido global • Realizar planificación, con la
y algunas cálculos aplicando dos información
informaciones numéricos con operaciones contenida en
específicas. números con números los textos
naturales y naturales como adecuados al
usarlos en máximo. nivel de edad.
situaciones de
• Lng.:
resolución de
Redactar,
problemas y
reescribir y
dominar los
resumir textos
algoritmos
en situaciones
escritos.
cotidianas y
• Utilizar escolares de
estrategias forma ordenada
personales de y adecuada ,
cálculo mental utilizando la
(en situaciones planificación.
sencillas) en
cálculos relativos
a la suma, resta,
multiplicación y
divisiones
simples.

• Aplicar los
conocimientos
adquiridos.

106 Anexo IV: Competencia Matemática


Evaluación General de Diagnóstico 2009
Marco de la evaluación

Reproducción Conexión Reflexión


Bloques
contenidos Acceso e Análisis y Síntesis y Juicio y
Compresión Aplicación
identificación valoración creación regulación
• Lng.: • Lng.: • Utilizar en • Resolver • Realizar, en
Localizar y Captar el situaciones problemas contextos reales,
recuperar sentido global cotidianas las relacionados estimaciones (a
información y algunas unidades de con el entorno partir de
explícita. informaciones medida propias que exijan cierta previsiones
específicas. del ciclo planificación, más o menos
(longitud, aplicando dos razonables) y
peso, masa, operaciones mediciones
capacidad, con números escogiendo,
tiempo y naturales como entre las
dinero). máximo. unidades e
instrumentos de
• Convertir unas • Expresar el medida visuales,
unidades de resultado de las los que mejor
medida en mediciones en se adapten al
otras. la unidad tamaño y
de medida naturaleza del
• Aplicar los adecuada. objeto a medir
conocimientos (los más
adquiridos sobre adecuados en
la medida. función de lo
que se vaya a
• C. Medio: medir).
Ordenar
temporalmente • Explicar por
hechos escrito los
relevantes de la razonamientos
vida familiar y seguidos.
Criterios de evaluación: 4 y 8

entorno
próximo. • Utilizar
estrategias
personales para
La medida

la resolución de
problemas.

• Expresar de
forma escrita, y
ordenada el
proceso.

• C. Medio:
Comunicar de
forma escrita los
resultados
acompañados
de tablas,
gráficos, etc.

• Lng.:
Interpretar e
integrar las
ideas propias
con la
información
contenida en
los textos
adecuados al
nivel de edad.

• Lng.:
Redactar,
reescribir y
resumir textos
en situaciones
cotidianas y
escolares de
forma ordenada
y adecuada,
utilizando la
planificación.

Anexo IV: Competencia Matemática 107


Evaluación General de Diagnóstico 2009
Marco de la evaluación

Cuadro de relaciones de Educación Primaria (continuación)

Reproducción Conexión Reflexión


Bloques
contenidos Acceso e Análisis y Síntesis y Juicio y
Compresión Aplicación
identificación valoración creación regulación
• Obtener • Comprender • Utilizar las • Clasificar • Describir, en • Valorar las
información situaciones nociones básicas tanto figuras situaciones de la diversas
puntual de una geométricas de los como cuerpos vida cotidiana, expresiones
representación de la vida movimientos con criterios una representa- artísticas.
espacial. cotidiana. geométricos. libremente ción espacial • Valorar la
• Conocer las elegidos. (croquis de un utilización de
• Reconocer • Describir itinerario, plano
formas y propiedades situaciones • Resolver propiedades
básicas de de una pista…) geométricas
cuerpos geométricas problemas tomando
geométricos. cuerpos y de la vida relacionados (alineamiento,
figuras planas. como referencia paralelismo,
cotidiana. con el entorno objetos
• Lng.: que exijan perpendiculari-
• Lng.: familiares. dad..) como
Localizar y • Utilizar los cierta
recuperar Captar el movimientos en • Tener elementos de
sentido global planificación,
información el plano para aplicando los capacidad de referencia para
explicita. y algunas emitir y recibir orientación y describir
informaciones contenidos
informaciones básicos de representación situaciones
específicas. sobre espacial, espaciales.
geometría.
situaciones teniendo en
cotidianas. • E. Art.: cuenta tanto
Interpretar el el lenguaje
• Reproducir contenido de utilizado
manifestaciones imágenes y como la
artísticas que representaciones representación
incluyan del espacio. en el plano
simetrías y de objetos y
traslaciones. contextos
cercanos.
• Identificar
Criterios de evaluación: 5, 6 y 8

manifestaciones • Utilizar
artísticas que estrategias
incluyan personales para
simetrías y la resolución de
traslaciones. problemas.
Geometría

• Describir • Utilizar más


formas y de un
cuerpos procedimiento
geométricos en la resolución.
del espacio
• Expresar de
(polígonos,
forma escrita y
círculos, cubos,
ordenada el
prismas,
proceso.
cilindros y
esferas). • C. Medio:
Comunicar de
• Clasificar forma escrita los
tanto figuras resultados
como cuerpos. acompañados
de tablas,
• Aplicar los
gráficos, etc.
conocimientos
adquiridos. • Lng.:
Interpretar e
• EF.: integrar las ideas
Girar sobre el propias con la
eje longitudinal información
y transversal contenida en
diversificando los textos
las posiciones adecuados al
segmentarias nivel de edad.
(utilizarlos en
las actividades • Lng.:
cotidianas). Redactar,
reescribir y
resumir textos
en situaciones
cotidianas y
escolares de
forma ordenada
y adecuada,
utilizando la
planificación.

108 Anexo IV: Competencia Matemática


Evaluación General de Diagnóstico 2009
Marco de la evaluación

Cuadro de relaciones de Educación Primaria (continuación)

Reproducción Conexión Reflexión


Bloques
contenidos Acceso e Análisis y Síntesis y Juicio y
Compresión Aplicación
identificación valoración creación regulación
• Lng.: • Describir un • Recoger datos • Ordenar los • Expresar el
Localizar y gráfico sencillo (con un efectivo datos resultado del
recuperar en una recuento) sobre obtenidos recuento de
información situación hechos y atendiendo a un datos en forma
explicita. familiar. objetos de criterio de de tabla o
la vida clasificación. gráfica.
• Lng.: cotidiana.
Captar el • Interpretar un • Representar los
sentido global • Utilizar gráfico sencillo resultados de
y algunas técnicas sencillas en una situación los recuentos
utilizando los
informaciones de recuento. familiar.
gráficos esta
específicas.
• Aplicar los • Resolver dísticos más
• C. Medio: conocimientos problemas adecuados.
Obtener adquiridos. relacionados • Utilizar
información con el entorno estrategias
relevante que exijan cierta personales para
sobre hechos o planificación, la resolución de
fenómenos aplicando los problemas.
delimitados. contenidos
básicos de • Utilizar más
tratamiento de de un
la información. procedimiento
en la resolución.

• Expresar de
Tratamiento de la información

forma escrita y
ordenada el
Criterios de evaluación: 7 y 8

proceso.

• C. Medio:
Hacer
predicciones
sobre sucesos
naturales y
sociales
integrando datos
de observación
directa e
indirecta.

• C. Medio:
Comunicar de
forma escrita los
resultados
acompañados de
tablas, gráficos,
etc.
• Lng.:
Interpretar e
integrar las ideas
propias con la
información
contenida en
los textos
adecuados al
nivel de edad.

• Lng.:
Redactar,
reescribir y
resumir textos en
situaciones
cotidianas y
escolares de
forma ordenada
y adecuada ,
utilizando la
planificación.

Anexo IV: Competencia Matemática 109


Evaluación General de Diagnóstico 2009
Marco de la evaluación

Cuadro de relaciones de Educación Secundaria Obligatoria

Reproducción Conexión Reflexión


Bloques
contenidos Acceso e Análisis y Síntesis y Juicio y
Compresión Aplicación
identificación valoración creación regulación
• Utilizar • Utilizar • Utilizar los • Utilizarlos • C. Nat.: • Valorar la
números números números y sus para resolver Resolver adecuación del
naturales, naturales, operaciones y problemas problemas resultado al
enteros, enteros, propiedades relacionados aplicando los contexto.
fracciones, fracciones, para transformar con la vida conocimientos
decimales y decimales y información. diaria. sobre el • Estimar la
porcentajes porcentajes concepto de coherencia y
sencillos sencillos para • Aplicar los • Elegir la forma temperatura y precisión de los
para recoger intercambiar cálculos de cálculo su medida, el resultados
información. información. numéricos a una apropiada equilibrio y el obtenidos.
amplia variedad (mental, desequilibrio
• Identificar los • Emplear los de contextos. escrita o con térmico, los
números y las números y las calculadora), efectos del calor
operaciones operaciones • Usar la utilizando sobre los cuerpos
hasta las de manera calculadora distintas y su forma de
potencias de adecuada, para realizar estrategias que propagación.
exponente siendo operaciones. permitan
natural. consciente de su simplificar el
• Utilizar las
significado y cálculo con
relaciones de
propiedades. fracciones,
proporcionalidad
decimales y
• Identificar en numérica para
porcentajes.
diferentes resolver
contextos problemas en
relaciones de situaciones de la
proporcionalidad vida cotidiana.
Critterios de evaluación: 1, 2 y 4

numérica entre
• Utilizar
dos magnitudes.
diferentes
• C. Nat.: estrategias
Identificar el (empleo de
Números

equilibrio tablas, obtención


térmico con la y uso de la
igualación de constante de
temperaturas. proporcionalidad,
reducción a la
unidad, etc.)
para obtener
elementos
numéricos
desconocidos en
un problema a
partir de otros
conocidos, en
situaciones de
la vida real en
las que existan
relaciones de
proporcionalidad.
• Utilizar en la
estimación o
el cálculo de
medidas una
precisión
acorde con la
situación
planteada.

• C.Nat.:
Utilizar
termómetros y
conocer su
fundamento.

110 Anexo IV: Competencia Matemática


Evaluación General de Diagnóstico 2009
Marco de la evaluación

Cuadro de relaciones de Educación Secundaria Obligatoria (continuación)

Reproducción Conexión Reflexión


Bloques
contenidos Acceso e Análisis y Síntesis y Juicio y
Compresión Aplicación
identificación valoración creación regulación
• Identificar • Utilizar el • Incorporar el • Valorar la • Poner en • Valorar la
regularidades, lenguaje planteamiento y utilización de práctica coherencia de
pautas y algebraico para resolución de métodos de estrategias los resultados.
relaciones en simbolizar y ecuaciones de ensayo y personales,
conjuntos de generalizar. primer grado error en la como alternativa
números. como una resolución al álgebra, a la
• Describir herramienta de un hora de plantear
• Percibir en un regularidades, más con la que planteamiento y resolver los
conjunto pautas y abordar y de ecuaciones problemas.
numérico relaciones en resolver de primer
aquello que es conjuntos de problemas. grado.
común, la números.
secuencia lógica • Plantear • Utilizar el
con que se ha • Interpretar ecuaciones de lenguaje
construido y un relaciones primer grado algebraico para
funcionales generalizar
Criterios de evaluación: 3 y 5

criterio que para resolverlas


permita ordenar sencillas dadas por métodos propiedades
sus elementos. en forma de algebraicos. sencillas y
expresión simbolizar
algebraica. • Utilizar el relaciones.
Álgebra

lenguaje
algebraico para
calcular el valor
numérico de una
expresión
algebraica.

• Obtener
valores a partir
de las relaciones
funcionales
en forma
algebraica.

• Pasar de la
gráfica a la forma
algebraica,
para el caso de
relaciones de
proporcionalidad.

Anexo IV: Competencia Matemática 111


Evaluación General de Diagnóstico 2009
Marco de la evaluación

Cuadro de relaciones de Educación Secundaria Obligatoria (continuación)

Reproducción Conexión Reflexión


Bloques
contenidos Acceso e Análisis y Síntesis y Juicio y
Compresión Aplicación
identificación valoración creación regulación
• Reconocer y • Identificar • Aplicar el • Utilizar • Utilizar las
describir figuras relaciones de conocimiento diferentes relaciones de
planas. proporcionalidad geométrico estrategias proporcionalidad
geométrica. adquirido para (empleo de geométrica para
interpretar y tablas, resolver
• Identificar, en describir el obtención y problemas en
diferentes mundo físico, uso de la situaciones de la
contextos, haciendo uso de constante de vida cotidiana.
una relación de la terminología proporcionalidad,
semejanza entre adecuada. reducción a la • Comprender
dos figuras unidad, etc.) los conceptos
planas. • Utilizar los para obtener implicados en el
conceptos elementos proceso de
• Utilizar las básicos de la estimación o
geométricos
propiedades de geometría para cálculo y la
desconocidos en
las figuras abordar diversidad de
un problema a
planas para diferentes métodos que se
partir de otros
clasificarlas. situaciones y es capaz de
conocidos, en
problemas de la situaciones de la poner en
• Utilizar
vida cotidiana. vida real en las marcha.
diferentes
elementos y que existan
Criterios de evaluación: 2 y 4

• EPV.: • Seleccionar la
formas relaciones de
Representar unidad adecuada
geométricas. semejanza de
objetos e ideas para cada
figuras planas.
de forma estimación o
Geometría

• Comprender
bidimensional • Estimar y cálculo.
los procesos de
aplicando calcular ángulos,
medida.
técnicas gráficas. longitudes, áreas
• Diferenciar los y volúmenes
• Tec.:
conceptos de de espacios y
Representar
longitud, objetos
mediante vistas
superficie y (utilizando para
y perspectivas
volumen. la estimación
objetos y
diferentes
• Comprender y sistemas
métodos:
los conceptos técnicos
descomposición
de longitud, sencillos (alzado,
en figuras planas
superficie y planta y perfil).
elementales,
volumen.
etc.).
• Tec.:
• Expresar el
Adquirir
resultado de la
destrezas para la
estimación o
realización de
el cálculo en la
perspectivas a
unidad de
mano alzada,
medida más
mediante
adecuada.
instrumentos de
dibujo.

112 Anexo IV: Competencia Matemática


Evaluación General de Diagnóstico 2009
Marco de la evaluación

Cuadro de relaciones de Educación Secundaria Obligatoria (continuación)

Reproducción Conexión Reflexión


Bloques
contenidos Acceso e Análisis y Síntesis y Juicio y
Compresión Aplicación
identificación valoración creación regulación
• Identificar las • Identificar • Organizar • Interpretar de • Extraer • CSGH.:
variables que relaciones de informaciones forma cualitativa conclusiones Presentar
intervienen en dependencia diversas la información acerca del conclusiones de
una situación en situaciones mediante tablas presentada en fenómeno un pequeño
cotidiana. cotidianas. y gráficas. forma de tablas estudiado. trabajo de
y gráficas. síntesis e
• CSGH.: • Interpretar • Obtener • CSGH.: indagación
Conocer los relaciones valores a • Analizar una Planificar y correctamente.
conceptos funcionales partir de las gráfica y realizar, un
básicos de la sencillas dadas relaciones relacionar el trabajo sencillo
demografía. en forma de funcionales. resultado de de síntesis sobre
tabla, gráfica o ese análisis con algún hecho o
• CSGH.: mediante un
• Usar las el significado de tema, utilizando
Conocer las enunciado.
tablas como las variables diversas fuentes.
tendencias del instrumento representadas.
Funciones y gráficas

Criterio de evaluación: 5

crecimiento. • Obtener la para recoger • CSGH.:


relación de información y • CSGH.: Planificar y
dependencia transferirla a Utilizar los realizar un
entre las unos ejes conceptos pequeño trabajo
variables coordenados. básicos de la de síntesis.
relacionadas, demografía para
• Manejar los su análisis. • CSGH.:
y visualizarla
mecanismos que Comunicar los
gráficamente.
relacionan • CSGH.: resultados del
los distintos Integrar la estudio con
tipos de información corrección y con
presentación de pertinente en el vocabulario
la información un esquema o adecuado.
(en especial, guión.
pasar de la
• CSGH.:
gráfica a la
Seleccionar
forma verbal o
información
numérica, para
pertinente,
el caso de
de manera
relaciones de
adecuada, en
proporcionalidad).
función del
objetivo
propuesto
en una
información de
tipo gráfico.

Reproducción Conexión Reflexión


Bloques
contenidos Acceso e Análisis y Síntesis y Juicio y
Compresión Aplicación
identificación valoración creación regulación
• Formular las • Diferenciar los • Recoger y • Utilizar los • Obtener
preguntas fenómenos organizar la métodos conclusiones
adecuadas para deterministas información estadísticos razonables a
conocer las de los aleatorios. obtenida en tablas apropiados para partir de los datos
características de y gráficas. realizar el análisis. obtenidos.
Estadísticas y probabilidad

una población.
• Presentar datos • Hacer
Criterio de evaluación: 6

• Formular la relevantes para predicciones sobre


pregunta o responder a las la posibilidad de
preguntas que preguntas que un suceso
darán lugar al planteadas en un ocurra a partir de
estudio. principio. información
previamente
• Hallar valores
obtenida de
relevantes: media,
forma empírica.
moda, valores
máximo y mínimo, • Hacer
rango. predicciones
razonables en
• Utilizar la
procesos aleatorios
calculadora para
a partir del análisis
la recoger y
de regularidades
organizar los datos
en la repetición
obtenidos.
sucesiva de los
mismos.

Anexo IV: Competencia Matemática 113


Evaluación General de Diagnóstico 2009
Marco de la evaluación

Cuadro de relaciones de Educación Secundaria Obligatoria (continuación)

Reproducción Conexión Reflexión


Bloques
contenidos Acceso e Análisis y Síntesis y Juicio y
Compresión Aplicación
identificación valoración creación regulación
• Comprender el • Aplicar las • Expresar, • Utilizar • Comprobación
enunciado de estrategias de utilizando el estrategias y de la coherencia
los problemas a resolución de lenguaje técnicas de de la solución
partir del problemas. matemático resolución de obtenida.
análisis de cada adecuado a problemas
• Lng.: • Tener
una de las su nivel, el mediante:
Seguir perseverancia en
partes del procedimiento análisis del
instrucciones la búsqueda de
texto y la que se ha enunciado,
escritas para soluciones y la
identificación de seguido en la división del
realizar tareas confianza en la
los aspectos más resolución. problema en
constitutivas por propia capacidad
relevantes. partes, ensayo y
una secuencia • Tener hábito y para lograrlo y
error sistemático,
• Lng.: de más de tres la destreza valorar la
la resolución de
Reconocer, junto actividades. necesarios para capacidad de
un problema
al propósito y la comprobar la transmitir con
• Lng.: más sencillo, etc.
idea general, corrección de la un lenguaje
Aplicar
hechos o datos solución. • Lng.: suficientemente
estrategias de
relevantes en Resumir preciso, las ideas
lectura como la • Tec.:
textos de por escrito y procesos
inferencia de Analizar el
ámbitos sociales presentaciones personales
significados con contexto de un
próximos a la breves, desarrollados,
el contexto. problema.
experiencia del estructuradas de modo que se
alumnado y en • Tec.: con claridad y hagan entender
el ámbito Emplear la con registro y entiendan
académico. resolución divulgativo, a sus
técnica de relacionadas compañeros.
Común a todos los bloques

• Lng.:
problemas. con temas
Comprender la • EPV.:
académicos.
información Tomar
Criterio de evaluación: 7

general y • Saber decisiones


específica de enfrentarse especificando los
diferentes a tareas de objetivos y las
textos escritos, resolución de dificultades y
adaptados y problemas para proponer
auténticos, y los que no se diferentes
adecuados a la dispone de un opciones
edad. procedimiento teniendo en
estándar que cuenta las
permita obtener consecuencias.
la solución.
• Tec.:
• Lng.: Proponer
Realizar soluciones
exposiciones alternativas a un
escritas sencillas problema, y
sobre temas desarrollar la más
próximos a su adecuada.
entorno que
• Tec.:
sean del interés
Valorar las
del alumnado.
necesidades
• Lng.: del proceso
Presentar de tecnológico.
forma
ordenada y clara
informaciones,
previamente
preparadas de
interés próximo
a su experiencia.
• Tec.:
Elaborar un
plan de
trabajo para
ejecutar un
proyecto
técnico.

114 Anexo IV: Competencia Matemática


Evaluación General de Diagnóstico 2009
Marco de la evaluación

4. Matriz de especificaciones de la competencia

Se presentan las matrices de especificaciones con la propuesta de pesos ponderales para


cada uno de los niveles objeto de evaluación.

Matriz de especificaciones de Educación Primaria

4º Educación Primaria
Procesos
Bloques de Reproducción Conexión Reflexión
Total
contenidos Acceso e Análisis y Síntesis y Juicio y
Comprensión Aplicación
identificación valoración creación regulación

Números y
35%
operaciones

La medida 20%

Geometría 25%

Tratamiento de la
información, azar 20%
y probabilidad

Total 10% 15% 25% 20% 20% 10% 100%

Matriz de especificaciones de Educación Secundaria Obligatoria

2º Educación Secundaria Obligatoria

Procesos
Reproducción Conexión Reflexión
Bloques de Total
contenidos Acceso e Análisis y Síntesis y Juicio y
Comprensión Aplicación
identificación valoración creación regulación

Números y
20%
operaciones

Algebra 15%

Geometría 15%

Funciones y
20%
gráficas

Estadísticas y
15%
probabilidad

Común a todos
15%
los bloques

Total 10% 20% 25% 15% 20% 10% 100%

Anexo IV: Competencia Matemática 115


Evaluación General de Diagnóstico 2009
Marco de la evaluación

5. Ejemplo de unidad de evaluación para Educación Primaria


(final del segundo ciclo)

RECREO

El patio de recreo del colegio


tiene forma rectangular. En él hay
pintadas dos pistas de fútbol, de
40 m x 20 m, y dos de balonces-
to, de 26 m x 13 m. Las pistas tie- b
nen una separación de 5 metros
entre ellas y con los límites del
patio, salvo en la zona E, reserva-
da para nuevas pistas.
a

1. ¿Cuáles son las pistas de fútbol?

A A y B.
B C y D.
C A y C.
D B y D.

2. ¿Qué longitud tiene el patio de recreo? Se pide la longitud del lado (a).

A 66 m.
B 76 m.
C 81 m.
D 86 m.

3. ¿Cuánto mide el perímetro de una pista de fútbol?

A 60 m.
B 90 m.
C 120 m.
D 160 m.

116 Anexo IV: Competencia Matemática


Evaluación General de Diagnóstico 2009
Marco de la evaluación

4. Se va a pintar el contorno (perímetro) de las dos pistas de baloncesto. El presu-


puesto, pintura y mano de obra, es de 1 ? el metro lineal. ¿Cuánto costará en
total?

A 39 €.
B 78 €.
C 117 €.
D 156 €.

5. Se quiere colocar una valla alrededor de una de las pistas de fútbol, de modo que
quede una franja entre la valla y la pista de fútbol, zona gris de la figura, de 3 m
de ancho. ¿Cuál será la longitud total de la valla?

A 126 m.
B 132 m.
C 138 m.
D 144 m. Pista de fútbol

6. Se quiere pintar una pista polideportiva de 14 m x 28 m en el espacio sobrante del


patio, zona E, que tiene unas dimensiones de 24 m x 41 m, que respete una dis-
tancia de 5 m tanto de las otras pistas, como de los límites del patio de recreo.
Responde si es posible y explica por qué.

¿Es posible?

¿Por qué?

Anexo IV: Competencia Matemática 117


ANEXO V

COMPETENCIA EN EL CONOCIMIENTO Y LA INTERACCIÓN CON EL MUNDO FÍSICO

1. Presentación de la competencia en el conocimiento y la interacción con el


mundo físico
Los Reales Decretos que establecen las enseñanzas mínimas describen la competencia en el
conocimiento y la interacción con el mundo físico como “la habilidad para interactuar con el
mundo físico, tanto en sus aspectos naturales como en los generados por la acción humana,
de tal modo que se posibilita la comprensión de sucesos, la predicción de consecuencias y la
actividad dirigida a la mejora y preservación de la condiciones de vida propia, de las demás
personas y del resto de los seres vivos”. Esta competencia “incorpora habilidades para desen-
volverse adecuadamente, con autonomía e iniciativa personal, en ámbitos de la vida y del
conocimiento muy diversos (salud, actividad productiva, consumo, ciencia, procesos tecnoló-
gicos, etc.) y para interpretar el mundo, lo que exige la aplicación de los conceptos y princi-
pios básicos que permiten el análisis de los fenómenos desde los diferentes campos de cono-
cimiento científico involucrados”.

“También incorpora la aplicación de algunas nociones, conceptos científicos y técnicos, y


de teorías científicas básicas previamente comprendidas. Esto implica la habilidad progre-
siva para poner en práctica los procesos y actitudes propios del análisis sistemático y de
indagación científica”.

La competencia en el conocimiento y la interacción con el mundo físico supone, en defi-


nitiva, el desarrollo y la aplicación del pensamiento tecnocientífico “para interpretar la
información que se recibe y para predecir y tomar decisiones con iniciativa y autonomía
personal en un mundo en el que los avances que se van produciendo en los ámbitos cien-
tífico y tecnológico tienen una influencia decisiva en la vida personal, la sociedad y el
mundo natural. Asimismo, implica la diferenciación y valoración del conocimiento cientí-
fico al lado de otras formas de conocimiento, y la utilización de valores y criterios éticos
asociados a la ciencia y el desarrollo tecnológico”.

Anexo V: Competencia en el Conocimiento y la Interacción con el Mundo Físico 119


Evaluación General de Diagnóstico 2009
Marco de la evaluación

2. Dimensiones de la competencia
Para la evaluación general de diagnóstico de la competencia en el conocimiento y la inter-
acción con el mundo físico, el grupo de trabajo propone adoptar y adaptar de acuerdo
con lo establecido en el currículo español el marco para la evaluación de las ciencias pro-
puesto por el proyecto PISA.

En el marco de la evaluación PISA 2006, se caracteriza la competencia científica a partir de


cuatro aspectos interrelacionados: contextos; capacidades, habilidades o competencias;
conocimientos, y actitudes. Así pues, de acuerdo con este marco, se describe la compe-
tencia en el conocimiento y la interacción con el mundo físico para las pruebas generales
de diagnóstico a partir de los cuatro aspectos citados: contextos y situaciones, procesos,
bloques de contenidos y actitudes.

2.a. Contextos y situaciones

La competencia en el conocimiento y la interacción con el mundo físico se despliega en


unas situaciones determinadas de la vida real. Estas situaciones son la parte del universo
del estudiante en el que se sitúan las tareas que se han de realizar, es decir, situaciones de
la vida diaria en las que el alumnado tiene que aplicar sus conocimientos para interactuar
con el mundo físico. En el caso de las pruebas de evaluación, el contexto de una pregun-
ta es el marco concreto en el que se presenta una situación determinada7. Las pruebas de
evaluación sobre la competencia en el conocimiento y la interacción con el mundo físico
no deben limitarse a situaciones de la vida escolar, sino que deben enmarcarse en situa-
ciones cotidianas de la vida real.

Las situaciones se pueden considerar desde una triple perspectiva: personal (situaciones pro-
blemáticas relacionadas con el estudiante, su familia o sus compañeros), social (relacionadas
con la comunidad próxima) y global (consideradas a escala mundial). Por ello, las preguntas
de las pruebas de evaluación pueden versar sobre situaciones de la vida real en las que se plan-
teen problemas al estudiante como individuo (por ejemplo, la alimentación), como ciudada-
no de una comunidad próxima (por ejemplo, la construcción de embalses) o como ciudada-
no del mundo (por ejemplo, el calentamiento por efecto invernadero).

Las situaciones en las que se puede desplegar la competencia en el conocimiento y la inter-


acción con el mundo físico se pueden considerar también desde las diversas áreas de apli-
cación de la competencia. De todas las posibles áreas de aplicación que pueden

7 PISA 2006. Marco de la evaluación.

120 Anexo V: Competencia en el Conocimiento y la Interacción con el Mundo Físico


Evaluación General de Diagnóstico 2009
Marco de la evaluación

seleccionarse, para la evaluación de la competencia en las pruebas generales de diagnós-


tico se han elegido cuatro; para esta elección se ha seguido el criterio de priorizar los con-
textos relacionados con problemas y temas que tienen repercusión en el bienestar huma-
no. Las cuatro áreas de aplicación seleccionadas son:
1. La vida y la salud.
2. La Tierra y el medio ambiente.
3. Los riesgos.
4. Las relaciones entre la ciencia y la tecnología.

Combinando las cuatro áreas de aplicación seleccionadas con las tres perspectivas contex-
tuales citadas, resultan doce marcos situacionales para elegir los contextos de las pruebas de
diagnóstico y seleccionar a partir de ellos los estímulos para las unidades de evaluación. Así,
el contexto de la higiene personal implica la consideración del área contextual de la vida y
la salud con una perspectiva personal, mientras que el contexto del cambio climático com-
bina el área de aplicación de los riesgos con la perspectiva global.

En la siguiente tabla se recogen posibles contextos, a modo de ejemplo, en cada uno de


los marcos situacionales descritos.

Contextos de la competencia en el conocimiento y la interacción con el mundo físico

Perspectivas contextuales
Áreas de aplicación
Personal Social Global

La vida y la salud Conservación de la salud, Distribución y utilización del Epidemias, propagación de


higiene personal, nutrición. agua, salud comunitaria. enfermedades infecciosas.

Consumo familiar de energía, Eliminación de residuos, Recursos renovables,


La Tierra y el uso y desecho de materiales, transporte y utilización de la desarrollo sostenible,
medio ambiente conservación del medio. energía, contaminación biodiversidad, erosión de
acústica. suelos.

Los riesgos Seguridad en el laboratorio, Erupciones volcánicas y Cambio climático, usos


riesgos naturales. terremotos. bélicos de la ciencia.

Explicación científica de Nuevos materiales, Exploración del universo.


Las relaciones entre
fenómenos naturales, uso de manipulación genética,
la ciencia y la tecnología
nuevas tecnologías. transportes.

La evaluación del grado de adquisición de la competencia en el conocimiento y la interac-


ción con el mundo físico no es, en ningún caso, una evaluación de contextos. Estos per-
miten remedar situaciones de la vida real en las que se apliquen los conocimientos para
interactuar con el mundo físico; se valoran los conocimientos sobre el mundo físico y el
desarrollo de destrezas y actitudes para interactuar con él, pero no los contextos. Las situa-
ciones contextuales sirven de referencia para diseñar estímulos para las unidades de eva-
luación que permitan valorar los distintos elementos de la competencia.

Anexo V: Competencia en el Conocimiento y la Interacción con el Mundo Físico 121


Evaluación General de Diagnóstico 2009
Marco de la evaluación

2.b. Procesos
El conocimiento y la interacción con el mundo físico requiere el desarrollo de destrezas y
habilidades como la aplicación del pensamiento tecnocientífico para interpretar la infor-
mación que se recibe y para predecir y tomar decisiones en situaciones de la vida cotidia-
na, la habilidad progresiva para explicar científicamente los fenómenos del mundo físico,
y para planificar y manejar soluciones técnicas “para satisfacer las necesidades de la vida
cotidiana y del mundo laboral”. La competencia en el conocimiento y la interacción con
el mundo físico “hace posible identificar preguntas o problemas y obtener conclusiones
basadas en pruebas, con la finalidad de comprender y tomar decisiones sobre el mundo
físico y sobre los cambios que la actividad humana produce sobre el medio ambiente, la
salud y la calidad de vida de las personas”8. En esta competencia se incluyen tres grupos
de habilidades, capacidades o competencias: 1) la habilidad para distinguir entre las cues-
tiones científicas y no científicas, 2) la capacidad para interpretar fenómenos del mundo
físico y predecir cambios y 3) la capacidad de obtener conclusiones basadas en pruebas.

Por ello, los procesos propios de la competencia en el conocimiento y la interacción con


el mundo físico, que se espera que hayan desarrollado los estudiantes de 10 y 14 años, son
los siguientes:

• Identificar temas científicos. Es decir, reconocer preguntas o problemas sobre


los cuales es posible investigar científicamente, identificar palabras clave para
buscar información relacionada con el mundo físico o la interacción con él, y
reconocer las características clave de la indagación científica.

• Explicar científicamente fenómenos del mundo físico. Implica aplicar conoci-


mientos sobre el mundo físico a una situación dada, describir fenómenos del
mundo físico de forma científica, interpretar fenómenos de forma científica y
predecir cambios, e identificar convenientemente descripciones, explicaciones y
predicciones en relación con el mundo físico.

• Utilizar pruebas científicas. Comprende interpretar pruebas científicas, identifi-


car las suposiciones, las pruebas y los razonamientos que hay detrás de las con-
clusiones, elaborar y comunicar conclusiones, y reflexionar sobre las implicacio-
nes sociales de los desarrollos científicos y tecnológicos.

8 RR.DD de Enseñanzas Mínimas. Competencia en el conocimiento y la interacción con el mundo físico.

122 Anexo V: Competencia en el Conocimiento y la Interacción con el Mundo Físico


Evaluación General de Diagnóstico 2009
Marco de la evaluación

La adquisición de estas destrezas y habilidades es progresiva, por lo que su grado de adqui-


sición no es el mismo a los 10 que a los 14 años. A los 10 años solo cabe esperar que el
alumnado haya desarrollado las habilidades para identificar y reconocer aspectos del
mundo físico susceptibles de indagación; describir fenómenos sencillos, elementos del
entorno y aparatos simples; relacionar factores del entorno con las formas de vida y actua-
ciones de las personas, y hacer predicciones sobre sucesos naturales y sociales. En el alum-
nado de 14 años cabe esperar además habilidades para reconocer algunas características
clave del trabajo científico, interpretar determinados fenómenos naturales, explicar cam-
bios en el mundo físico, utilizar pruebas para elaborar conclusiones y reflexionar sobre las
implicaciones sociales del desarrollo tecnocientífico.

2.c. Bloques de contenidos

La competencia en el conocimiento y la interacción con el mundo físico incorpora la habi-


lidad “para interpretar el mundo físico, lo que exige la aplicación de los conceptos y prin-
cipios básicos que permiten el análisis de los fenómenos desde los diferentes campos de
conocimiento científico involucrados”. Es decir, el conocimiento del mundo natural
requiere el aprendizaje de los conceptos y procedimientos característicos de las ciencias de
la naturaleza. Por ello, la competencia en el conocimiento y la interacción con el mundo
físico incorpora “la aplicación de algunas nociones, conceptos científicos y técnicos, y de
teorías científicas básicas previamente comprendidas”9.

Los conocimientos sobre el mundo físico se refieren, en consecuencia, tanto al conoci-


miento sobre el mundo natural como al conocimiento acerca de los procedimientos para
interactuar con el mundo físico o conocimiento acerca de las ciencias de la naturaleza.

2.c.1. Conocimiento del mundo natural

Puesto que solo se puede evaluar una muestra del conocimiento del mundo físico del
alumnado, se presentan a continuación los criterios utilizados para la selección de los blo-
ques de contenidos:
• relevancia para las situaciones de la vida diaria,
• conceptos del mundo físico importantes y, por tanto, de utilidad a largo plazo,
• conocimientos apropiados para el nivel de desarrollo de estudiantes de 10 años
y de jóvenes de 14 años.

9 RR.DD de Enseñanzas Mínimas. Competencia en el conocimiento y la interacción con el mundo físico.

Anexo V: Competencia en el Conocimiento y la Interacción con el Mundo Físico 123


Evaluación General de Diagnóstico 2009
Marco de la evaluación

Los bloques seleccionados son materia y energía, los seres vivos, el entorno próximo y su
conservación, la Tierra y el Universo, y ciencia, tecnología y sociedad. Se indican a conti-
nuación algunas referencias del nivel de conocimientos esperados en los estudiantes de 10
años y en los jóvenes de 14 años. Estas referencias no delimitan los contenidos de cada
bloque sino que, a modo de ejemplo, señalan el grado de adquisición de conocimientos
del mundo físico que pueden esperarse en cada edad.

a) ALUMNADO DE 10 AÑOS:

Materia y energía. Por ejemplo: Medida, comparación y ordenación de propiedades de los


materiales. Conservación de la masa y el volumen con los cambios de forma. Identificación
de distintas fuentes de energía relacionadas con los cambios observados en la vida cotidia-
na. Transmisión del sonido en los distintos medios.

Los seres vivos. Por ejemplo: Aves, mamíferos, reptiles, peces, anfibios; características bási-
cas, reconocimiento y clasificación. Plantas: hierbas, arbustos y árboles; características bási-
cas, reconocimiento y clasificación. Identificación y adopción de hábitos de higiene, de
descanso y de alimentación sana.

El entorno próximo y su conservación. Por ejemplo: Variables meteorológicas: temperatura,


humedad, viento, precipitaciones. Orientación en el espacio. Uso de planos del barrio o
de la localidad. Observación y descripción de distintos tipos de paisaje: interacción de
naturaleza y seres humanos. El ciclo del agua.

La Tierra y el Universo. Por ejemplo: Movimientos de la Tierra y fases de la Luna. Las esta-
ciones del año.

Ciencia, tecnología y sociedad. Por ejemplo: Identificación de la diversidad de máquinas en


el entorno. Relevancia de algunos de los grandes inventos y de su contribución a la mejo-
ra de las condiciones de vida.

b) ALUMNADO DE 14 AÑOS:

Materia y energía. Por ejemplo: Propiedades generales de la materia y estados en los que
se presenta. Análisis y valoración de las distintas fuentes de energía, renovables y no reno-
vables. Propagación y reflexión del sonido.

Los seres vivos. Por ejemplo: La respiración en los seres vivos. Introducción al estudio de la
biodiversidad. Las funciones de relación: percepción, coordinación y movimiento.

124 Anexo V: Competencia en el Conocimiento y la Interacción con el Mundo Físico


Evaluación General de Diagnóstico 2009
Marco de la evaluación

El entorno próximo y su conservación. Por ejemplo: Biosfera, ecosfera y ecosistema.


Influencia de los factores abióticos y bióticos en los ecosistemas. Ecosistemas acuáticos de
agua dulce y marinos.

La Tierra y el Universo. Por ejemplo: El lugar de la Tierra en el Universo: el paso del geocen-
trismo al heliocentrismo como primera y gran revolución científica. Diversidad de rocas y
minerales y características que permitan identificarlos. Introducción a la estructura interna
de la Tierra.

Ciencia, tecnología y sociedad. Por ejemplo: Utilización de los medios de comunicación y las
tecnologías de la información y la comunicación para obtener información sobre fenóme-
nos del mundo físico. Papel del conocimiento científico en el desarrollo tecnológico y en
la vida de las personas. La contaminación acústica y lumínica.

2.c.2. Conocimientos sobre la interacción con el mundo físico

Los conocimientos sobre la interacción con el mundo físico se han agrupado en dos blo-
ques de contenidos: la investigación científica, entendida como el modo de obtener datos
sobre el mundo físico, y las explicaciones científicas, entendidas como el modo de utilizar
los datos obtenidos.

El nivel esperado de adquisición de estos conocimientos sobre la interacción con el mundo


físico es diferente para los estudiantes de 10 años y para los jóvenes de 14 años, como se
detalla a continuación, señalando algunos contenidos, a título de ejemplo, de estos blo-
ques para cada edad.

a) ALUMNADO DE 10 AÑOS:

Investigación científica. Por ejemplo: Comprensión de fenómenos físicos y situaciones, y


obtención de datos: observación, clasificación, utilización de fuentes diversas, uso de ins-
trumentos, elaboración de tablas y gráficos. Extracción de conclusiones: comparación
entre observaciones y medidas, identificación de regularidades.

Explicaciones científicas. Por ejemplo: Tipos de explicaciones: conjeturas, modelos. Normas:


coherencia en los razonamientos, basadas en pruebas.

Anexo V: Competencia en el Conocimiento y la Interacción con el Mundo Físico 125


Evaluación General de Diagnóstico 2009
Marco de la evaluación

b) ALUMNADO DE 14 AÑOS:

Investigación científica. Por ejemplo: Comprensión de fenómenos físicos y situaciones: plan-


teamiento de problemas, marco interpretativo de las observaciones, formulación de con-
jeturas, identificación de variables, diseños experimentales, utilización de fuentes diversas,
uso de instrumentos, elaboración de tablas y gráficos. Investigación de problemas, obten-
ción de datos y reconocimiento de pruebas: diseño de investigaciones, aplicación de pro-
cedimientos, elaboración de tablas y gráficos, identificación de variables, análisis de la
información. Extracción de conclusiones: comparación entre observaciones y medidas,
análisis de resultados, identificación de regularidades.

Explicaciones científicas. Por ejemplo: Tipos de explicaciones: conjeturas, hipótesis, modelos.


Normas: consistentes lógicamente, basadas en pruebas y en el conocimiento histórico y
actual. Resultados: producir nuevo conocimiento, nuevos métodos, nuevas tecnologías; con-
ducir a nuevas preguntes e investigaciones.

2.d. Actitudes relacionadas con el conocimiento y la interacción con el mundo físico

Dado que la actitud de las personas desempeña un papel importante en la comprensión


y en la interacción con el mundo físico, determinadas actitudes deben considerarse como
componentes básicos de esta competencia: el interés por los temas relacionados con el
mundo físico, las actitudes propias del trabajo científico (como el espíritu crítico, la aper-
tura a nuevas ideas, el cuestionamiento de lo obvio, etc.) y el compromiso con el futuro
sostenible manifestado en “el uso responsable de los recursos naturales, el cuidado del
medio ambiente, el consumo racional y responsable, y la protección de la salud individual
y colectiva”. Asimismo, esta competencia “implica la diferenciación y valoración del cono-
cimiento científico al lado de otras formas de conocimiento, y la utilización de valores y
criterios éticos asociados a la ciencia y el desarrollo tecnológico”10.

Uno de los objetivos de la educación es que el alumnado desarrolle actitudes que promue-
van su interés por los temas científicos y la adquisición y aplicación del conocimiento del
mundo físico para el beneficio personal, social y global. Para la evaluación de las actitudes
se puede utilizar tanto el cuestionario del alumnado como las preguntas contextualizados
para recoger información sobre las actitudes del alumnado en esta competencia.

10 RR.DD de Enseñanzas Mínimas. Competencia en el conocimiento y la interacción con el mundo físico.

126 Anexo V: Competencia en el Conocimiento y la Interacción con el Mundo Físico


Evaluación General de Diagnóstico 2009
Marco de la evaluación

No se han diferenciado las actitudes en el conocimiento y la interacción con el mundo físi-


co en función de la edad del alumnado (10 y 14 años) aunque, al igual que en los bloques
de contenidos y en las destrezas, se considera que su consecución es progresiva. Así pues,
lo único que varía es el grado de adquisición: mientras que para estudiantes de 10 años se
considera que han desarrollado las actitudes si muestran receptividad y formulan respues-
tas (tener conciencia, reconocer, mostrar disposición a...), para los de 14 años, además
de las anteriores, se espera aceptación del valor y compromiso (valorar, tender a actuar,
utilizar,...).

Las actitudes relacionadas con el conocimiento y la interacción con el mundo físico se han
agrupado en tres grandes categorías: el interés por el mundo físico, el respaldo a la inves-
tigación científica y el sentido de responsabilidad hacia los recursos y los entornos. Se
señalan en cada una, a modo de ejemplo, las actitudes que se espera que desarrolle el
alumnado.

Interés por el mundo físico

• Mostrar curiosidad por el mundo físico, los procesos e interacciones.

• Mostrar disposición para adquirir conocimientos y habilidades adicionales en


relación con el mundo físico, utilizando diversos recursos y métodos.

• Reconocer la práctica de hábitos asociados a la higiene, la alimentación, el


ejercicio físico y el descanso como hábitos saludables.

• Tender a actuar en función de actitudes y hábitos favorables al cuidado y


promoción de la salud individual y comunitaria, en aspectos relacionados con
la alimentación, las enfermedades y las adicciones.

• Reconocer que para el ejercicio de muchas profesiones son necesarios conoci-


mientos científicos y saber aplicar procesos propios de este campo del saber.

• Valorar las repercusiones del desarrollo tecnocientífico.

Respaldo a la investigación científica

• Valorar la importancia de utilizar los conocimientos científicos y las interaccio-


nes de la ciencia y la tecnología para satisfacer las necesidades humanas
individuales y comunitarias.

• Mostrar disposición a utilizar conceptos y estrategias propios del trabajo científi-


co para plantear preguntas relevantes y obtener conclusiones a partir de
evidencias y pruebas experimentales.

Anexo V: Competencia en el Conocimiento y la Interacción con el Mundo Físico 127


Evaluación General de Diagnóstico 2009
Marco de la evaluación

Sentido de responsabilidad hacia los recursos y los entornos

• Mostrar disposición a mantener la limpieza, el orden y el ahorro de materiales


y energía en la escuela y en casa.

• Mostrar disposición al uso responsable del agua y de las fuentes de energía.

• Valorar la responsabilidad personal hacia el medio ambiente y a favor de la


conservación de los recursos naturales.

• Valorar la importancia de participar en la toma de decisiones con vistas a ase-


gurar un futuro sostenible.

• Mostrar interés por las cuestiones éticas asociadas al conocimiento del mundo
físico en relación con la ciencia, el desarrollo tecnológico y la relación de las
personas con la naturaleza.

• Tender a actuar de manera coherente, responsable y solidaria en cuestiones


científicas socialmente controvertidas.

128 Anexo V: Competencia en el Conocimiento y la Interacción con el Mundo Físico


Evaluación General de Diagnóstico 2009
Marco de la evaluación

3. Cuadros de relaciones
3.a. Cuadros en los que se relacionan, a través de los criterios de evaluación, los procesos
y los bloques de contenidos
Cuadro de relaciones de Educación Primaria

Procesos
Bloques
contenidos Identificar Explicar Utilizar
temas científicos fenómenos científicamente pruebas científicas
Materia y energía
C. Medio

• Identificar fuentes de energía • poner ejemplos de usos prácticos


2º c

comunes y procedimientos y de la energía.


8

máquinas para obtenerla,


C. Medio

• Identificar diferencias en las • relacionando algunas de ellas con


1º c

propiedades elementales de los sus usos, y reconocer efectos


8

materiales, visibles de las fuerzas sobre los


objetos.
C. Medio

• Identificar animales, plantas y • Clasificar animales, plantas y


2º c
Los seres vivos

rocas. rocas, según criterios científicos.

• Identificar hábitos de alimenta- • explicar las consecuencias para


C. Medio

ción, higiene, ejercicio físico y des- la salud y el desarrollo personal de


2º c
3

canso, y determinados hábitos de


alimentación, higiene, ejercicio
físico y descanso.

• Reconocer y explicar, recogiendo


datos y utilizando aparatos de
C. Medio

medida, las relaciones entre algunos


2º c
1

factores del medio físico (relieve,


suelo, clima, vegetación...) y las
formas de vida y actuaciones de las
personas.
El entorno próximo y su conservación

• Identificar, a partir de ejemplos • y analizar el proceso seguido por


C. Medio

de la vida diaria, algunos de los algún bien o servicio, desde su


2º c

principales usos que las personas origen hasta el consumidor,


4

hacen de los recursos naturales, señalando ventajas e


inconvenientes.

• Utilizar las nociones espaciales y la


referencia a los puntos cardinales
C. Medio

para situarse en el entorno, para


2º c

localizar y describir la situación de


6

los objetos en espacios delimitados,


y utilizar planos y mapas con escala
gráfica para desplazarse.
E. Art.

• Describir las características de


2º c
1

elementos presentes en el entorno.

• Interpretar el contenido de
E. Art.
2º c

imágenes y representaciones del


6

espacio presentes en el entorno.

• Resolver problemas relacionados


Mat.

con el entorno que exijan cierta


2º c
8

planificación utilizando estrategias


personales de resolución.

Anexo V: Competencia en el Conocimiento y la Interacción con el Mundo Físico 129


Evaluación General de Diagnóstico 2009
Marco de la evaluación

Cuadro de relaciones de Educación Primaria (continuación)

Procesos
Bloques
contenidos Identificar Explicar Utilizar
temas científicos fenómenos científicamente pruebas científicas
C. Medio
Ciencia, tecnología

• Describir el funcionamiento de • Montar y desmontar objetos y


1er c
9

objetos y aparatos simples y la aparatos simples.


forma de utilizarlos con precaución.
y sociedad

• Analizar las partes principales de • planificar y realizar un proceso


C. Medio

objetos y máquinas, las funciones sencillo de construcción de algún


2º c

de cada una de ellas y objeto.


9
C. Medio C. Medio

• Utilizar algunos instrumentos y


1º c
10

hacer registros claros.


La investigación científica

• Obtener información relevante


2º c
10

sobre hechos o fenómenos


previamente delimitados.

• Realizar en contextos reales esti-


maciones y mediciones escogiendo
Mat.
2º c

entre las unidades e instrumentos,


4

los que mejor se ajustan al tamaño


del objeto a medir.

• Recoger datos sobre hechos y • ordenar estos datos atendiendo a • expresar el resultado en forma de
Mat.

objetos de la vida cotidiana un criterio de clasificación y tabla o gráfica.


2º c
7

utilizando técnicas sencillas de


recuento,

• Hacer predicciones sobre sucesos • integrando datos de observación


C. Medio

naturales y sociales, directa e indirecta a partir de


2º c
10

fuentes de consulta básicas y


comunicar los resultados.

• Participar en la situaciones de
comunicación del aula normas de
Lng.
2º c

intercambio: guardar el turno de


1

palabra, escuchar, exponer con


claridad.
La explicación científica

• Expresarse de forma oral median-


te textos que presenten de manera
Lng.
2º c
2

sencilla y coherente conocimientos,


ideas, hechos y vivencias.

• Captar el sentido de textos orales


Lng.
2º c

de uso habitual reconociendo las


3

ideas principales y secundarias.

• Localizar y recuperar información


Lng.
2º c

explícita y realizar inferencias


4

directas en la lectura de textos.

• Redactar, reescribir y resumir • utilizando la planificación y


diferentes textos significativos en revisión de los textos y los aspectos
Lng.
2º c
6

situaciones cotidianas y escolares de formales, tanto en soporte papel


forma ordenada y adecuada, como digital.

• Obtener información puntual, • y describir una representación • y utilizar las nociones básicas de
espacial (croquis de un itinerario, movimientos geométricos para
Mat.
2º c

plano de una pista…) tomando describir y comprender situaciones


5

como referencia objetos familiares de la vida cotidiana.


(comparar)

130 Anexo V: Competencia en el Conocimiento y la Interacción con el Mundo Físico


Evaluación General de Diagnóstico 2009
Marco de la evaluación

Cuadro de relaciones de Educación Secundaria Obligatoria

Procesos
Bloques
contenidos Identificar Explicar Utilizar
temas científicos fenómenos científicamente pruebas científicas
• Establecer procedimientos para
describir las propiedades de
C. Nat.

materiales que nos rodean, tales



3

como la masa, el volumen, los


estados en los que se presentan y
sus cambios.

• Relacionar propiedades de los • así como aplicar algunas técnicas


materiales con el uso que se de separación.
C. Nat.
Materia y energía

hace de ellos y diferenciar entre



4

mezclas y sustancias, gracias a las


propiedades características de estas
últimas,

• Reconocer la importancia y • Utilizar el concepto cualitativo de


C. Nat.

repercusiones para la sociedad y el energía para explicar su papel en


medio ambiente de las diferentes las transformaciones que tienen


1

fuentes de energías renovables y lugar en nuestro entorno.


no renovables.
C. Nat.

• Resolver problemas aplicando los conocimientos sobre el concepto de


temperatura y su medida, el equilibrio y desequilibrio térmico, los efectos


2

del calor sobre los cuerpos y su forma de propagación.


C. Nat.

• Explicar fenómenos naturales • reproducir algunos de ellos


referidos a la transmisión de la luz teniendo en cuenta sus


3

y del sonido y propiedades.

• Reconocer que los seres vivos


están constituidos por células y que
llevan a cabo funciones vitales que
C. Nat.

les diferencian de la materia inerte.



8

• Identificar y reconocer las


peculiaridades de los grupos más
importantes, utilizando claves
dicotómicas para su identificación.

• A partir de distintas observaciones • Interpretar los aspectos


y experiencias realizadas con relacionados con las funciones
C. Nat.

organismos sencillos, comprobando vitales de los seres vivos.


Los seres vivos


6

el efecto que tienen determinadas


variables en los procesos de
nutrición, relación y reproducción.

• Identificar los hábitos higiénicos y


posturales saludables relacionados
EF

2

con la actividad física y con la vida


cotidiana.

• Incrementar las cualidades físicas


relacionadas con la salud,
EF

3

trabajadas durante el curso,


respecto a su nivel inicial.

• Reconocer a través de la práctica,


las actividades físicas que se
EF

desarrollan en una franja de la


2

frecuencia cardiaca beneficiosa


para la salud.

Anexo V: Competencia en el Conocimiento y la Interacción con el Mundo Físico 131


Evaluación General de Diagnóstico 2009
Marco de la evaluación

Cuadro de relaciones de Educación Secundaria Obligatoria (continuación)

Procesos
Bloques
contenidos Identificar Explicar Utilizar
temas científicos fenómenos científicamente pruebas científicas
C. Nat.

•Explicar, a partir del conocimiento • considerando las repercusiones de


de las propiedades del agua, el ciclo las actividades humanas en relación


6

del agua en la naturaleza y su con su utilización.


importancia para los seres vivos,
El entorno próximo
y su conservación

• Conocer las rocas y los minerales • utilizando claves sencillas y recono-


C. Nat.

más frecuentes, en especial los cer sus aplicaciones más frecuentes.



7

que se encuentran en el entorno


próximo,

• Identificar los componentes bióti- • representar gráficamente las


cos y abióticos de un ecosistema relaciones tróficas establecidas entre
C. Nat.

cercano, así como conocer las los seres vivos del mismo.

6

principales características de los


grandes biomas de la Tierra y

• Identificar y • explicar, algunos ejemplos de los • aportando medidas y conductas


CSGH

impactos que la acción humana que serían necesarias para



4

tiene sobre el medio natural limitarlos.


analizando sus causas y efectos y

• Interpretar algunos fenómenos


naturales mediante la elaboración
C. Nat.

de modelos sencillos y representa-



1

ciones a escala del Sistema Solar y


de los movimientos relativos entre
la Luna, la Tierra y el Sol.

• Describir razonadamente algunas


de las observaciones y
C. Nat.

procedimientos científicos que



2

han permitido avanzar en el


conocimiento de nuestro planeta y
del lugar que ocupa en el Universo.

• Conocer la existencia de la • llegar a interpretar cualitativamen-


atmósfera y las propiedades del aire, te fenómenos atmosféricos y valorar
C. Nat.

la importancia del papel protector



5

de la atmósfera para los seres vivos,


La Tierra y el Universo

considerando las repercusiones de la


actividad humana en la misma.
C. Nat.

• Identificar las acciones de los agentes geológicos internos en el origen del relieve

4

terrestre, así como en el proceso de formación de las rocas magmáticas y metamórficas.


C. Nat.

• Reconocer • e interpretar los riesgos asociados


a los procesos geológicos internos y


5

en su prevención y predicción.

• Localizar lugares o espacios en un • obtener información sobre el


mapa utilizando datos de espacio representado a partir de
CSGH

coordenadas geográficas y la leyenda y la simbología,


1

comunicando las conclusiones de


forma oral o escrita.

• Localizar en un mapa los elemen- • caracterizando los rasgos que


tos básicos que configuran el medio predominan en un espacio
CSGH

2

físico mundial, de Europa y España concreto.


(océanos y mares, continentes,
unidades de relieve y ríos),

• Comparar los rasgos físicos más • relacionándolos con las


destacados (relieve, clima, aguas y posibilidades que ofrecen a los
elementos biogeográficos) que grupos humanos.
CSGH

configuran los grandes medios


3

naturales del planeta, con especial


referencia a España, localizándolos
en el espacio representado y

132 Anexo V: Competencia en el Conocimiento y la Interacción con el Mundo Físico


Evaluación General de Diagnóstico 2009
Marco de la evaluación

Cuadro de relaciones de Educación Secundaria Obligatoria (continuación)

Procesos
Bloques
contenidos Identificar Explicar Utilizar
temas científicos fenómenos científicamente pruebas científicas
• Identificarlos en aplicaciones • describir propiedades básicas de • Emplear técnicas básicas de
comunes y materiales técnicos y sus variedades conformación, unión y acabado.
Tec.

comerciales: madera, metales,


4
Ciencia, tecnología

materiales plásticos, cerámicos y


pétreos.
y sociedad

• Elaborar, almacenar y recuperar,


documentos en soporte electrónico
Tec.

6

que incorporen información textual


y gráfica.

• Acceder a Internet para la


utilización de servicios básicos:
navegación para la localización de
Tec.

10

información, correo electrónico,


comunicación intergrupal y
publicación de información.

• Realizar una lectura comprensiva • comunicar la información


CSGH

de fuentes de fuentes de informa- obtenida de forma correcta por


9

ción escrita y escrito.

• Realizar de forma individual y en


grupo, con ayuda del profesor, un
trabajo sencillo de tipo descriptivo
sobre algún hecho o tema,
utilizando fuentes diversas
CSGH

(observación, prensa, bibliografía,



8

páginas Web,…) seleccionando la


información pertinente,
integrándola en un esquema o
guión y comunicando los resultados
del estudio con corrección y con el
vocabulario adecuado.

• Identificar los elementos


constitutivos esenciales
La investigación científica

1º 2º

(configuraciones estructurales,
EPV

variaciones cromáticas, orientación


espacial y textura) de objetos y/o
aspectos de la realidad.

• Reconocer, junto al propósito y la • seguir instrucciones para realizar


idea general, ideas, hechos o datos autónomamente tareas de
relevantes en textos orales de aprendizaje.
ámbitos sociales próximos a la
1º 2º
Lng.

experiencia del alumno y en el


1

ámbito académico. Captar la idea


global y la relevancia de
informaciones oídas en radio o
televisión y

• Extraer informaciones concretas e • Seguir instrucciones de cierta


identificar el propósito en textos extensión en procesos poco
escritos de ámbitos sociales complejos.
1º 2º
Lng.

próximos a la experiencia del


2

alumnado;
• Identificar el tema general y temas
secundarios y distinguir cómo está
organizada la información.

• Utilizar números naturales, • recoger, trasformar e intercambiar


1º 2º
Mat.

enteros y fracciones, decimales y información y resolver problemas


1

porcentajes para relacionados con la vida diaria.

• Representar objetos de forma bi o • aplicando técnicas gráficas y plás-


tridimensional. ticas y conseguir resultados concre-
1º 2º
EPV

tos en función de unas intenciones


2

en cuanto a los elementos visuales


(luz, sombra, textura) y de relación.

Anexo V: Competencia en el Conocimiento y la Interacción con el Mundo Físico 133


Evaluación General de Diagnóstico 2009
Marco de la evaluación

Cuadro de relaciones de Educación Secundaria Obligatoria (continuación)

Procesos
Bloques
contenidos Identificar Explicar Utilizar
temas científicos fenómenos científicamente pruebas científicas
• Identificar relaciones de • utilizarlas para resolver
proporcionalidad numérica y problemas en la vida cotidiana.
Mat.

La explicación

geométrica y
científica

• Reconocer y • describir figuras planas y • utilizar sus propiedades para


clasificarlas y aplicar el
1º 2º
Mat.

conocimiento geométrico
4

adquirido para interpretar y


describir el mundo físico, haciendo
uso de la terminología adecuada.

Nota: En el área de Ciencias de la naturaleza de los dos primeros cursos de Educación


Secundaria Obligatoria no se incluye ningún criterio de evaluación relacionado explícita-
mente con los bloques La investigación científica y Las explicaciones científicas, sin embargo,
en ambos cursos existe, dentro de los contenidos, un bloque cuyos criterios de evaluación
podrían formularse como sigue:

Procesos
Bloques
contenidos Identificar Explicar Utilizar
temas científicos fenómenos científicamente pruebas científicas
• Reconocer la importancia del • Familiarizarse con las características básicas del trabajo científico, por
La investigación

conocimiento científico para tomar medio de: planteamiento de temas, discusión de su interés, formulación de
científica (*)

decisiones sobre los objetos y conjeturas, diseños experimentales, etc., para comprender mejor los
sobre uno mismo. fenómenos naturales y resolver los problemas que su estudio plantea.

• Utilizar correctamente los


materiales e instrumentos básicos
de un laboratorio.

• Utilizar los medios de • Interpretar la información de


Las expliaciones
científicas (*)

comunicación y las tecnologías de carácter científico y utilizarla para


la información y la comunicación formarse una opinión propia y
para obtener información sobre los expresarse adecuadamente.
fenómenos naturales.

134 Anexo V: Competencia en el Conocimiento y la Interacción con el Mundo Físico


Evaluación General de Diagnóstico 2009
Marco de la evaluación

3.b. Tabla resumen de las actitudes y su relación con los criterios de evaluación

Tabla de actitudes de Educación Primaria

Actitudes
C. Medio C. Medio

Valorar las consecuencias para la salud y el desarrollo personal de determinados hábitos de alimentación,
1erc
Interés por

3
el mundo

higiene, ejercicio físico y descanso.


físico

2º c

Mostrar actitudes de cooperación en el trabajo en equipo y el cuidado por la seguridad.


9
Respaldo a la
investigación
científica

C. Medio C. Medio C. Medio

Reconocer la importancia de la sostenibilidad del equilibrio ecológico y la necesidad de adoptar actitudes


2º c
1

respetuosas con el medio, la necesidad de conservar estos recursos, especialmente con respecto al uso del
hacia los recursos

agua.
responsabilidad

y los entornos
Sentido de

2º c
4

Darse cuenta de las consecuencias derivadas del uso inadecuado del medio y de los recursos.
2º c
8

Valorar la importancia de hacer un uso responsable de las fuentes de energía del planeta.

Tabla de actitudes de Educación Secundaria Obligatoria

Actitudes
C. Nat. C. Nat. C. Nat.

5

Valorar el efecto que tienen determinadas variables en los procesos de nutrición, relación y reproducción.
Interés por
el mundo
físico

Valorar la importancia del papel protector de la atmósfera para los seres vivos, considerando las repercu-

5

siones de la actividad humana en la misma.


Valorar los riesgos asociados a los procesos geológicos internos, su predicción y prevención.
4
Respaldo a la
investigación
científica

C. Nat. C. Nat.

Reconocer la importancia y repercusiones para la sociedad y el medio ambiente de las diferentes fuentes

hacia los recursos

1
responsabilidad

y los entornos

de energía renovable y no renovable.


Sentido de


6

Valorar las repercusiones de las actividades humanas en relación con la utilización del agua.
CSGH

Apreciar las medidas y conductas que serían necesarias para limitar los impactos que la acción humana

4

tiene sobre el medio natural.

Anexo V: Competencia en el Conocimiento y la Interacción con el Mundo Físico 135


Evaluación General de Diagnóstico 2009
Marco de la evaluación

4. Matriz de especificaciones de la competencia


Los pesos ponderales de los procesos y de los conocimientos que se proponen para las
pruebas de diagnóstico son:

Matriz de especificaciones de Educación Primaria

Procesos
Explicar
Bloques de contenidos Identificar Utilizar pruebas Pesos (%)
fenómenos
temas científicos científicas
científicamente

Materia y energía 15+- 5


Los seres vivos 25+- 5

El entorno próximo y su conservación 25+- 5

La Tierra y el universo 10+- 5

Ciencia, tecnología y sociedad 15+- 5

La investigación científica y las


10+- 5
explicaciones científicas

Pesos (%) 40 +-10 40 +-10 20 +-5

Matriz de especificaciones de Educación Secundaria Obligatoria

Procesos
Explicar
Bloques de contenidos Identificar Utilizar pruebas Pesos (%)
fenómenos
temas científicos científicas
científicamente

Materia y energía 25+- 5


Los seres vivos 20+- 5

El entorno próximo y su conservación 10+- 5

La Tierra y el universo 20+- 5

Ciencia, tecnología y sociedad 10+- 5

La investigación científica y las


15+- 5
explicaciones científicas

Pesos (%) 20 +-5 50 +-10 30 +-5

136 Anexo V: Competencia en el Conocimiento y la Interacción con el Mundo Físico


Evaluación General de Diagnóstico 2009
Marco de la evaluación

5. Ejemplo de unidad de evaluación para Educación Primaria


(final del segundo ciclo)

LAS TORMENTAS
En verano es frecuente que se produzcan tormentas. Si nos sorprende una tormenta
en el campo, debemos tomar las siguientes precauciones:
No acercarnos a grandes masas de agua, como embalses o lagos.
No resguardarnos debajo de los árboles.
No situarnos junto a torres metálicas, como postes de luz o tendidos eléctricos.

Todos estos elementos atraen los rayos y por tanto, es peligroso estar junto a ellos
durante una tormenta.

1. Subraya, a partir de la información del texto, la frase correcta:

A Los embalses protegen de los rayos.


B Las torres eléctricas rechazan los rayos.
C Las tormentas son frecuentes en verano.
D Los árboles ofrecen protección de los rayos.

2. Completa el siguiente texto utilizando las palabras:

una brisa, un huracán, un vendaval

Un viento suave es Un viento fuerte es

Un viento muy fuerte es

Anexo V: Competencia en el Conocimiento y la Interacción con el Mundo Físico 137


Evaluación General de Diagnóstico 2009
Marco de la evaluación

3. El aparato que mide la presión atmosférica se llama:

A Barómetro.
B Pluviómetro.
C Termómetro.
D Anemómetro.

4. Señala qué factor influye más en el clima de una determinada zona:

A La proximidad al mar.
B La altura de los edificios.
C El número de habitantes.
D Las carreteras de la zona.

5. Las altas o bajas temperaturas, las excesivas o nulas precipitaciones y el fuerte


viento pueden causar catástrofes naturales que, como podemos ver en los medios
de comunicación, son cada vez más frecuentes.

Explica a qué nos referimos cuando hablamos de cambio climático y cita dos
causas que lo provocan.

138 Anexo V: Competencia en el Conocimiento y la Interacción con el Mundo Físico


ANEXO VI

COMPETENCIA SOCIAL Y CIUDADANA

1. Presentación de la competencia social y ciudadana


De acuerdo con los Reales Decretos que establecen las enseñanzas mínimas, la competen-
cia social y ciudadana es la que permite a los ciudadanos comprender la realidad social en
la que viven, convivir y ejercer la ciudadanía democrática en una sociedad plural y com-
prometerse a contribuir a su mejora.

El ejercicio de esta competencia implica, adquirir conocimientos en torno a la pluralidad y


evolución de las sociedades actuales; los fundamentos, la organización y el funcionamien-
to de los estados y sociedades democráticas; establecer gradualmente un sistema de valo-
res autónomo; asumir y valorar los principales derechos y deberes que derivan de la
Declaración Universal de los Derechos Humanos, de la Convención sobre los Derechos del
Niño, de la Constitución española y de la legislación de las comunidades autónomas.

Se pretende, asimismo, obtener las habilidades y actitudes sociales para desarrollar la auto-
estima, la afectividad y la autonomía personal en relación con las demás personas; acep-
tar y practicar las normas de convivencia a través del ejercicio responsable de los derechos
y deberes ciudadanos; tomar decisiones y participar activamente en la construcción y
mejora de una sociedad democrática, solidaria y tolerante.

Los aspectos más destacados de la competencia social y ciudadana se organizan en torno


a tres núcleos clave:
• La autonomía personal, que permite al ciudadano obtener la información
pertinente, estar en condiciones de analizarla e interpretarla y comprenderla
con éxito y, en consecuencia, actuar, elaborar opiniones, juicios y tomar deci-
siones de forma razonada y responsable.
• La convivencia con los otros ciudadanos, que exige poder expresar ideas de
forma argumentada, estar en situación de poder comprender el punto de vista

Anexo VI: Competencia Social y Ciudadana 139


Evaluación General de Diagnóstico 2009
Marco de la evaluación

de los otros (empatía) y resolver contradicciones y conflictos mediante el


diálogo y la mediación.
• La participación democrática, que exige conocer y asumir los derechos y
deberes de una sociedad democrática, participar y tomar iniciativas ciudada-
nas en el marco de la convivencia democrática, mostrar una actitud coopera-
tiva, solidaria y respetuosa con la sociedad, el medio natural y el patrimonio
cultural y hacer compatible la identidad propia con la de otros pueblos, lo cual
permite a su vez fomentar la convivencia y la paz.

2. Dimensiones de la competencia
De acuerdo con lo establecido en este marco de la evaluación, se organiza la competen-
cia social y ciudadana en las cuatro dimensiones consideradas: contextos y situaciones, pro-
cesos, bloques de contenidos y actitudes.

2.a. Contextos y situaciones

La competencia social y ciudadana se ejerce en las situaciones de la vida real. Para un alum-
no, estas situaciones son las que vive en el entorno personal y familiar y en la escuela con
compañeros y profesores, en primer lugar. Pero también en el entorno social que le sitúa
en su barrio, en su ciudad, en el país en el que vive y en el mundo global al cual pertene-
ce. En estas situaciones y contexto es en el que se deben situar las tareas que el alumno
debe realizar, para las que tiene que aplicar sus conocimientos y poder así ejercer como
individuo y como ciudadano que convive y participa en sus entornos.

En la evaluación, el contexto de una pregunta es el marco concreto en el que se presenta


una situación determinada, de modo que las pruebas de evaluación de la competencia
social y ciudadana deben referirse al entorno individual, al escolar y al social de los alumnos
y enmarcarse en situaciones cotidianas de la vida real. Es decir, las preguntas de las pruebas
de evaluación deben referirse a circunstancias concretas de esos diversos entornos.

Asimismo, estas situaciones en las que se puede aplicar la competencia social y ciudadana
se deben considerar también desde las diversas áreas de aplicación de la competencia: la
identidad, la autonomía y las responsabilidades personales, el entorno de la familia, los ami-
gos y las escuelas, la convivencia y la participación en los mismos. Pero, también en el entor-
no social, que representan los servicios públicos que rodean a los alumnos, las instituciones,
los derechos, deberes ciudadanos que comparte el alumno como miembro de la sociedad
y que establecen la Constitución, la legislación autonómica y las normativas locales.

140 Anexo VI: Competencia Social y Ciudadana


Evaluación General de Diagnóstico 2009
Marco de la evaluación

2.b. Procesos

En la competencia social y ciudadana se consideran tres grupos de habilidades y capaci-


dades que constituyen los denominados procesos en este marco: la capacidad para utili-
zar la información, la capacidad para comprender los hechos sociales y la capacidad para
convivir en sociedad. Por ello, los procesos de la competencia social y ciudadana son los
siguientes:

• Utilizar la información: obtener, analizar y sintetizar la información. Es decir,


realizar observaciones, analizar críticamente la información, y reorganizar y sinte-
tizar la información recogida.

• Comprender los hechos sociales: analizar, explicar e interpretar los hechos


sociales. Es decir, explicar e interpretar situaciones y hechos personales y socia-
les; analizar críticamente situaciones y explicar hechos sociales a partir de su ori-
gen e interrelación causal. Identificar y evaluar las posibles interpretaciones de
los hechos sociales. Realizar predicciones de las consecuencias y de la evolución
de situaciones y hechos sociales.

• Convivir en sociedad: comunicarse, dialogar y participar. Se trata de expresar


las propias ideas, defender y asumir los puntos de vista propios, con autonomía
y de acuerdo con el propio sistema de valores. Comprender y respetar el punto
de vista de los otros, y utilizar el diálogo para favorecer los acuerdos. Tener en
cuenta las motivaciones e intereses individuales y colectivos, para explicar los
hechos sociales, avanzar en el diálogo, etc. Trabajar en equipo y colaborar con
los demás. Participar en la toma de decisiones de grupo, favorecer los acuerdos
y asumir sus obligaciones.

La adquisición de estas destrezas y habilidades, como se ha señalado para el resto de las


competencias básicas, es progresiva, por lo que su grado de adquisición se amplía de los
10 a los 14 años.

2.c. Bloques de contenidos

Los contenidos de la competencia social y ciudadana se organizan en las dos etapas en un


ámbito relacionando las persona y la organización social, que se desarrolla en tres bloques
de contenidos: el individuo, la sociedad y la organización social y en otro ámbito correspon-
diente a las sociedades en el pasado y a las sociedades democráticas actuales y a los dere-
chos humanos.

Anexo VI: Competencia Social y Ciudadana 141


Evaluación General de Diagnóstico 2009
Marco de la evaluación

EDUCACIÓN PRIMARIA

El individuo
• Identidad personal.
• Autoestima y asertividad.
• Autonomía y responsabilidad.
• Historia personal.

La sociedad
• El entorno próximo: familia, amigos, escuela.
• Las normas de convivencia: roles, derechos y deberes en los grupos.
• La participación en la familia y en la escuela.
• Situaciones y conflictos de convivencia.

La organización social
• Las instituciones del municipio, de las comunidades autónomas y del
Estado español.
• Las Administraciones como garantías de los servicios públicos.
• Responsabilidades, roles, derechos y deberes ciudadanos.
• Las normas de convivencia establecidas por la Constitución española y la
legislación autonómica.

El pasado
• Aproximación a las sociedades históricas.
• El papel de los hombres y mujeres en la historia.
• Evolución de las sociedades en hechos históricos relevantes.

El presente
• Los fundamentos de la democracia.
• La diversidad y el interculturalismo social.
• Los derechos humanos universales.

142 Anexo VI: Competencia Social y Ciudadana


Evaluación General de Diagnóstico 2009
Marco de la evaluación

EDUCACIÓN SECUNDARIA OBLIGATORIA (ESO)

El individuo
• Identidad personal.
• Autoestima y asertividad.
• Autonomía y responsabilidad.
• Historia personal.

La sociedad y la organización social


• El proceso de socialización y sus agentes: familia, amigos, escuela,
trabajo, medios de comunicación social.
• Los grupos, categorías e instituciones sociales.
• La regulación del comportamiento: derechos y deberes.
• Las desigualdades sociales.
• Los conflictos sociales y formas de resolución.

Las sociedades humanas en el pasado


• Organización política, económica y social.
• Cambios y permanencias.
• Aportación de las diversas culturas al progreso de la humanidad.

Las sociedades actuales y los sistemas democráticos


• Logros y problemas.
• Globalidad, pluralidad, diversidad y sostenibilidad.
• Ciudadanía global e identidad local.
• Funcionamiento democrático: instituciones, leyes, derechos y deberes
reconocidos por las instituciones.
• La Constitución española, y los Estatutos de autonomía.
• Los derechos humanos.
• La Declaración Universal de Derechos Humanos.

2.d. Actitudes relacionadas con la competencia social y ciudadana

La actitud de los ciudadanos es fundamental en la determinación de las normas de convi-


vencia y su aplicación y respeto, así como en el respeto de los derechos individuales y colec-
tivos. Por tanto, determinadas actitudes son componentes básicos de esta competencia: el

Anexo VI: Competencia Social y Ciudadana 143


Evaluación General de Diagnóstico 2009
Marco de la evaluación

sentido de pertenencia a una comunidad y participar en ella, el reconocimiento, la acep-


tación y la práctica de las normas de convivencia democráticas, el reconocimiento de la
igualdad de derechos entre hombres, mujeres y diferentes colectivos, valorar el diálogo.

Uno de los objetivos fundamentales de la educación es que el alumnado desarrolle actitu-


des cívicas y adopte un comportamiento ciudadano que le permita ejercer sus derechos y
sus responsabilidades y convivir en libertad.

Para la evaluación de las actitudes se puede utilizar tanto el cuestionario del alumnado
como las preguntas contextualizados para recoger información sobre las actitudes del alum-
nado en esta competencia.

Se consideran las mismas actitudes en ambas etapas educativas, pero se tiene en cuenta
que su consecución es progresiva. Por tanto, al igual que en otras competencias, para los
alumnos de 4º de Educación Primaria se considera que han desarrollado las actitudes pre-
vistas si muestran receptividad y formulan respuestas adecuadas: son conscientes, recono-
cen, muestran disposición. En 2º de ESO se trata de verificar si además los alumnos acep-
tan los valores democráticos, asumen compromisos y participan, respetan los derechos de
los demás y tienden a utilizar el diálogo para la resolución de conflictos.

Pueden resumirse las actitudes relacionadas con la competencia social y ciudadana del
siguiente modo:

• Conocerse y valorarse a uno mismo.


• Tener sentido de pertenencia a los grupos sociales de los que forma parte.
• Reconocer, aceptar y practicar las normas y deberes de la convivencia
humana.
• Valorar las diferencias y aportaciones de las diferentes culturas a la humanidad.
• Reconocer los valores de la democracia y derechos humanos como los
principios rectores de la convivencia.
• Reconocer la igualdad de derechos entre el hombre y la mujer y los diferentes
colectivos humanos.
• Valorar la cooperación y el diálogo como forma de evitar y resolver conflictos.
• Participar activamente de forma democrática, solidaria e integradora en la
colectividad social (ciudadanía activa y democrática).

144 Anexo VI: Competencia Social y Ciudadana


Evaluación General de Diagnóstico 2009
Marco de la evaluación

3. Cuadros de relaciones

3.a. Cuadros en los que se relacionan, a través de los criterios de evaluación, los procesos
y los bloques de contenidos

Cuadro de relaciones de Educación Primaria

Procesos
Bloques
contenidos Obtener, analizar y sintetizar Analizar, explicar e interpretar Comunicarse, dialogar
la información los hechos sociales y participar
• Participar en las situaciones de
comunicación del aula, respetando
1er ciclo, 1

las normas del intercambio:


Lng.,

guardar el turno de palabra,


escuchar, mirar al interlocutor,
mantener el tema.

• Participar y disfrutar en juegos


1er ciclo, 5

ajustando su actuación, tanto en lo


EF,

que se refiere a aspectos motores


como a aspectos de relación con
los compañeros y compañeras.

• Participar del juego y las


2º ciclo, 5

actividades deportivas con


EF,
C1. El individuo

conocimiento de las normas


mostrando una actitud de
aceptación hacia las demás personas.
2º ciclo, 4

• Actuar de forma coordinada y


cooperativa para resolver retos o
EF,

para oponerse a uno o varios


adversarios en un juego colectivo.

• Utilizar los recursos expresivos del


2º ciclo, 7

cuerpo e implicarse en el grupo


para la comunicación de ideas,
EF,

sentimientos y representación de
personajes e historias, reales o
imaginarias.

• Identificar y explicar las


consecuencias para la salud y
2º ciclo, 3
C. Medio,

el desarrollo personal de
determinados hábitos de
alimentación, higiene, ejercicio
físico y descanso.

Anexo VI: Competencia Social y Ciudadana 145


Evaluación General de Diagnóstico 2009
Marco de la evaluación

Cuadro de relaciones de Educación Primaria (continuación)

Procesos
Bloques
contenidos Obtener, analizar y sintetizar Analizar, explicar e interpretar Comunicarse, dialogar
la información los hechos sociales y participar
1er ciclo, 4

• Localizar información concreta y


Lng.,

realizar inferencias directas en la


lectura de textos.

• Reconocer, identificar y poner


1er ciclo, 4
C. Medio,

ejemplos sencillos sobre las


principales profesiones y
responsabilidades que desempeñan
las personas del entorno.

• Reconocer y explicar, recogiendo


datos y utilizando aparatos de
medida, las relaciones entre algunos
2º ciclo, 1
C. Medio,
C2. La sociedad

factores del medio físico (relieve,


suelo, clima, vegetación...) y las for-
mas de vida y actuaciones de las
personas, valorando la adopción de
actitudes de respeto por el equili-
brio ecológico.

• Participar del juego y las


2º ciclo, 5

actividades deportivas con


EF,

conocimiento de las normas y


mostrando una actitud de
aceptación hacia los demás.

• Obtener información relevante


sobre hechos o fenómenos previa-
2º ciclo, 10
C. Medio,

mente delimitados, hacer prediccio-


nes sobre hechos naturales o socia-
les, integrando datos de observa-
ción directa e indirecta a partir de
la consulta de fuentes básicas y
comunicar los resultados.
2º ciclo, 5
C. Medio,

• Señalar algunas funciones de las administraciones y de organizaciones


C3. La organización social

diversas y su contribución al funcionamiento de la sociedad, valorando la


importancia de la participación personal en las responsabilidades colectivas.
2º ciclo, 4

• Localizar y recuperar información


Lng.,

explícita y realizar inferencias


directas en la lectura de textos.

• Interpretar e integrar las ideas


2º ciclo, 5

propias con la información


Lng.,

contenida en los textos de uso


escolar y social, y mostrar la
comprensión a través de la lectura
en voz alta.

146 Anexo VI: Competencia Social y Ciudadana


Evaluación General de Diagnóstico 2009
Marco de la evaluación

Cuadro de relaciones de Educación Primaria (continuación)

Procesos
Bloques
contenidos Obtener, analizar y sintetizar Analizar, explicar e interpretar Comunicarse, dialogar
la información los hechos sociales y participar
1er ciclo, 7
C. Medio,

• Ordenar temporalmente algunos


hechos relevantes de la vida
familiar o del entorno próximo.

• Explicar con ejemplos concretos,


2º ciclo, 7
C4. El pasado

C. Medio,

la evolución de algún aspecto de la


vida cotidiana relacionado con
hechos históricos relevantes,
identificando las nociones de
duración, sucesión y simultaneidad.

• Realizar interpretaciones
1er ciclo, 7

elementales de los datos


Mat.,

presentados en gráficas de barras.


Formular y resolver sencillos
problemas en los que intervenga
la lectura de gráficos.

• Poner ejemplos de elementos y


1er ciclo, 1
C. Medio,

recursos fundamentales del medio


físico (sol, agua, aire), y su relación
con la vida de las personas,
tomando conciencia de la
necesidad de su uso responsable.

• Identificar los medios de transpor-


1er ciclo, 6
C. Medio,

te más comunes en el entorno y


conocer las normas básicas como
peatones y usuarios de los medios
de locomoción.
C5. El presente

• Realizar interpretaciones
1er ciclo, 7

elementales de los datos


Mat.,

presentados en gráficas de barras.


Formular y resolver sencillos
problemas en los que intervenga la
lectura de gráficos.

• Recoger datos sobre hechos y


objetos de la vida cotidiana
2ºr ciclo, 7

utilizando técnicas sencillas de


Mat.,

recuento, ordenar estos datos


atendiendo a un criterio de
clasificación y expresar el
resultado en forma de tabla o
gráfica.
1er ciclo, 5
C. Medio,

• Reconocer algunas manifestacio-


nes culturales presentes en el ámbi-
to escolar, local y autonómico,
valorando su diversidad y riqueza.

Anexo VI: Competencia Social y Ciudadana 147


Evaluación General de Diagnóstico 2009
Marco de la evaluación

Cuadro de relaciones de 2º. de Educación Secundaria Obligatoria

Procesos
Bloques
contenidos Obtener, analizar y sintetizar Analizar, explicar e interpretar Comunicarse, dialogar
la información los hechos sociales y participar
• Identificar y explicar las
2º ciclo, EP,

consecuencias para la salud y


C. Medio,

el desarrollo personal de
3

determinados hábitos de
alimentación, higiene, ejercicio
físico y descanso.

• Identificar, como valores


3º ciclo, EP,

fundamentales de los juegos y la


práctica de actividades deportivas, el
EF

esfuerzo personal y las relaciones


que se establecen con el grupo y
actuar de acuerdo con ellos.
3º ciclo, EP,

• Opinar coherente y críticamente


con relación a las situaciones
EF

conflictivas surgidas en la práctica


de la actividad física y el deporte.

• Mostrar respeto por las diferencias


3º ciclo, EP,

y características personales propias


E. Ciud.,

y de sus compañeros y compañeras,


1

valorar las consecuencias de las


C1. El individuo

propias acciones y responsabilizarse


de las mismas.

• Argumentar y defender las propias


3º ciclo, EP,
E. Ciud.,

opiniones, escuchar y valorar


críticamente las opiniones de los
2

demás, mostrando una actitud de


respeto a las personas.

• Aceptar y practicar las normas


3º ciclo, EP,

de convivencia. Participar en la
E. Ciud.,

toma de decisiones del grupo,


3

utilizando el diálogo para favorecer


los acuerdos y asumiendo sus
obligaciones.

• Participar en las situaciones de


3º ciclo, EP,

comunicación del aula, respetando


Lng.,

las normas del intercambio: guardar


1

el turno de palabra, organizar el


discurso, escuchar e incorporar las
intervenciones de los demás.

• Manifestar actitudes de
2º curso ESO,

cooperación, tolerancia y
deportividad tanto cuando se
EF,

adopta el papel de participante


como el de espectador en la
práctica de un deporte colectivo.

148 Anexo VI: Competencia Social y Ciudadana


Evaluación General de Diagnóstico 2009
Marco de la evaluación

Cuadro de relaciones de 2º. de Educación Secundaria Obligatoria (continuación)

Procesos
Bloques
contenidos Obtener, analizar y sintetizar Analizar, explicar e interpretar Comunicarse, dialogar
la información los hechos sociales y participar
• Reconocer y explicar, recogiendo
datos y utilizando aparatos de
medida, las relaciones entre algunos
2º ciclo, EP,
C. Medio,

factores del medio físico (relieve,


suelo, clima, vegetación...) y las
1

formas de vida y actuaciones de


las personas, valorando la adopción
de actitudes de respeto por el
equilibrio ecológico.

• Obtener información relevante


sobre hechos o fenómenos
2º ciclo, EP,
C. Medio,

previamente delimitados, hacer


predicciones sobre hechos naturales
10

o sociales, integrando datos de


observación directa e indirecta a
partir de la consulta de fuentes
básicas y comunicar los resultados.
C2. La sociedad y la organización social

2º ciclo, EP,
C. Medio,

• Señalar algunas funciones de las administraciones y de organizaciones


diversas y su contribución al funcionamiento de la sociedad, valorando la
5

importancia de la participación personal en las responsabilidades colectivas.


3º ciclo, EP,
C. Medio,

• Concretar ejemplos en los que el


comportamiento humano influya de
1

manera positiva o negativa sobre el


medio ambiente.

• Analizar algunos cambios que las


3º ciclo, EP,
C. Medio,

comunicaciones y la introducción
de nuevas actividades económicas
4

relacionadas con la producción de


bienes y servicios han supuesto
para la vida humana y el entorno.
3er ciclo, EP,

• Realizar, leer e interpretar


Mat.,

representaciones gráficas de un
7

conjunto de datos relativos al


entorno inmediato.

• Poner ejemplos de servicios


3er ciclo, EP,

públicos prestados por diferentes


E. Ciud.,

instituciones y reconocer la
6

obligación de los ciudadanos de


contribuir a su mantenimiento a
través de los impuestos.

• Explicar el papel que cumplen los


servicios públicos en la vida de los
3er ciclo, EP,

ciudadanos y mostrar actitudes


E. Ciud.,

cívicas en aspectos relativos a la


7

seguridad vial, a la protección civil,


a la defensa al servicio de la paz y
a la seguridad integral de los
ciudadanos.

Anexo VI: Competencia Social y Ciudadana 149


Evaluación General de Diagnóstico 2009
Marco de la evaluación

Cuadro de relaciones de 2º. de Educación Secundaria Obligatoria (continuación)

Procesos
Bloques
contenidos Obtener, analizar y sintetizar Analizar, explicar e interpretar Comunicarse, dialogar
la información los hechos sociales y participar
• Explicar con ejemplos concretos,
2º ciclo, EP,

la evolución de algún aspecto de la


C. Medio,

vida cotidiana relacionado con


7

hechos históricos relevantes,


C3. Las sociedades humanas

identificando las nociones de


duración sucesión y simultaneidad,
en el pasado

3º ciclo, EP,

• Buscar, seleccionar y organizar


E. Art.,

informaciones sobre manifestaciones


1

artísticas del patrimonio cultural


propio y de otras culturas.
1º ESO,

• Utilizar las convenciones y


CSGH,

unidades cronológicas y las


5

nociones de evolución y cambio.


1º ESO,

• Realizar una lectura comprensiva


CSGH,

de información escrita de contenido


9

geográfico o histórico.
3er ciclo, EP,

• Realizar, leer e interpretar


Mat.,

representaciones gráficas de un
7
C4. Las sociedades actuales y los sistemas democráticos

conjunto de datos relativos al


entorno inmediato.

• Utilizar los números decimales,


3er ciclo, EP,

fraccionarios y los porcentajes


Mat.,

sencillos para interpretar e


3

intercambiar información en
contextos de la vida cotidiana.
3er ciclo, EP,

• Conocer los principales órganos


C. Medio,

de gobierno y las funciones del


5

Municipio, de las Comunidades


Autónomas, del Estado Español y
de la Unión Europea.

• Conocer algunos de los derechos


humanos recogidos en la
Declaración Universal de Derechos
3er ciclo, EP,
E. Ciud.,

Humanos y en la Convención sobre


los Derechos del Niño y los
4

principios de convivencia que


recoge la Constitución Española e
identificar los deberes más
relevantes asociados a ellos.

• Reconocer y rechazar situaciones


3er ciclo, EP,
E. Ciud.,

de discriminación, marginación e
injusticia e identificar los factores
5

sociales, económicos, de origen, de


género o de cualquier otro tipo que
las provocan.

150 Anexo VI: Competencia Social y Ciudadana


Evaluación General de Diagnóstico 2009
Marco de la evaluación

Cuadro de relaciones de 2º. de Educación Secundaria Obligatoria (continuación)

Procesos
Bloques
contenidos Obtener, analizar y sintetizar Analizar, explicar e interpretar Comunicarse, dialogar
la información los hechos sociales y participar
• Identificar y explicar algunos
ejemplos de los impactos que la
acción humana tiene sobre el
1º ESO,
CSGH,

medio natural, analizando sus


4

causas y efectos, y aportando


medidas y conductas que serían
necesarias para limitarlos.
y los sistemas democráticos
C4. Las sociedades actuales

• Extraer informaciones concretas e


2º ESO,

identificar el propósito en textos


Lng.,

escritos de ámbitos sociales


2

próximos a la experiencia del


alumnado.

• Identificar y poner ejemplos de


algunos aspectos sociales,
2º ESO,
Lng.,

culturales, históricos, geográficos o


8

literarios propios de los países


donde se habla la lengua extranjera
y mostrar interés por conocerlos.

• Organizar e interpretar
1º ESO,

informaciones diversas mediante


Mat.,

tablas y gráficas, e identificar


relaciones de dependencia en
situaciones cotidianas.

Anexo VI: Competencia Social y Ciudadana 151


Evaluación General de Diagnóstico 2009
Marco de la evaluación

3.b. Tabla resumen de las actitudes y su relación con los criterios de evaluación

Tabla de actitudes de Educación Primaria

Actitudes

C. Medio, Mostrar actitudes de cooperación en el trabajo en equipo.


2ºc, 9

Conocimiento EF,
Mostrar una actitud de aceptación hacia las demás personas.
2ºc, 5
y valoración
de uno mismo.
Las relaciones EF,
Implicarse en el grupo para la comunicación de ideas y sentimientos.
con los otros 2ºc, 7

EF,
Mostrar interés en el cuidado del cuerpo.
2ºc, 8

Sentido de
pertenencia
a los grupos C. Medio, Valorar la diversidad y riqueza de las manifestaciones culturales.
sociales de los que 1erc, 5
se forma parte

L. Ext., Reconocer la diversidad lingüística como elemento enriquecedor.


1erc, 7

L. Ext., Mostrar curiosidad e interés hacia las personas que hablan una lengua extranjera.
Valoración de 2ºc, 7
las normas de
la convivencia
C. Medio, Tomar conciencia de la necesidad del uso responsable de las fuentes de energía y de los
social y de la recursos del medio.
1erc, 1
diversidad y de
la organización
social C. Medio,
Adoptar actitudes de respeto por el equilibrio ecológico.
2ºc, 1

C. Medio, Valorar la importancia de hacer un uso responsable de las fuentes de energía del planeta.
2ºc, 8

Reconocimiento
de los valores de
la democracia y
de la igualdad de C. Medio, Valorar la importancia de la participación personal en las responsabilidades colectivas.
derechos de 2ºc, 5
los distintos
colectivos
humanos

152 Anexo VI: Competencia Social y Ciudadana


Evaluación General de Diagnóstico 2009
Marco de la evaluación

Tabla de actitudes de Educación Secundaria Obligatoria

Actitudes

EF,
2ºc, EP, 5 Mostrar una actitud de aceptación hacia las demás personas.

EF,
Implicarse en el grupo para la comunicación de ideas y sentimientos.
2ºc, EP, 7

EF,
Mostrar interés en el cuidado del cuerpo.
2ºc, EP, 8

C. Medio, Mostar una actitud cooperativa e igualitaria en el trabajo en equipo, apreciando el cuida-
Conocimiento 3ºc, EP, 9 do por la seguridad propia y la de los demás.
y valoración
de uno mismo. E. Art.,
Las relaciones Ajustar la propia acción a la de los otros miembros del grupo.
3ºc, EP, 4
con los otros
EF, Actuar de forma coordinada y cooperativa para resolver retos.
3ºc, EP, 3

Identificar algunas de las relaciones que se establecen entre la práctica correcta y habitual
EF,
del ejercicio físico y la mejora de la salud y actuar de acuerdo con ellas.
3ºc, EP, 8

Reconocer los sentimientos y emociones en las personas que le rodean, aceptar las
E. Ciud.,
diferencias interpersonales y adoptar actitudes constructivas y respetuosas ante las
3ºc, EP, 1
conductas de los demás.

Sentido de C. Medio,
Valorar la diversidad y riqueza de las manifestaciones culturales.
pertenencia a los 3ºc, EP, 5
grupos sociales
E. Ciud.,
de los que se 3ºc, EP,
Valorar el diálogo y la convivencia en las relaciones interpersonales en los diferentes gru-
forma parte pos de pertenencia.
1, 2 y 3

Valoración de L. Ext.,
Mostrar curiosidad e interés hacia las personas que hablan una lengua extranjera.
3ºc, EP, 7
las normas de la
convivencia
C. Nat., Valorar la necesidad de superar las desigualdades provocadas por las diferencias en el
social y de la 3ºc, EP, 4 acceso a bienes y servicios.
diversidad y de
la organización E. Ciud.,
social 3ºc, EP, 7
Mostrar actitudes cívicas en el cuidado del entorno y de los bienes comunes.

C. Medio,
Reconocimiento Valorar la importancia de la participación personal en las responsabilidades colectivas.
2ºc, EP, 5
de los valores de
la democracia y
C. Medio, Valorar el interés de la gestión de los servicios públicos para la ciudadanía y la importancia
de la igualdad 3ºc, EP, 5 de la participación democrática.
de derechos
de los distintos
colectivos E. Ciud., Rechazar las consecuencias de las marginaciones o discriminaciones derivadas de situacio-
humanos 3ºc, EP, 5 nes injustas.

C. Medio, Valorar la importancia de hacer un uso responsable de las fuentes de energía del planeta.
2ºc, EP, 8

C. Medio,
Globalización 3ºc, EP, 1
Valorar el impacto positivo o negativo del ser humano sobre el medio ambiente.
y sostenibilidad
C. Medio, Valorar el impacto de las actividades humanas en el territorio y la importancia de su
3ºc, EP, 2 conservación.

Anexo VI: Competencia Social y Ciudadana 153


Evaluación General de Diagnóstico 2009
Marco de la evaluación

4. Matriz de especificaciones de la competencia

Matriz de especificaciones de Educación Primaria

Procesos

Bloques de contenidos Utilizar la Comprender los Convivir en Pesos (%)


información hechos reales sociedad

El individuo 23%

La sociedad 19%

La organización social 20%

El pasado 16%

El presente 22%

Pesos (%) 32% 34% 34% 100%

Matriz de especificaciones de Secundaria Obligatoria

Procesos

Bloques de contenidos Utilizar la Comprender los Convivir en Pesos (%)


información hechos reales sociedad

El individuo 25%

La sociedad 20%

La organización social 20%

El pasado 15%

El presente 20%

Pesos (%) 30% 35% 35% 100%

154 Anexo VI: Competencia Social y Ciudadana


Evaluación General de Diagnóstico 2009
Marco de la evaluación

5. Ejemplo de unidad de evaluación para Educación Primaria


(final del segundo ciclo)

EL PASADO DE LA
EDUCACIÓN DE LAS
NIÑAS EN ESPAÑA

Hasta hace algunos años, la educación


de las niñas era muy distinta a la de los
niños. Iban a escuelas separadas. Allí
aprendían a leer, a escribir, algo de
matemáticas básicas y de religión.
Mientras los niños, además de esto, estudiaban economía, ciencias, dibujo, etc., las
niñas aprendían a coser o cocinar.

Los estudios de bachillerato y los universitarios estaban prohibidos para las chicas.
Además, se consideraba peligroso e inmoral que las chicas salieran de su casa y fue-
ran a clase junto con los chicos.

La historia de Concepción Arenal, que nació en 1820 y murió en 1893, es un buen


ejemplo de cómo luchar contra esa situación y estudiar lo que quería. Tuvo que dis-
frazarse de hombre y colarse en las clases de la Universidad de Madrid, en donde las
mujeres tenían prohibida la entrada. Estudió Derecho. Y aunque las normas le impi-
dieron obtener el título de abogada, nada ni nadie pudo impedirle que fuera una de
las primeras mujeres pensadoras de España.

La lucha de Concepción Arenal y otras mujeres como ella ayudó a que el gobierno
iniciara unos mínimos cambios que permitieron a unas pocas mujeres estudiar en
institutos de bachillerato y en Universidades. Más adelante, durante el siglo XX, el
número de chicas en la Universidad fue creciendo lentamente hasta los años 1960.

A partir de esos años comenzaron a ser ya muy numerosas. En los últimos cuarenta
años la situación en educación ha cambiado mucho y hoy en día hay incluso más
mujeres que hombres estudiando en la Universidad.

1. Concepción Arenal:

A Fue profesora de la Universidad.


B Terminó su carrera y fue una famosa abogada.
C Terminó su carrera y llegó a ser una famosa escritora.
D Fue a la Universidad aunque no obtuvo el título por ser mujer.

Anexo VI: Competencia Social y Ciudadana 155


Evaluación General de Diagnóstico 2009
Marco de la evaluación

2. ¿Por qué crees que es importante la figura de Concepción Arenal? Porque…

A fue muy famosa en su época.


B consiguió lo que se propuso y estudió Derecho.
C llevan su nombre calles, plazas y centros de estudio.
D gracias a ella, muchas mujeres siguieron su lucha y lograron estudiar.

3. Según el texto, ¿ha habido algún cambio en la incorporación de las mujeres a los
estudios universitarios desde la época de Concepción Arenal hasta ahora?

A Sí. Ahora son los hombres los que no estudian.


B No. Las mujeres siguen sin poder realizar estudios universitarios.
C No. Sigue habiendo más hombres que mujeres en la Universidad.
D Sí. Actualmente hay más mujeres que hombres estudiando en la
Universidad.

4. Escribe tres tipos de comportamientos que discriminen a unas personas de otras


a causa de:

Su nacimiento

Su raza

Su religión

156 Anexo VI: Competencia Social y Ciudadana

También podría gustarte